You are on page 1of 142

5

Transitive Relations:
Definition:A relation R on a set A is called transitive if whenever (a, b)R and (b,
c)R, then (a, c)R i.e.
(a, b)R and (b, c)R  (a, c)R.
R = {(1, 1), (1, 2), (2, 2), (2, 1), (3, 3)} is a transitive relation.
R = {(2, 4), (4, 3), (2, 3), (4, 1)} is not a transitive relation.

Equivalence Relation:
Definition: A relation on a set A is called an equivalence relation if it is reflexive,
symmetric, and transitive.

Function:
A relation from a set A to another set B is called a function if it relates each element of
set A to a unique element of set B.

Injective Function:
A function 𝒇 from set A to set B is called an injective function or into function if no
two distinct element of set A be related to the same element of set B under this map 𝒇,
i.e. 𝑥 ≠ 𝑦  𝑓(𝑥) ≠ 𝑓(𝑦)
i.e. 𝑓(𝑥) = 𝑓(𝑦)  𝑥 = 𝑦.
Surjective Function or Onto Map:
A function 𝒇 from set A to set B is called a Surjective function or Onto function if
every element of set B has a pre-image in set A under the function𝒇, i.e. Range𝒇 = B.

Bijective Map or One-One Onto Map:


A function 𝒇 from set A to set B is called a Bijective Map if it is both injective as well
as surjective.
MCQ:
1
1). Let f : R → R be defined by f (x) = 𝑥 ∀ x ∈ R. Then f is:

(A) one-one (B) onto (C) bijective (D) f is not defined

ANS: For x=0, f (x) is not defined. Hence f (x) is a not define function.
2). Domain of the function in real number defined by 𝑓(𝑥)=√4 − 𝑥 2 is

(A) (-2, 2) (B) [-2, 2) (C) [-2, 2] (D) (-2, 2]


ANS: (C) [-2, 2]
3). Assertion (A): If n (A) =p and n (B) = q then the number of relations from A to B is 2pq.
Reason(R): A relation from A to B is a subset of A x B.
6

A. Both A and R are true and R is the correct explanation of A


B. Both A and R are true but R is NOT the correct explanation of A
C. A is true but R is false
D. A is false but R is true
ANS; A
4). Let f : R → R be the function defined by f (x) = 4x – 3 ∀ x ∈ R. Then write f –1.
𝒚+𝟑
ANS: f (x) = 4x – 3 = y (say) then 4x = y + 3 ⇒x=
𝟒
𝒚+𝟑 𝒙+𝟑
Hence f –1 (y) = ⇒ f –1 (x) =
𝟒 𝟒

1 MARK QUESTIONS
1. Check whether the relation R defined in the set {1, 2, 3, 4} as
R = {(1, 2), (2, 2), (1, 1), (4, 4), (1, 3), (3, 3), (3, 2)} is Reflexive, Symmetric and
Transitive.
Ans: Not Reflexive and Symmetric but Transitive)
2. Show that the relation R in the set {1, 2, 3} given by R = {(1, 2), (2, 1)} is
symmetric but neither reflexive nor Transitive.
ANS: (1, 1) does not belongs to R)
3𝑥−4
3. If f is an invertible function, find the inverse of f(x)=
5
2 Marks Questions:
1). Show that the relation R in the set {1, 2, 3} given by R = {(1, 2), (2, 1)} is
symmetric but neither reflexive nor Transitive. ( (1,1) not belons to R)
ANS: Relation R = {(1, 2), (2, 1)} is symmetric as (a, b)R⇒(b, a)R,
It is not transitive as (1, 2)R and (2, 1)R but (1, 1)R.
It is not reflexive as 1{1, 2, 3} but (1, 1)∉ 𝑹.
Relation R = {(a, b): a ≤ b, a, b ∈R} is reflexive as
𝒙≤𝒙 𝒊. 𝒆. 𝒙𝑹𝒙 ∀𝒙 ∈ 𝑹.
It is transitive as
𝒙 ≤ 𝒚, 𝒚 ≤ 𝒛  𝒙 ≤ 𝒛 𝒊. 𝒆. 𝒙𝑹𝒚, 𝒚𝑹𝒛  𝒙𝑹𝒛 .
But, it is not symmetric as 𝟐 ≤ 𝟓 𝒃𝒖𝒕 𝟓 > 𝟐.
2) Show that R defined as R = {(a, b): a ≤ b, a, b ∈R} is Reflexive and Transitive but
not Symmetric.

3). Let A = {1, 2, 3}, B = {4, 5, 6, 7} and let f = {(1, 4), (2, 5), (3, 6)} be a function
from A to B. whether f is one-one? Whether it is onto?
Let 𝐴 = {1, 2, 3}, 𝐵 = {4, 5, 6, 7} and let 𝑓 = {(1, 4), (2, 5), (3, 6)}.
The 𝑓 is one-one as distinct elements of set 𝐴 is related to distinct elements of 𝐵.
7

This function 𝑓 is not onto as 7∈ 𝑩 which do not have pre-image in 𝑨.

3x − 2
4). If f is an invertible function, find the inverse of f(x) = .
5
3x − 2
ANS: For invertible function f(x) = .
5
3x − 2 5𝑦+2
Let y = 𝑥= .
5 3
5𝑥+2
So the inverse function of the function is given by 𝑓 −1 such that 𝑓 −1 (𝑥) = .
3

5) Let C be the set of complex numbers. Prove that the mapping f : C →R given by
f (z) = |z|, ∀ z ∈ C, is neither one-one nor onto.
ANS: Given, by f (z) = |z|, ∀ z ∈ C
f (1) = |1| = 1
f (-1) = 1
f (1) = f(-1)
but 1 ≠ −1 So, f (z) is not one-one.
Also, f (z) is not onto as there is no pre-image for any negative element of R under the
Mapping f ( z).
6). If A = {1, 2, 3, 4} define relations on A which have properties of being:
(a) reflexive, transitive but not symmetric
(b) reflexive, symmetric and transitive.

ANS: Given that A = {1, 2, 3, 4}


(a) Let R1 = { (1,1) , (1, 2) , (2,3) , (2,2) , (1,3) , (3,3)}.
R1 is reflexive, since, (1,1) (2,2) (3,3) lie is R1
Now, (1, 2) ∈ R1 (2, 3) ∈ R1 ⇒ (1, 3) ∈ R1
Hence, R1 is also transitive, but (1, 2) ∈ R1, ⇒ (2, 1) ∉ R So, it is not symmetric.

(b) Let R2 = { (1,2) , (2,1) ,(1,1), (2, 2) , (3,3) , (1,3) , (3,1) , (2,3) }
Hence, R2 is reflexive, symmetric and transitive.
7). Let R be relation defined on the set of natural number N as follows:
R = {(x, y): x ∈ N, y ∈ N, 2x + y = 41}. Find the domain and range of the relation R. Also
verify whether R is reflexive, symmetric and transitive.
ANS: Given that, R = {(x, y): x ∈ N, y ∈ N, 2x + y = 41}
Domain = {1, 2, 3……….20}
Range = {1, 3, 5, 7…………………39}
R = {(1, 39), (2, 37), (3, 35) ...... (19, 3), (20,1)}
8

R is not reflexive as (2, 2) ∉ R 2 x 2 + 2 ≠ 41


So, R is not symmetric As (1, 39) ∈ R but (39, 1) ∉ R So, R is not transitive.
As (11, 19) ∈ R, (19, 3) ∈ R But (11,3) ∉R
Hence, R is neither reflexive, nor symmetric and nor transitive.

3 MARKS QUESTIONS & 5 MARKS QUESTIONS

1). Prove that the relation R in the set A = {1,2,3,4,5} given by R = {(a, b): a, b∈A and
|𝑎 − 𝑏|is even} is an equivalence relation
ANS: Consider any a, b, c  A where A = {1, 2, 3, 4, 5}
Since |a – a| = 0, which is even  a − a is even
 (a, a )  R  R is reflexive. R is Symmetric
(a, b)  R  a − b is even  −(b − a) is even  b − a is even
 (b, a )  R  R is symmetric R is transitive
( a, b)  R
and (b, c)  R
 a − b is even and b − c is even
 a − b is even and b − c is even
 a − b + b − c is even
 a − c is even  a − c is even
 ( a, c )  R  R is transitive
Hence, R is an equivalence relation.

2). Check whether the function f: R→R defined by f(x) = 4+ 3x is one – one and onto.
If so, find the inverse of f.
ANS: f is One-one function if f(x1) = f(x2) → x1 = x2 for x1, x2 ∈ R
f(x1) = f(x2) 4 + 3x1 = 4 + 3x2
x 1 = x2 f is one-one
Let y∈ R and y = f(x)
𝑦−4 𝑥−4
y = 4 + 3x x= f is onto and f-1 (x) =
3 3
3). Define the relation R in the set 𝑁 × 𝑁 as follows: For (a, b), (c, d) ∈ 𝑁 × 𝑁, (a, b) R (c, d)
iff ad = bc. Prove that R is an equivalence relation in 𝑁 × 𝑁.
ANS: Let (𝑎, 𝑏) ∈ 𝑁 × 𝑁. Then we have ab = ba (by commutative property of N)
⟹ (𝑎, 𝑏) 𝑅 (𝑎, 𝑏) Hence, R is reflexive.
Let (𝑎, 𝑏), (𝑐, 𝑑) ∈ 𝑁 × 𝑁 such that (a, b) R (c, d).
Then ad = bc ⟹ 𝑐𝑏 = 𝑑𝑎 (by commutative property of multiplication of N)
⟹ (𝑐, 𝑑) 𝑅 (𝑎, 𝑏) Hence, R is symmetric.
9

Let (𝑎, 𝑏), (𝑐, 𝑑), (𝑒, 𝑓) ∈ 𝑁 × 𝑁


(a, b) R (c, d) then ad = bc ……. (1)
(c, d) R (e, f) then cf = de ….……(2)
From (1), (2) 𝑎𝑑𝑐𝑓 = 𝑏𝑐𝑑𝑒 ⟹ 𝑎𝑓 = 𝑏𝑒 ⟹ (𝑎, 𝑏)(𝑒, 𝑓) Hence, R is transitive.
Since, R is reflexive, symmetric and transitive, R is an equivalence relation on 𝑁 × 𝑁.
𝒙−𝟐
4). Let A = R – {3}, B = R {– 1}. If f: A → B be defined by f (x) = ∀ x ∈ A. Then, show
𝒙−𝟑
that f is bijective.
𝒙−𝟐
ANS: Given A = R – {3}, B = R {– 1} f: A → B be defined by f (x) = ∀x∈A
𝒙−𝟑

For injectivity if f(x1) = f(x2) → x1 = x2 for x1, x2 ∈ R


𝒙𝟏 −𝟐 𝒙𝟐 −𝟐
Let f (𝑥1 ) = f (𝑥2 ) ⟹ = ⟹ (𝑥1 − 2)( 𝑥2 − 3) = (𝑥2 − 2) (𝑥1 − 3)
𝒙𝟏 −𝟑 𝒙𝟐 −𝟑

⟹ 𝑥1 = 𝑥2 So, f (x) is an injective


𝒙−𝟐 𝟑𝒚 − 𝟐
For surjectivity Let y = ⟹ xy – 3y = 𝑥 − 2 ⟹x=
𝒙−𝟑 𝒚−𝟏

⟹ So, f (x) is surjective function. Since f is one & Onto f(x) is a bijective function.
CASE STUDY

In two different societies, there are some school going students – including girls as well as
boys. Satish forms two sets with these students, as his college project.

Let 𝐴 = {𝑎1 , 𝑎2 , 𝑎3 , 𝑎4 , 𝑎5 } and 𝐵 = {𝑏1 , 𝑏2 , 𝑏3 , 𝑏4 } where 𝑎𝑖 ′𝑠, 𝑏𝑖 ′𝑠are the school going students
of first and second society respectively.

Using the information given above, answer the following question:


1. Satish wishes to know the number of reflexive relations defined on set 𝐴. How many such
relations are possible?

a) 0 b)25 c)210 d)220


2. Let 𝑅: 𝐴 → 𝐴, 𝑅 = {(𝑥, 𝑦): 𝑥 𝑎𝑛𝑑 𝑦 𝑎𝑟𝑒 𝑠𝑡𝑢𝑑𝑒𝑛𝑡𝑠 𝑜𝑓 𝑠𝑎𝑚𝑒 𝑠𝑒𝑥}. Then relation R is

a) Reflexive only b) Reflexive and symmetric but not transitive

c) Reflexive and transitive but not symmetric d) An equivalence relation

3. Satish and his friend Rajat are interested to know the number of symmetric relations defined
on both the sets A and B, separately. Satish decides to find the symmetric relation on set A,
while Rajat decides to find symmetric relation on set B. What is difference between their
results?
10

a)1024 b)210 (15) c)210(31) d)210(63)

4. Let 𝑅: 𝐴 → 𝐵, 𝑅 = {(𝑎1 , 𝑏1 ), (𝑎1 , 𝑏2 ), (𝑎2 , 𝑏1 ), (𝑎3 , 𝑏3 ), (𝑎4 , 𝑏2 ), (𝑎5 , 𝑏2 )}, then 𝑅 is

a) Neither one-one nor onto b) One-one but not onto

c) Only onto but not one-one d) One-one and onto both

5. To help Satish in his project, Rajat decides to form onto function from set A to itself. How
many such functions are possible?

a) 342 b)243 c)729 d)120

ANS: 1). (d) 2. (d) 3. (c) 4. (a) 5. (d)

𝟒𝒙 + 𝟑
5). Show that the function f : R –{-4/3) → R –{4/3) defined by f(x) = is one-one
𝟑𝒙 + 𝟒
and onto.Hence find inverse of f and find f-1(0) and x such that f-1(x) = 2.
ANS: Let x1, x2 ∈ R –{-4/3) and f(x1) = f(x2)
𝟒𝒙𝟏+𝟑 𝟒𝒙𝟐+𝟑
=
𝟑𝒙𝟏+𝟒 𝟑𝒙𝟐+𝟒
(4x1+3)(3x2+4) = (3x1+4) (4x2+3) x1 - x2 = 0 x1 = x2 Hence f is one-one.
4𝑥+3
Let y∈ R – {-4/3) and y =
3𝑥+4
4𝑦−3
3xy +4y = 4x + 3 x=
4−3𝑦
Hence f is onto and so bijective
𝟒𝒚 − 𝟑
f-1 (y) = f-1 (0) = -3/4
𝟒 − 𝟑𝒚
𝟒𝒙 − 𝟑
f-1(x) = 2  = 2  4x – 3 = 8 – 6x x = 11/10
𝟒 − 𝟑𝒙

2 – INVERSE TRIGONOMETRIC FUNCTIONS


SOME IMPORTANT RESULTS AND CONCEPTS
FUNCTIONS DOMAIN RANGE (Principal Value Branch)
Sin-1: [-1 1] [ -π/2 π/2 ]
Cos- :1
[-1 1] [ 0 π]
Cosec-1: R-(-1 1) [-π/2 π/2 ] -{0}
-1
Sec : R-(-1 1) [ 0 π]- { π/2 }
Tan :-1
R (-π/2 π/2 )
Cot-1: R ( 0 π)
PROPERTIES OF INVERSE TRIGONOMETRIC FUNCTIONS
Sin-1 (sinx) = x if x Ɛ[ -π/2, π/2 ] Cos-1 (cosx)= x if x Ɛ[ 0, π]
Tan-1 (tanx) = x if x Ɛ(-π/2 , π/2 ) Cosec-1(cosecx)= x if x Ɛ [-π/2, π/2 ] -{0}
11

Sec-1 (secx) = x , if x Ɛ[ 0 π]- { π/2 } Cot-1(cotx)= x, if x Ɛ ( 0 π)


Sin (Sin-1 x ) = x , if x Ɛ[ -1 1 ] cos(Cos-1 x) = x, f x Ɛ[ -1 1]
Tan (Tan-1 x ) = x if x ƐR cosec(Cosec-1 x)= x, if x Ɛ [-∞, -1 ] U [1, ∞)
Sec (Sec-1 x ) = x if x Ɛ [-∞ -1 ] U [1 ∞) cot (Cot-1 x)= x, if x Ɛ R
sin-1(x) = cosec-1(1/x) & cosec-1(x)= Sin-1(1/x)
cos -1(x) = sec-1 (1/x) & sec-1 (x)= Cos -1(1/x)
tan-1(x)= cot-1 (1/x) & cot-1 (x) = Tan-1(1/x)
sin-1(-x)= - Sin-1x tan-1(-x)= - Tan-1x
cosec-1(-x)= - cosec-1x cos -1(-x)= π - Cos -1(x)
sec-1 (-x)= π - sec-1 (x) cot -1(-x)= π - Cot -1(x)
sin-1(x)+ Cos -1(x)= π/2 tan-1 x+ cot-1 x= π/2 sec-1 x+ cosec-1 x= π/2
2 tan-1 x = tan-1 ( 2x/1- x2 ) = Cos -1 (1- x2 / 1+ x2 ) = tan-1 ( 2x /1+ x2 )
tan-1 x + tan-1 y = tan-1 ( x+y / 1-xy) if xy <1
tan-1 x - tan-1 y = tan-1 ( x-y / 1+xy) if xy >1

LEVEL 1: WRITE THE PRINCIPAL VALUE OF THE FOLLOWING


Q1. Sin-1 (-1/2)
ANS: Sin-1 {sin(-π/6)
.i.e - π/6 as this value belongs to the range of Sin-1 i.e [ -π/2 π/2 ]
Q2.cos-1 (√3/2)
ANS: Cos-1 (cos π/6) i.e π/6 which belongs to the range of Cos-1 i.e [ 0 π]
Q3. Tan-1 (- √3)
ANS: Let Tan-1 (- √3) = y
Then tan y = - √3 and tan y = - tan π/3
tan y = tan (-π/3) i.e (-π/3) which belongs to the range of tan-1x i.e (-π/2, π/2)
Q4. Cos-1(-1/√2)
ANS: Let . Cos-1(-1/√2)= y therefore cos y = (-1/√2)
Cos y = - cos π/4 but does not belongs to the rage [0, π] hence cos (π- π/4)= 3 π/4
Q5. Prove that: 2 tan-1 1/2 + tan-1 1/7 = tan-1 31/17
ANS: tan-1 2 [(1/2) / 1 - ¼ ] + tan-1 1/7
12

Tan-1 1/ ¾ + tan-1 1/7 i.e tan-1 4/3 + tan-1 1/7 = tan-1 31/17
√𝟏+𝒔𝒊𝒏𝒙 + √𝟏−𝒔𝒊𝒏𝒙
Q6. Prove that cot -1 ( ) = x/2 , x ∈ (0 π/4 )
√𝟏+𝒔𝒊𝒏𝒙 − √𝟏−𝒔𝒊𝒏𝒙

ANS: √(1 + sin 𝑥 ) = √( sin 2 x/2 + cos 2


x/2 + 2 sinx/2 X cos x/2 )
𝑥 𝑥
= √(cos + sin ) 2
2 2

Similarly
𝑥 𝑥
= √(1 − sin 𝑥 ) = √( sin 2 x/2 + cos 2
x/2 - 2 sinx/2 X cos x/2 ) = √(cos − sin ) 2
2 2
𝑥 𝑥 𝑥
(cos +sin 𝑥/2)+(cos −sin )
2 2 2
: simplifying cot -1 ( 𝑥 𝑥 𝑥 )
(cos +sin 𝑥/2)−+(cos −sin )
2 2 2

simplifying cot -1 ( 2cos x/2 ÷ 2 sin x/2 ) i.e cot -1 ( cot x/2 ) = x/2
√𝟏+𝒙 + √𝟏−𝒙
Q7. Prove that tan -1 ( ) = π/4 – x/2
√𝟏+𝒙 − √𝟏−𝒙

ANS: substitute x = cos 2θ therefore 2θ = cos -1 x or θ = 1/2 cos -1 x


√1+𝑐𝑜𝑠2θ + √1−cos 2θ
tan -1 ( )
√1+cos 2θ − √1−cos 2θ

1+ cos 2θ = 1+2 cos 2θ – 1 = 2 cos 2θ and 1- cos 2θ = 1-( 1- 2 sin 2θ ) = 2 sin 2θ


tan -1{ ( √2 cos 2θ + √2 sin 2θ )/ ( √2 cos 2θ - √2 sin 2θ )}
tan -1{ ( cos θ – sin θ) / ( cos θ + sin θ)}
dividing the numerator and denominator by cos θ
we get tan -1 { ( 1- tan θ) / ( 1+ tan θ)}
tan -1 ( tan (π/4 - θ) i.e π/4 - θ
π/4 - 1/2 cos -1 x

-1
Q8. If sin (sin 1/5 + cos -1 x) = 1 , find the x .
-1
ANS: sin 1/5 + cos -1 x = sin -1 1
sin -1
1/5 + cos -1 x = π/2, comparing with sin -1
x + cos -1 x= π/2 x = 1/5
𝒂𝒄𝒐𝒔𝒙 − 𝒃𝒔𝒊𝒏𝒙
Q9. Simplify: tan -1 ( )
𝒃𝒄𝒐𝒔𝒙 + 𝒂𝒔𝒊𝒏𝒙
𝑎𝑐𝑜𝑠𝑥−𝑏𝑠𝑖𝑛𝑥
Solution: we have tan -1 ( ) dividing Num and Deno by b cos x
𝑏𝑐𝑜𝑠𝑥+𝑎𝑠𝑖𝑛𝑥
𝑎 𝑎
Now we get tan -1 { ( - tanx) / ( 1+ tanx)}
𝑏 𝑏

We have tan-1 a/b - tan -1 (tanx) tan-1 a/b - x


13

3 – MATRICES
One marks and two marks questions .
1. A matrix has 12 elements. What possible orders it can have?
ANS: A matrix has 12 elements. 12can be factorized as:
12 = 1x12 = 2x6 = 3x4 = 4x3 = 6x2 = 12x1
Thus possible orders of matrix are: 1x12, 2x6, 3x4, 4x3, 6x2, 12x1
3 0 2
2. If a matrix A = [1 −5 7] , then find the order of matrix A.
8 4 9
3 0 2
ANS: Given matrix A= [1 −5 7] since there are 3 rows and 3 columns
8 4 9
therefore its order is 3x3.
7 2
3. If matrix A = | | , find its determinant.
1 3
7 2 7 2
Sol. Given A =| | , |𝐴| = det A = | |= 7x3 – 2x1 = 21 – 2 = 19
1 3 1 3
3 1
4. If a 2x2 matrix A = [ ] , verify: |3𝐴| = 32 IAI = 9 |𝐴| .
4 5
3 1 3 1
ANS: It is given that matrix A = [ ]  |𝐴| = | | = 15 – 4 = 11
4 5 4 5
3 1 9 3 9 3
3A = 3[ ]=| |  |3𝐴| =| | = 135 – 36 = 99
4 5 12 15 12 15
 |3𝐴| = 9 x |𝐴|
2 3
3 2 1
5. If a matrix A = [ ] and matrix B = [1 0] , then find: A + B1
4 −1 3
4 5
3 2 1 2 1 4 5 3 5
ANS: A + B1=[ ]+ [ ]=[ ]
4 −1 3 3 0 5 7 −1 8
6. If A is a 3x3 invertible matrix then what will be the value of K if
det (A-1) = (detA)k.
ANS: AA-1 = I  IAA-1I= |𝐼|  |𝐴| |𝐴|-1 = 1
 |𝐴|-1 = |𝐴|k  k=–1
5 1
7. If matrix A = [ ], then verify: A2 – 8A + 13I = 0 and hence find A3.
2 3
5 1
ANS: It is given that matrix A = [ ]
2 3
5 1 5 1 5 1 1 0
A2 – 8A + 13I = [ ][ ] – 8[ ] + 13 [ ]
2 3 2 3 2 3 0 1
14

25 + 2 5 + 3 40 8 13 0
=[ ]– [ ]+[ ]
10 + 6 2 + 9 16 24 0 13
27 − 40 + 13 8−8+0 0 0
=[ ]=[ ]=0
16 − 16 + 0 11 − 24 + 13 0 0
A2 – 8A + 13I = 0
Now A2 = 8A2 – 13A
To find A3, multiply by A on both sides of equations, we get
151 51
A3 = 8A2 – 13A  A3 = [ ]
102 49
2 1 3 0 9 1
8. Find matrix X if 5A – 7B + 3X = 0, where A = [ ]&B=[ ]
1 4 7 5 3 5
2 1 3 0 9 1
ANS: A=[ ] and B = [ ]. 5A – 7B + 3X = 0  3X = 7B – 5A
1 4 7 5 3 5
0 9 1 2 1 3
3X = 7 [ ]–5[ ]
5 3 5 1 4 7
0 63 7 10 5 15
=[ ]–[ ]
35 21 35 5 20 35
−10 58 −8
=[ ]
30 1 0
1 −10 58 −8
X= [ ]
3 30 1 0
9. A book store has 20 mathematics books, 15 physics books, and 12 chemistry books.
Their selling prices are Rs.300, Rs. 320, Rs. 340 each respectively .Find the total
amount the store will receive from selling all the items. Do you think that we must
make books our best friends? Why?
Sol. We can represent the given price information as the following Price Matrix:
300
Mathematics [320]
340
we can represent the given sale information as the following sale matrix:
𝑚𝑎𝑡ℎ𝑒𝑚𝑎𝑡𝑖𝑐𝑠 𝑝ℎ𝑦𝑠𝑖𝑐𝑠 𝑐ℎ𝑒𝑚𝑖𝑠𝑡𝑟𝑦
Book store [ ]
20 15 12
Then, the amount received by the book store from selling all the items can be obtained
300
by multiplying the sale matrix, which is given by: [20 15 12] [320]
340
15

= 6000 + 4800 + 4080 = 14880.


Hence total amount is Rs.14880.
Yes, we must make books our best friends because books help us increase our
knowledge.
𝑥 0 0 𝑎 0 0 𝑥𝑎 0 0
10. If P =[0 𝑦 0] and Q = [0 𝑏 0] , prove that PQ = [ 0 𝑦𝑏 0 ] =QP.
0 0 𝑧 0 0 𝑐 0 0 𝑧𝑐
𝑥 0 0 𝑎 0 0
Sol. Given P = [0 𝑦 0] and Q = [0 𝑏 0]
0 0 𝑧 0 0 𝑐
𝑥 0 0 𝑎 0 0 𝑥𝑎 0 0
Then PQ = [0 𝑦 0] [0 𝑏 0] = [ 0 𝑦𝑏 0 ]
0 0 𝑧 0 0 𝑐 0 0 𝑧𝑐
𝑎 0 0 𝑥 0 0 𝑥𝑎 0 0 𝑥𝑎 0 0
Also QP =[0 𝑏 0] [0 𝑦 0] = [ 0 𝑦𝑏 0 ] = [ 0 𝑦𝑏 0 ]
0 0 𝑐 0 0 𝑧 0 0 𝑧𝑐 0 0 𝑧𝑐
𝑥𝑎 0 0
Hence PQ = [ 0 𝑦𝑏 0 ] = QP.
0 0 𝑧𝑐
5 2 𝑎
11. If A = [𝑏 𝑐 −3] is a symmetric matrix, find a, b, c, d.
4 𝑑 −7
5 2 𝑎
Sol. Given A = [𝑏 𝑐 −3] since A is symmetric matrix AT = A
4 𝑑 −7
5 𝑏 4 5 2 𝑎
[2 𝑐 𝑑 ] = [𝑏 𝑐 −3]
𝑎 −3 −7 4 𝑑 −7
Thus corresponding elements of above matrices must be equal.
Hence a = 4, b = 2, c be any real number and d = – 3.

12. ( Value based question) There are two families F1 and F2 .There are 6 men 5

women and 2 children in family F1 and 3 men ,3women and4 children in family F2.
The recommended daily requirement for protein is Man:70g, women:50g, child:30g,
and for carbohydrates is man:500g, women:400g,:child:300: using matrix
multiplication ,calculate the total requirement of proteins and carbohydrates for each
16

of two families. What other nutrients should be included in the diet for a healthy
living? Mention any three.
So, we can represent the given family information for F1 and F2 as the following
𝑚𝑒𝑛 𝑤𝑜𝑚𝑒𝑛 𝑐ℎ𝑖𝑙𝑑𝑟𝑒𝑛
count matrix: [ 6 5 2 ].
3 3 4

4 – DETERMINANTS
Key facts :-
Only square matrices have determinants.
𝑎 𝑏 𝑎 𝑏
𝐿𝑒𝑡 𝑀𝑎𝑡𝑟𝑖𝑥 𝐴 = [ ] 𝑡ℎ𝑒𝑛 𝐷𝑒𝑡𝐴 = |𝐴| = | | = 𝑎𝑑 − 𝑏𝑐
𝑐 𝑑 𝑐 𝑑
To prove three points collinear, we show area of the triangle formed by three points is
zero
Very short answer questions
𝒔𝒊𝒏𝟐𝟎𝒐 −𝒄𝒐𝒔𝟐𝟎𝒐
Q1) Write the value of | |
𝒔𝒊𝒏𝟕𝟎𝒐 𝒄𝒐𝒔𝟕𝟎𝒐

𝒐
Sol) |𝒔𝒊𝒏𝟐𝟎𝒐 −𝒄𝒐𝒔𝟐𝟎𝟎 | = sin20o. cos70o + cos20o.sin70o = sin 90 = 1
𝒔𝒊𝒏𝟕𝟎 𝒄𝒐𝒔𝟕𝟎𝒐
Q2) If A is square matrix of order 3 such that |𝐴| = k, then write the value of |−𝐴|.
Sol) Since |𝐴| = k and order of A = 3
Hence |−| = (-1)3. |𝐴|
= -1×k =-k
Q3) For what value of x, the following matrix is singular?
𝟓−𝒙 𝒙+𝟏
| |
𝟐 𝟒
𝟓−𝒙 𝒙+𝟏
Sol). Let A = | | For A to be singular , |𝐴| =0
𝟐 𝟒
𝟓−𝒙 𝒙+𝟏
| |
𝟐 𝟒
4(5 – x) – 2(x + 1) = 0
20 – 4x – 2x – 2 = 0 18 = 6x x=3
𝑥 4
Q4) Find the value of x, [ ] is singular matrix
2 2𝑥
17

𝒙 𝟒
Sol). If A is singular, |𝐴| = 0 | | = 0 2x2 – 8 = 0 x2 = 4 x = +2 , -2
𝟐 𝟐𝒙
Short answer type questions
𝟏 𝟐
Q5) If A = | | , then find the value of k if |𝟐𝑨| = k|𝑨|.
𝟒 𝟐
𝟏 𝟐 𝟐 𝟒
Sol) Since A = | | 2A =| |
𝟒 𝟐 𝟖 𝟒
Given |𝟐𝑨| = k|𝑨|
𝟐 𝟒 𝟏 𝟐
| | = k| | 8 – 32 = k (2 – 8) – 24 = – 6k k=4
𝟖 𝟒 𝟒 𝟐

𝒙 𝒙 𝟑 𝟒
Q6). If | |=| | , then write the positive value of x .
𝟏 𝒙 𝟏 𝟐
𝒙 𝒙 𝟑 𝟒
Sol). | |=| | x2 – x = 6-4
𝟏 𝒙 𝟏 𝟐
x2 – x – 2 = 0 x = 2 or x = – 1 (not accepted) x=2
𝒙+𝟏 𝒙−𝟏 𝟒 −𝟏
Q7). If | |=| | , then write the value of x.
𝒙−𝟑 𝒙+𝟐 𝟏 𝟑
𝒙 + 𝟏 𝒙 − 𝟏 𝟒 −𝟏
Sol) Given: | |= | |
𝒙−𝟑 𝒙+𝟐 𝟏 𝟑
(x + 1) (x + 2) – (x – 1) (x – 3) = (12 + 1)
x2 + 2x + x + 2 – x2 + 3x + x – 3 = 13
7x – 1 = 13 7x = 14 x=2
Q8). Find the equation of the line joining A (1, 3) and B (0, 0) using determinants.
Also find k if D (k, 0) is a point such that the area of ∆ABD is 3 sq units.
Sol) Let P(x, y) be any point on the line AB. Then, ar (∆ ABP) = 0
𝟏 𝟑 𝟏
𝟏 𝟏
⇨ |𝟎 𝟎 𝟏| = 0 ⇨ [1(0 – y) – 3(0 – x) + 1(0 – 0)] = 0
𝟐 𝟐
𝒙 𝒚 𝟏
⇨ 3x – y = 0, which is the required equation of line AB.
Now area (∆ABD) = 3sq units
𝟏 𝟑 𝟏 𝟏 𝟑 𝟏
𝟏

𝟐
⌈𝟎 𝟎 𝟏⌉ = +3, – 3 ⇨ |𝟎 𝟎 𝟏| = +6, – 6
𝒌 𝟎 𝟏 𝒌 𝟎 𝟏
1(0 – 0) – 3(0 – k) +1 (0 – 0) = +6, – 6 3k = +6, – 6 k = +2, -2
18

Ex: Solve the system of equations using matrix method: 5x + 2y = 3, 3x + 2y = 5

SOL: Given Equations: 5x + 2y = 3, 3x + 2y = 5

The above system of equations can be written in matrix form

𝟓 𝟐 𝒙 𝟑 𝑨𝒅𝒋 𝑨
A=[ ] X = [ 𝒚] B = [ ] AX = B X = 𝑨−𝟏 𝑩 𝑨−𝟏 =
𝟑 𝟐 𝟓 |𝑨|

|𝑨| = 𝟏𝟎 − 𝟔 = 𝟒 |𝑨| ≠ 𝟎 𝑨−𝟏 exists

𝟐 −𝟐 𝑨𝒅𝒋 𝑨 𝟏 𝟐 −𝟐
AdjA = [ ] 𝑨−𝟏 = = 𝟒[ ]
−𝟑 𝟓 |𝑨| −𝟑 𝟓
𝟏 𝟐 −𝟐 𝟑 𝟏 𝟔 − 𝟏𝟎 𝟏 −𝟒 −𝟏 𝒙
AX = B X = 𝑨−𝟏 𝑩 = 𝟒 [ ][ ] = [ ] = 𝟒 [ ] = [ ] = [ 𝒚]
−𝟑 𝟓 𝟓 𝟒 −𝟗 + 𝟐𝟓 𝟏𝟔 𝟒

The solution is 𝒙 = −𝟏, 𝒚 = 𝟒

CHECK: Sub in Equations (1); 𝟓𝒙 + 𝟐𝒚 = 𝟑, 𝟓(−𝟏) + 𝟐(𝟒) = −𝟓 + 𝟖 = 𝟑 = 𝑹𝑯𝑺

Ex: Solve the system of equations using matrix method

x – y + 2z = 7, 3x + 4y – 5z = – 5, 2x – y + 3z = 12
SOL: Given Equations: x – y + 2z = 7 ; 3x + 4y – 5z = – 5 ; 2x – y + 3z = 12

The above system of equations can be written in matrix form

𝟏 −𝟏 𝟐 𝒙 𝟕
𝑨𝒅𝒋 𝑨
A = [𝟑 𝟒 −𝟓] X = [𝒚] B = [−𝟓] AX = B X = 𝑨−𝟏 𝑩 𝑨−𝟏 = |𝑨|
𝟐 −𝟏 𝟑 𝒛 𝟏𝟐
|𝑨| = 𝟏(𝟏𝟐 − 𝟓) + 𝟏(𝟗 + 𝟏𝟎) + 𝟐(−𝟑 − 𝟖) = 𝟕 + 𝟏𝟗 − 𝟐𝟐 = 𝟒 |𝑨| ≠ 𝟎 𝑨−𝟏 exists

𝑨𝟏𝟏 = (−𝟏)𝟏+𝟏 (𝟏𝟐 − 𝟓) = 𝟕 𝑨𝟐𝟏 = (−𝟏)𝟐+𝟏 (−𝟑 + 𝟐) = 𝟏

𝑨𝟏𝟐 = (−𝟏)𝟏+𝟐 (𝟗 + 𝟏𝟎) = −𝟏𝟗 𝑨𝟐𝟐 = (−𝟏)𝟐+𝟐 (𝟑 − 𝟒) = −𝟏

𝑨𝟏𝟑 = (−𝟏)𝟏+𝟑 (−𝟑 − 𝟖) = −𝟏𝟏 𝑨𝟐𝟑 = (−𝟏)𝟐+𝟑 (−𝟏 + 𝟐) = −𝟏

𝑨𝟑𝟏 = (−𝟏)𝟑+𝟏 (𝟓 − 𝟖) = −𝟑

𝑨𝟑𝟐 = (−𝟏)𝟑+𝟐 (−𝟓 − 𝟔) = 𝟏𝟏

𝑨𝟑𝟑 = (−𝟏)𝟑+𝟑 (𝟒 + 𝟑) = 𝟕

𝟕 𝟏 −𝟑 𝟕 𝟏 −𝟑
𝑨𝒅𝒋 𝑨 𝟏
AdjA = [−𝟏𝟗 −𝟏 𝟏𝟏 ] 𝑨−𝟏 = |𝑨|
= 𝟒 [−𝟏𝟗 −𝟏 𝟏𝟏 ]
−𝟏𝟏 −𝟏 𝟕 −𝟏𝟏 −𝟏 𝟕
19

𝟕 𝟏 −𝟑 𝟕 𝟒𝟗 − 𝟓 − 𝟑𝟔 𝟖 𝟐 𝒙
−𝟏 𝟏 𝟏 𝟏
AX = B X = 𝑨 𝑩 = 𝟒 [−𝟏𝟗 −𝟏 𝟏𝟏 ] [−𝟓] = [−𝟏𝟑𝟑 + 𝟓 + 𝟏𝟑𝟐] = 𝟒 [ 𝟒 ] = [𝟏] = [𝒚]
𝟒
−𝟏𝟏 −𝟏 𝟕 𝟏𝟐 −𝟕𝟕 + 𝟓 + 𝟖𝟒 𝟏𝟐 𝟑 𝒛

The solution is 𝒙 = 𝟐, 𝒚 = 𝟏 , 𝒛 = 𝟑

CHECK: Sub in Equations (1), x – y + 2z = 7; 𝟐 − 𝟏 + 𝟐(𝟑) = 𝟕 = 𝑹𝑯𝑺

𝟐 −𝟑 𝟓
Ex: If A = [𝟑 𝟐 −𝟒] , find 𝑨−𝟏 and hence Solve the system of equations.
𝟏 𝟏 −𝟐
2x – 3y + 5z = 11, 3x + 2y – 4z = – 5, x + y – 2z = – 3
𝟐 −𝟑 𝟓
𝑨𝒅𝒋 𝑨
SOL: A = [𝟑 𝟐 −𝟒] 𝑨−𝟏 = |𝑨|
𝟏 𝟏 −𝟐
|𝑨| = 𝟐(−𝟒 + 𝟒) + 𝟑(−𝟔 + 𝟒) + 𝟓(𝟑 − 𝟐) = 𝟎 − 𝟔 + 𝟓 = −𝟏 |𝑨| ≠ 𝟎 𝑨−𝟏 exists

𝑨𝟏𝟏 = (−𝟏)𝟏+𝟏 (−𝟒 + 𝟒) = 𝟎 𝑨𝟐𝟏 = (−𝟏)𝟐+𝟏 (𝟔 − 𝟓) = −𝟏

𝑨𝟏𝟐 = (−𝟏)𝟏+𝟐 (−𝟔 + 𝟒) = 𝟐 𝑨𝟐𝟐 = (−𝟏)𝟐+𝟐 (−𝟒 − 𝟓) = −𝟗

𝑨𝟏𝟑 = (−𝟏)𝟏+𝟑 (𝟑 − 𝟐) = 𝟏 𝑨𝟐𝟑 = (−𝟏)𝟐+𝟑 (𝟐 + 𝟑) = −𝟓

𝑨𝟑𝟏 = (−𝟏)𝟑+𝟏 (𝟏𝟐 − 𝟏𝟎) = 𝟐

𝑨𝟑𝟐 = (−𝟏)𝟑+𝟐 (−𝟖 − 𝟏𝟓) = 𝟐𝟑

𝑨𝟑𝟑 = (−𝟏)𝟑+𝟑 (𝟒 + 𝟗) = 𝟏𝟑

𝟎 −𝟏 𝟐 𝟎 −𝟏 𝟐 𝟎 𝟏 −𝟐
𝑨𝒅𝒋 𝑨
AdjA = [𝟐 −𝟗 𝟐𝟑] 𝑨−𝟏 = |𝑨|
= (−𝟏) [𝟐 −𝟗 𝟐𝟑 ] = [− 𝟐 𝟗 −𝟐𝟑]
𝟏 −𝟓 𝟏𝟑 𝟏 −𝟓 𝟏𝟑 −𝟏 𝟓 −𝟏𝟑

Given Equations: 2x – 3y + 5z = 11, 3x + 2y – 4z = – 5, x + y – 2z = – 3


The above system of equations can be written in matrix form

𝟐 −𝟑 𝟓 𝒙 𝟏𝟏
𝑨𝒅𝒋 𝑨
A = [𝟑 𝟐 −𝟒] X = [𝒚] B = [−𝟓] AX = B X = 𝑨−𝟏 𝑩 𝑨−𝟏 = |𝑨|
𝟏 𝟏 −𝟐 𝒛 −𝟑
𝟎 𝟏 −𝟐 𝟏𝟏 𝟎+𝟓+𝟔 𝟏 𝒙
−𝟏
X= 𝑨 𝑩 = [−𝟐 𝟗 −𝟐𝟑] [−𝟓] = [−𝟐𝟐 − 𝟒𝟓 + 𝟔𝟗] = [𝟐] = [𝒚]
−𝟏 𝟓 −𝟏𝟑 −𝟑 −𝟏𝟏 − 𝟐𝟓 + 𝟑𝟗 𝟑 𝒛

The solution is 𝒙 = 𝟏, 𝒚 = 𝟐 , 𝒛 = 𝟑
CHECK: Sub in Equations (3), x + y –2 z = 0; 𝟏 + 𝟐 − 𝟐(𝟑) = −𝟑 = 𝑹𝑯𝑺

Ex: Solve the system of equations.


20

𝟐 𝟑 𝟏𝟎 𝟒 𝟔 𝟓 𝟔 𝟗 𝟐𝟎
+ + = 𝟒, − + =𝟏, + − =𝟐
𝒙 𝒚 𝒛 𝒙 𝒚 𝒛 𝒙 𝒚 𝒛

𝟐 𝟑 𝟏𝟎 𝟒 𝟔 𝟓 𝟔 𝟗 𝟐𝟎
SOL: Given + + = 𝟒, − + =𝟏, + − =𝟐
𝒙 𝒚 𝒛 𝒙 𝒚 𝒛 𝒙 𝒚 𝒛

𝟏 𝟏 𝟏
Let = 𝒑, =𝒒, =𝒓 then Equations are
𝒙 𝒚 𝒛

2p + 3q + 10r = 4, 4p – 6q + 5r = 1, 6p + 9q – 20r = 2
The above system of equations can be written in matrix form

𝟐 𝟑 𝟏𝟎 𝒑 𝟒
𝑨𝒅𝒋 𝑨
A = [𝟒 −𝟔 𝟓 ] X = [𝒒] B = [𝟏] AX = B X = 𝑨−𝟏 𝑩 𝑨−𝟏 = |𝑨|
𝟔 𝟗 −𝟐𝟎 𝒓 𝟐
|𝑨| = 𝟐(𝟏𝟐𝟎 − 𝟒𝟓) − 𝟑(−𝟖𝟎 − 𝟑𝟎) + 𝟏𝟎(𝟑𝟔 + 𝟑𝟔) = 𝟏𝟓𝟎 + 𝟑𝟑𝟎 + 𝟕𝟐𝟎 = 𝟏𝟐𝟎𝟎

|𝑨| ≠ 𝟎 𝑨−𝟏 exists


𝟕𝟓 𝟏𝟓𝟎 𝟕𝟓 𝟕𝟓 𝟏𝟓𝟎 𝟕𝟓
𝑨𝒅𝒋 𝑨 𝟏
AdjA = [𝟏𝟏𝟎 −𝟏𝟎𝟎 𝟑𝟎 ] 𝑨−𝟏 = |𝑨|
= 𝟏𝟐𝟎𝟎 [𝟏𝟏𝟎 −𝟏𝟎𝟎 𝟑𝟎 ]
𝟕𝟐 𝟎 −𝟐𝟒 𝟕𝟐 𝟎 −𝟐𝟒
𝟏

𝒑 𝟔𝟎𝟎 𝟐
𝟕𝟓 𝟏𝟓𝟎 𝟕𝟓 𝟒 𝟏 𝟏
−𝟏 𝟏
AX = B X = 𝑨 𝑩 = 𝟏𝟐𝟎𝟎 [𝟏𝟏𝟎 −𝟏𝟎𝟎 𝟑𝟎 ] [𝟏] [𝒒]= [ 𝟒𝟎𝟎 ] = 𝟑
𝟏𝟐𝟎𝟎
𝟕𝟐 𝟎 −𝟐𝟒 𝟐 𝒓 𝟐𝟒𝟎 𝟏
[𝟓]
𝟏 𝟏 𝟏 𝟏 𝟏 𝟏
The solution is 𝒑= = ; 𝒒= = ; 𝒓= = 𝒙 = 𝟐, 𝒚 = 𝟑, 𝒛 = 𝟓
𝟐 𝒙 𝟑 𝒚 𝟓 𝒛
𝟐 𝟑 𝟏𝟎 𝟐 𝟑 𝟏𝟎
CHECK: Sub in Eqn (1), + + = 𝟒; + + = 𝟏 + 𝟏 + 𝟐 = 𝟒 = 𝑹𝑯𝑺
𝒙 𝒚 𝒛 𝟐 𝟑 𝟓
𝟏 −𝟏 𝟎 𝟐 𝟐 −𝟒
Ex: If A = [𝟐 𝟑 𝟒] and B = [−𝟒 𝟐 −𝟒] find the product of AB and hence solve the
𝟎 𝟏 𝟐 𝟐 −𝟏 𝟓
equations: x – y = 3, 2x + 3y + 4z = 17, y + 2z = 7

𝟏 −𝟏 𝟎 𝟐 𝟐 −𝟒
SOL: Given: A = [𝟐 𝟑 𝟒] and B = [−𝟒 𝟐 −𝟒]
𝟎 𝟏 𝟐 𝟐 −𝟏 𝟓
𝟏 −𝟏 𝟎 𝟐 𝟐 −𝟒 𝟔 𝟎 𝟎
Product of AB = [𝟐 𝟑 𝟒] [−𝟒 𝟐 −𝟒] = [𝟎 𝟔 𝟎] = 𝟔 𝑰 𝑨𝑩 = 𝟔𝑰
𝟎 𝟏 𝟐 𝟐 −𝟏 𝟓 𝟎 𝟎 𝟔
𝑩 𝑩
So, 𝑨 ( 𝟔 ) = 𝑰 The inverse of A = 𝟔

Given equations: x – y = 3, 2x + 3y + 4z = 17, y + 2z = 7


21

The above system of equations can be written in matrix form

𝟏 −𝟏 𝟎 𝒙 𝟑
A = [𝟐 𝟑 𝟒] X = [𝒚] C = [𝟏𝟕] AX = C
𝟎 𝟏 𝟐 𝒛 𝟕
𝟐 𝟐 −𝟒 𝟑 𝟔 + 𝟑𝟒 − 𝟐𝟖 𝟏𝟐 𝟐 𝒙
𝟏 𝟏 𝟏
X = 𝑨−𝟏 𝑪 = 𝟔 [−𝟒 𝟐 −𝟒] [𝟏𝟕] = [ −𝟏𝟐 + 𝟑𝟒 − 𝟐𝟖 ] = [ −𝟔 ] = [ −𝟏 ] = [ 𝒚]
𝟔 𝟔
𝟐 −𝟏 𝟓 𝟕 𝟔 − 𝟏𝟕 + 𝟑𝟓 𝟐𝟒 𝟒 𝒛

The solution is 𝒙 = 𝟐, 𝒚 = −𝟏 , 𝒛 = 𝟒

CHECK: Sub in Equations (1), x – y = 3; 𝟐 + 𝟏 = 𝟑 = 𝑹𝑯𝑺

𝟏 −𝟏 𝟏
Ex: If A = [𝟐 𝟏 −𝟑] , find 𝑨−𝟏 and hence Solve the system of equations.
𝟏 𝟏 𝟏
x + 2y + z = 4, –x + y + z = 0, x – 3y + z = 2
𝟏 −𝟏 𝟏
𝑨𝒅𝒋 𝑨
SOL: A = [𝟐 𝟏 −𝟑] 𝑨−𝟏 = |𝑨|
𝟏 𝟏 𝟏
|𝑨| = 𝟏(𝟏 + 𝟑) + 𝟏(𝟐 + 𝟑) + 𝟏(𝟐 − 𝟏) = 𝟒 + 𝟓 + 𝟏 = 𝟏𝟎 |𝑨| ≠ 𝟎 𝑨−𝟏 exists

𝟒 𝟐 𝟐 𝟒 𝟐 𝟐
−𝟏 𝑨𝒅𝒋 𝑨 𝟏
AdjA = [−𝟓 𝟎 𝟓] 𝑨 = |𝑨|
= 𝟏𝟎 [−𝟓 𝟎 𝟓]
𝟏 −𝟐 𝟑 𝟏 −𝟐 𝟑

Given Equations: x + 2y + z = 4, – x + y + z = 0, x – 3y + z = 2
The above system of equations can be written in matrix form

𝟏 𝟐 𝟏 𝒙 𝟒
𝑨𝒅𝒋 𝑨
A = [−𝟏 𝟏 𝟏] X = [𝒚] B = [𝟎] AX = B X = 𝑨−𝟏 𝑩 𝑨−𝟏 = |𝑨|
𝟏 −𝟑 𝟏 𝒛 𝟐
𝟗
𝟒 −𝟓 𝟏 𝟒 𝟏𝟔 − 𝟎 + 𝟐 𝟏𝟖 𝟓 𝒙
𝟏 𝟏 𝟏 𝟐
X= (𝑨−𝟏 )𝑻 𝑩 = [𝟐 𝟎 −𝟐] [𝟎] = [ 𝟖 + 𝟎 − 𝟒 ] = 𝟏𝟎 [ 𝟒 ] = = [𝒚]
𝟏𝟎 𝟏𝟎 𝟓
𝟐 𝟓 𝟑 𝟐 𝟖+𝟎+𝟔 𝟏𝟒 𝟕 𝒛
[𝟓]
𝟗 𝟐 𝟕
The solution is 𝒙 = 𝟓, 𝒚 = ,𝒛 =
𝟓 𝟓
−𝟗 𝟐 𝟕
CHECK: Sub in Equations (2), – x + y + z = 0; + 𝟓 + 𝟓 = 𝟎 = 𝑹𝑯𝑺
𝟓
22

5 – CONTINUITY AND DERIVATIVE


𝑲𝒆𝒚 𝒔𝒕𝒆𝒑𝒔 𝒕𝒐 𝑹𝒆𝒎𝒆𝒎𝒃𝒆𝒓
𝐴 𝑟𝑒𝑎𝑙 𝑣𝑎𝑙𝑢𝑒 𝑓𝑢𝑛𝑐𝑡𝑖𝑜𝑛 𝑓 𝑖𝑠 𝑐𝑜𝑛𝑡𝑖𝑛𝑢𝑜𝑢𝑠𝑎𝑡 𝑥 = 𝑎, 𝑖𝑓 lim 𝑓(𝑥) = 𝑓(𝑎)
𝑥→𝑎
𝑇ℎ𝑎𝑡 𝑖𝑠 𝑎 𝑓𝑢𝑛𝑐𝑡𝑖𝑜𝑛 𝑓 𝑖𝑠 𝑐𝑜𝑛𝑡𝑖𝑛𝑢𝑜𝑢𝑠𝑎𝑡 𝑥 = 𝑎,
𝑖𝑓 lim 𝑓(𝑥) = lim 𝑓(𝑥) = 𝑓(𝑎)
𝑥→𝑎− 𝑥→𝑎+
𝐿𝐻𝐿 = 𝑅𝐻𝐿 = 𝑓(𝑎)
𝐹𝑜𝑟𝑚𝑢𝑙𝑎𝑒: 𝑆𝑡𝑎𝑛𝑑𝑎𝑟𝑑 𝐿𝑖𝑚𝑖𝑡𝑠
𝑥 𝑛 − 𝑎𝑛 𝑠𝑖𝑛𝑥
1). lim = 𝑛 𝑎𝑛−1 2). lim = 1, 𝑤ℎ𝑒𝑟𝑒 𝑥 𝑖𝑠 𝑖𝑛 𝑟𝑎𝑖𝑑𝑎𝑛
𝑥→𝑎 𝑥−𝑎 𝑥→0 𝑥
𝑎𝑥 −𝑥 𝑒 𝑥 −1
3). lim = 𝑙𝑜𝑔𝑎 4). lim = 𝑙𝑜𝑔𝑎 = 1
𝑥→𝑎 𝑥 𝑥→𝑎 𝑥
𝑮𝒆𝒏𝒆𝒓𝒂𝒍 𝑴𝒊𝒔𝒕𝒂𝒌𝒆𝒔 𝒄𝒐𝒎𝒎𝒊𝒕𝒆𝒅 𝒃𝒚 𝑺𝒕𝒖𝒅𝒆𝒏𝒕𝒔
No Mistakes Remarks
1 When to Consider LHL and Generally when ≤ ≥ > < Conditions
RHL are given, Modulus, Steps Function
questions
2 Basic Standard Limits Formula test and more practice
3 Phobia for steps and attempting Discussion of various Model Questions
4 Conclusion not proper Practice in class tests
5 Finding LHL and RHL without Explain LHL and RHL
Limits

3 Marks and 5 Marks Questions


𝑥 4 −16
, 𝑥≠2
1). 𝐿𝑜𝑐𝑎𝑡𝑒 𝑡ℎ𝑒 𝑝𝑜𝑖𝑛𝑡𝑠 𝑜𝑓 𝑑𝑖𝑠𝑐𝑜𝑛𝑡𝑖𝑛𝑢𝑖𝑡𝑦 𝑜𝑓 𝑓(𝑥) = { 𝑥−2
16, 𝑥 = 2,
𝑥 4 − 16
𝐺𝑖𝑣𝑒𝑛: 𝑓(𝑥) = { 𝑥 − 2 , 𝑥 ≠ 2
16, 𝑥 = 2,
𝑓(𝑎) = 𝑓(2) = 16
𝑥 4 − 16 𝑥 4 − 24
𝐿𝑖𝑚 𝑓(𝑥) = 𝐿𝑖𝑚 = 𝐿𝑖𝑚 = 4. (2)4−1 = 32
𝑥→2 𝑥→2 𝑥−2 𝑥→2 𝑥−2
𝑓(𝑥) 𝑖𝑠 𝐶𝑜𝑛𝑡𝑖𝑛𝑢𝑜𝑢𝑠 𝑖𝑓 , 𝐿𝑖𝑚 𝑓(𝑥) = 𝑓(𝑎), 32 ≠ 𝑓(𝑎)
𝑥→𝑎
𝑓 𝑖𝑠 𝑑𝑖𝑠𝑐𝑜𝑛𝑡𝑖𝑛𝑢𝑜𝑢𝑠 𝑎𝑡 𝑥 = 2

2𝑥 + 3, 𝑥 ≤ 2
2. 𝑉𝑒𝑟𝑖𝑓𝑦 𝑡ℎ𝑒 𝐶𝑜𝑛𝑡𝑖𝑛𝑢𝑖𝑡𝑦 𝑜𝑓 𝑓(𝑥) = { 𝑎𝑡 𝑥 = 2.
2𝑥 − 3, 𝑥 > 2,
2𝑥 + 3, 𝑥 ≤ 2
𝐺𝑖𝑣𝑒𝑛: 𝑓(𝑥) = { 𝑎𝑡 𝑥 = 2
2𝑥 − 3, 𝑥 > 2,
23

𝑓(𝑎) = 𝑓(2) = 4 + 3 = 7
𝐿𝐻𝐿 = 𝐿𝑖𝑚 𝑓(𝑥) = 𝐿𝑖𝑚 (2𝑥 + 3) = 7
𝑥→2− 𝑥→2
𝑅𝐻𝐿 = 𝐿𝑖𝑚 𝑓(𝑥)
𝑥→2+
= 𝐿𝑖𝑚 (2𝑥 − 3) = 1
𝑥→2
𝐿𝐻𝐿 𝑅𝐻𝐿, 𝐿𝑖𝑚𝑖𝑡 𝑑𝑜𝑒𝑠 𝑛𝑜𝑡 𝑒𝑥𝑖𝑠𝑡𝑠
𝑓(𝑥)𝑖𝑠 𝑛𝑜𝑡 𝐶𝑜𝑛𝑡𝑖𝑛𝑢𝑜𝑢𝑠 𝑎𝑡 𝑥 = 2

𝑠𝑖𝑛𝑥
3. 𝑉𝑒𝑟𝑖𝑓𝑦 𝑡ℎ𝑒 𝐶𝑜𝑛𝑡𝑖𝑛𝑢𝑖𝑡𝑦 𝑜𝑓 𝑓(𝑥) = { 𝑥 , 𝑥 ≠ 0
𝑥 + 1, 𝑥 = 0
𝑠𝑖𝑛𝑥
𝐺𝑖𝑣𝑒𝑛: 𝑓(𝑥) = { 𝑥 , 𝑥 ≠ 0
𝑥 + 1, 𝑥 = 0
𝑓(𝑎) = 𝑓(0) = 0 + 1 = 1
𝑠𝑖𝑛𝑥
𝐿𝑖𝑚 𝑓(𝑥) = 𝐿𝑖𝑚 =1
𝑥→0 𝑥→0 𝑥
𝐿𝑖𝑚 𝑓(𝑥) = 𝑓(𝑎), 𝑓(𝑥) 𝑖𝑠 𝐶𝑜𝑛𝑡𝑖𝑛𝑢𝑜𝑢𝑠 𝑎𝑡 𝑥 = 0.
𝑥→𝑎

|𝑥|
4. 𝑉𝑒𝑟𝑖𝑓𝑦 𝑡ℎ𝑒 𝐶𝑜𝑛𝑡𝑖𝑛𝑢𝑖𝑡𝑦 𝑜𝑓 𝑓(𝑥) = { 𝑥 , 𝑥 ≠ 0
0, 𝑥 = 0
|𝑥|
𝐺𝑖𝑣𝑒𝑛: 𝑓(𝑥) = { 𝑥 , 𝑥 ≠ 0
0, 𝑥 = 0
𝑓(𝑎) = 𝑓(0) = 0
|𝑥| |0 − ℎ| |ℎ|
𝐿𝐻𝐿 = 𝐿𝑖𝑚 𝑓(𝑥) = 𝐿𝑖𝑚 = 𝐿𝑖𝑚 = 𝐿𝑖𝑚 = −1
𝑥→0 𝑥→0− 𝑥 ℎ→0 0 − ℎ 𝑥→0 −ℎ
𝑅𝐻𝐿 = 𝐿𝑖𝑚 𝑓(𝑥)
𝑥→0+
|𝑥| |0 + ℎ| |ℎ|
= 𝐿𝑖𝑚 = 𝐿𝑖𝑚 = 𝐿𝑖𝑚 =1
𝑥→0+ 𝑥 ℎ→0 0+ℎ 𝑥→0 ℎ
𝐿𝐻𝐿 𝑅𝐻𝐿
𝐿𝑖𝑚 𝑓(𝑥) 𝑛𝑜𝑡𝑒𝑥𝑖𝑠𝑡𝑠 𝑓(𝑥)𝑖𝑠 𝑛𝑜𝑡 𝐶𝑜𝑛𝑡𝑖𝑛𝑢𝑜𝑢𝑠
𝑥→𝑎
𝑥 + 1, 𝑥 ≥ 1
5. 𝐹𝑖𝑛𝑑 𝑡ℎ𝑒 𝑡ℎ𝑒 𝑝𝑜𝑖𝑛𝑡𝑠 𝑜𝑓 𝑑𝑖𝑠𝑐𝑜𝑛𝑡𝑖𝑛𝑢𝑖𝑡𝑦 𝑜𝑓 𝑓(𝑥), 𝑖𝑓 𝑓(𝑥) = {
𝑥 2 + 1, 𝑥 < 1.
𝑥 + 1, 𝑥 ≥ 1
𝐺𝑖𝑣𝑒𝑛: 𝑓(𝑥) = { 2
𝑥 + 1, 𝑥 < 1.
𝑓(𝑎) = 𝑓(1) = 1 + 1 = 2
𝐿𝐻𝐿 = 𝐿𝑖𝑚 𝑓(𝑥) = 𝐿𝑖𝑚 (𝑥 + 1) = 2
𝑥→1− 𝑥→1
𝑅𝐻𝐿 = 𝐿𝑖𝑚 𝑓(𝑥) = 𝐿𝑖𝑚 ( 𝑥 2 + 1) = 2
𝑥→1+ 𝑥→1
𝑓 𝑖𝑠 𝑐𝑜𝑛𝑡𝑖𝑛𝑢𝑜𝑢𝑠 𝑖𝑓 𝐿𝑖𝑚 𝑓(𝑥) = 𝑓(𝑎)
𝑥→𝑎
𝐿𝐻𝐿 = 𝑅𝐻𝐿 = 𝑓(𝑎) = 2,  𝑓 𝑖𝑠 𝑐𝑜𝑛𝑡𝑖𝑛𝑜𝑢𝑠 𝑎𝑡 𝑥 = 1
24

√1 + 𝑥 − √1 − 𝑥
6. 𝐷𝑖𝑠𝑐𝑢𝑠𝑠 𝑡ℎ𝑒 𝑐𝑜𝑛𝑡𝑖𝑛𝑢𝑖𝑡𝑦 𝑜𝑓 𝑓 𝑎𝑡 𝑥 = 0 𝑖𝑓, 𝑓(𝑥) = { , 𝑖𝑓 𝑥 ≠ 0
𝑠𝑖𝑛𝑥
1, 𝑖𝑓 𝑥 = 0.
√1 + 𝑥 − √1 − 𝑥
𝐺𝑖𝑣𝑒𝑛: 𝑓(𝑥) = { , 𝑖𝑓 𝑥 ≠ 0
𝑠𝑖𝑛𝑥
1, 𝑖𝑓 𝑥 = 0.

𝑓(𝑎) = 𝑓(0) = 1
√1+𝑥− √1− 𝑥 √1+𝑥− √1− 𝑥 ( √1+𝑥+ √1− 𝑥 )
𝐿𝑖𝑚 𝑓(𝑥) = 𝐿𝑖𝑚 = 𝐿𝑖𝑚 =
𝑥→0 𝑥→0 𝑠𝑖𝑛𝑥 𝑥→0 𝑠𝑖𝑛𝑥 √1+𝑥+ √1− 𝑥
1+𝑥−1+𝑥
= 𝐿𝑖𝑚
𝑥→0 𝑠𝑖𝑛𝑥 ( √1+𝑘𝑥+ √1− 𝑘𝑥 )
2𝑥 𝑥 2 2
= 𝐿𝑖𝑚 = 𝐿𝑖𝑚 𝐿𝑖𝑚 = 1. = 1
𝑥→0 𝑠𝑖𝑛𝑥 ( √1+𝑥+ √1− 𝑥 ) 𝑥→0 𝑠𝑖𝑛𝑥 𝑥→0 ( √1+𝑥+ √1− 𝑥 ) 2

𝑓 𝑖𝑠 𝑐𝑜𝑛𝑡𝑖𝑛𝑢𝑜𝑢𝑠 𝑖𝑓 𝐿𝑖𝑚 𝑓(𝑥) = 𝑓(𝑎) 1 = 𝑓(𝑎) = 1,


𝑥→𝑎

 𝑓(𝑥)𝑖𝑠 𝑐𝑜𝑛𝑡𝑖𝑛𝑢𝑜𝑢𝑠 𝑎𝑡 𝑥 = 0
7. 𝐹𝑖𝑛𝑑 𝑡ℎ𝑒 𝑣𝑎𝑙𝑢𝑒 𝑜𝑓 𝑘 𝑓𝑜𝑟 𝑤ℎ𝑖𝑐ℎ 𝑓 𝑖𝑠 𝑐𝑜𝑛𝑡𝑖𝑛𝑢𝑜𝑢𝑠 𝑤ℎ𝑒𝑛 𝑓 𝑑𝑒𝑓𝑖𝑛𝑒𝑑 𝑏𝑦
√1 + 𝑘𝑥 − √1 − 𝑘𝑥
, 𝑖𝑓 − 1 ≤ 𝑥 < 0
𝑓(𝑥) = 𝑥
2𝑥 + 1
{ , 𝑖𝑓 0 ≤ 𝑥 < 1.
𝑥−1
√1 + 𝑘𝑥 − √1 − 𝑘𝑥
, 𝑖𝑓 − 1 ≤ 𝑥 < 0
𝐺𝑖𝑣𝑒𝑛: 𝑓(𝑥) = 𝑥
2𝑥 + 1
{ , 𝑖𝑓 0 ≤ 𝑥 < 1.
𝑥−1
𝑓(𝑎) = 𝑓(0) = −1
√1+𝑘𝑥− √1− 𝑘𝑥
𝐿𝐻𝐿 = 𝐿𝑖𝑚 𝑓(𝑥) = 𝐿𝑖𝑚
𝑥→0− 𝑥→0 𝑥

√1+𝑘𝑥− √1− 𝑘𝑥 ( √1+𝑘𝑥+ √1− 𝑘𝑥 )


= 𝐿𝑖𝑚 =
𝑥→0 𝑥 √1+𝑘𝑥+ √1− 𝑘𝑥
1+𝑘𝑥−1+𝑘𝑥
= 𝐿𝑖𝑚
𝑥→0 𝑥 ( √1+𝑘𝑥+ √1− 𝑘𝑥 )
2𝑘𝑥 2𝑘 2𝑘
= 𝐿𝑖𝑚 = 𝐿𝑖𝑚 = = 𝑘
𝑥→0 𝑥 ( √1+𝑘𝑥+ √1− 𝑘𝑥 ) 𝑥→0 ( √1+𝑘𝑥+ √1− 𝑘𝑥 ) 2
2𝑥+1
𝑅𝐻𝐿 = 𝐿𝑖𝑚 𝑓(𝑥) = 𝐿𝑖𝑚 = −1
𝑥→0+ 𝑥→0 𝑥−1

𝑓 𝑖𝑠 𝑐𝑜𝑛𝑡𝑖𝑛𝑢𝑜𝑢𝑠 𝑖𝑓 𝐿𝑖𝑚 𝑓(𝑥) = 𝑓(𝑎)


𝑥→𝑎

𝐿𝐻𝐿 = 𝑅𝐻𝐿 = 𝑓(𝑎),  𝑘 = −1


25

𝑘𝑐𝑜𝑠𝑥 𝜋
, 𝑥≠
8. 𝐹𝑜𝑟 𝑤ℎ𝑎𝑡 𝑣𝑎𝑙𝑢𝑒𝑠 𝑜𝑓 𝑘 𝑡ℎ𝑒 𝑓𝑢𝑛𝑐𝑡𝑖𝑜𝑛 𝑓 𝑖𝑠 𝑐𝑜𝑛𝑡𝑖𝑛𝑢𝑜𝑢𝑠𝑓(𝑥) = { 𝜋−2𝑥 𝜋
2
3, 𝑥 =
2
𝑘𝑐𝑜𝑠𝑥 𝜋
, 𝑥≠ 𝜋
𝐺𝑖𝑣𝑒𝑛: 𝑓(𝑥) = {𝜋 − 2𝑥 2
𝜋 𝑓(𝑎) = 𝑓 ( ) = 3
2
3, 𝑥 =
2
𝑘𝑐𝑜𝑠𝑥
𝐿𝑖𝑚𝜋
𝑓(𝑥) = 𝐿𝑖𝑚𝜋 𝜋 − 2𝑥
𝑥→ 𝑥→
2 2
𝜋 𝑡 𝜋
𝜋 − 2𝑥 = 𝑡, 𝑥 = − , 𝐴𝑠 𝑥 → , 𝑡→0
2 2 2
𝜋 𝑡 𝑡
𝑘𝑐𝑜𝑠 ( − ) 𝑠𝑖𝑛 1 𝑘
= 𝐿𝑖𝑚 2 2 =𝑘 𝐿𝑖𝑚 𝑡 2 =
𝑡→0 𝑡 𝑡
→0 2 2
2 2
𝑘
𝑓(𝑥)𝑖𝑠 𝐶𝑜𝑛𝑡𝑖𝑛𝑢𝑜𝑢𝑠 𝑖𝑓 𝐿𝑖𝑚 𝑓(𝑥) = 𝑓(𝑎), =3 → 𝑘=6
𝑥→𝑎 2

5, 𝑖𝑓 𝑥 ≤ 2
9. 𝐹𝑖𝑛𝑑 𝑡ℎ𝑒 𝑣𝑎𝑙𝑢𝑒𝑠 𝑎 𝑎𝑛𝑑 𝑏 𝑠𝑜 𝑡ℎ𝑎𝑡 𝑓 𝑖𝑠 𝑐𝑜𝑛𝑡𝑖𝑛𝑢𝑜𝑢𝑠, 𝑖𝑓 𝑓(𝑥) = {𝑎𝑥 + 𝑏, 𝑖𝑓 2 < 𝑥 < 10
21, 𝑖𝑓 𝑥 ≥ 10

5, 𝑖𝑓 𝑥 ≤ 2
𝐺𝑖𝑣𝑒𝑛: 𝑓(𝑥) = {𝑎𝑥 + 𝑏, 𝑖𝑓 2 < 𝑥 < 10
21, 𝑖𝑓 𝑥 ≥ 10
𝐹𝑜𝑟 𝑐𝑜𝑛𝑡𝑖𝑛𝑢𝑖𝑡𝑦 𝑎𝑡 𝑥 = 2: 𝑓(𝑎) = 𝑓(2) = 5
𝐿𝐻𝐿 = 𝐿𝑖𝑚 𝑓(𝑥) = 𝐿𝑖𝑚 5 = 5
𝑥→2− 𝑥→2

𝑅𝐻𝐿 = 𝐿𝑖𝑚 𝑓(𝑥) = 𝐿𝑖𝑚 (𝑎𝑥 + 𝑏) = 2𝑎 + 𝑏


𝑥→2+ 𝑥→2

𝑓 𝑖𝑠 𝑐𝑜𝑛𝑡𝑖𝑛𝑢𝑜𝑢𝑠 𝑖𝑓 𝐿𝑖𝑚 𝑓(𝑥) = 𝑓(𝑎)


𝑥→𝑎

𝐿𝐻𝐿 = 𝑅𝐻𝐿 = 𝑓(𝑎), 5 = 2𝑎 + 𝑏 = 5


2𝑎 + 𝑏 = 5 … … … … … . (1)
𝐹𝑜𝑟 𝑐𝑜𝑛𝑡𝑖𝑛𝑢𝑖𝑡𝑦 𝑎𝑡 𝑥 = 10:
𝑓(𝑎) = 𝑓(10) = 21
𝐿𝐻𝐿 = 𝐿𝑖𝑚 𝑓(𝑥) = 𝐿𝑖𝑚 (𝑎𝑥 + 𝑏) = 10𝑎 + 𝑏
𝑥→10− 𝑥→10

𝑅𝐻𝐿 = 𝐿𝑖𝑚 𝑓(𝑥)


𝑥→10+

= 𝐿𝑖𝑚 (𝑎𝑥 + 𝑏) = 21
𝑥→10
26

𝑓 𝑖𝑠 𝑐𝑜𝑛𝑡𝑖𝑛𝑢𝑜𝑢𝑠 𝑖𝑓 𝐿𝑖𝑚 𝑓(𝑥) = 𝑓(𝑎)


𝑥→𝑎

𝐿𝐻𝐿 = 𝑅𝐻𝐿 = 𝑓(𝑎), 10𝑎 + 𝑏 = 21 = 21


10𝑎 + 𝑏 = 21 … … … … … . (2)
2𝑎 + 𝑏 = 5 … … … … … . (1)
𝑆𝑜𝑙𝑣𝑖𝑛𝑔 (1)𝑎𝑛𝑑 (2)𝑤𝑒 𝑔𝑒𝑡 𝑎 = 2, 𝑏 = 1
√𝑥
, 𝑥≠0
10. 𝐹𝑖𝑛𝑑 𝑡ℎ𝑒 𝑣𝑎𝑙𝑢𝑒 𝑜𝑓 ′𝑎′𝑓𝑜𝑟 𝑤ℎ𝑖𝑐ℎ 𝑓 𝑖𝑠 𝑐𝑜𝑛𝑡𝑖𝑛𝑢𝑜𝑢𝑠, 𝑖𝑓 𝑓(𝑥) = {√16 + √𝑥 − 4
𝑎, 𝑥=0
√𝑥
, 𝑥≠0
𝐺𝑖𝑣𝑒𝑛: 𝑓(𝑥) = { √16+√𝑥 −4
𝑎, 𝑥=0
√𝑥 √16+√𝑥+4
= 𝐿𝑖𝑚 𝑓(𝑥) = 𝐿𝑖𝑚
𝑥→0+ 𝑥→0 √16+√𝑥 −4 √16+√𝑥+4

√𝑥(√16+√𝑥+4) √𝑥(√16+√𝑥+4)
= 𝐿𝑖𝑚 = 𝐿𝑖𝑚
𝑥→0 16+√𝑥 −16 𝑥→0 √𝑥

= 𝐿𝑖𝑚(√16 + √𝑥 + 4)
𝑥→0

=4+4= 8

𝑆𝑖𝑛𝑐𝑒 𝑓 𝑖𝑠 𝐶𝑜𝑛𝑡𝑖𝑛𝑢𝑜𝑢𝑠, 𝐿𝑖𝑚 𝑓(𝑥) = 𝑓(𝑎), 𝑎 =8


𝑥→𝑎

3𝑎𝑥 + 𝑏 , 𝑤ℎ𝑒𝑛 𝑥 > 1


11. 𝐼𝑓 𝑓 𝑖𝑠 𝑐𝑜𝑛𝑡𝑖𝑛𝑢𝑜𝑢𝑠 𝑓𝑖𝑛𝑑 𝑎, 𝑏 𝑤ℎ𝑒𝑛 𝑓(𝑥) = { 11 , 𝑤ℎ𝑒𝑛 𝑥 = 1
5𝑎𝑥 − 2𝑏 , 𝑤ℎ𝑒𝑛 𝑥 < 1
3𝑎𝑥 + 𝑏 , 𝑤ℎ𝑒𝑛 𝑥 > 1
𝐺𝑖𝑣𝑒𝑛: 𝑓(𝑥) = { 11 , 𝑤ℎ𝑒𝑛 𝑥 = 1
5𝑎𝑥 − 2𝑏 , 𝑤ℎ𝑒𝑛 𝑥 < 1
𝑓(𝑎) = 𝑓(1) = 11
𝐿𝐻𝐿 = 𝐿𝑖𝑚 𝑓(𝑥) = 𝐿𝑖𝑚 (3𝑎𝑥 + 𝑏) = 3𝑎 + 𝑏
𝑥→1− 𝑥→1
𝑅𝐻𝐿 = 𝐿𝑖𝑚 𝑓(𝑥) = 𝐿𝑖𝑚 (5𝑎𝑥 − 2𝑏) = 5𝑎 − 2𝑏
𝑥→1+ 𝑥→1
𝑓 𝑖𝑠 𝑐𝑜𝑛𝑡𝑖𝑛𝑢𝑜𝑢𝑠 𝑖𝑓 𝐿𝑖𝑚 𝑓(𝑥) = 𝑓(𝑎)
𝑥→𝑎
𝐿𝐻𝐿 = 𝑅𝐻𝐿 = 𝑓(𝑎), 3𝑎 + 𝑏 = 5𝑎 − 2𝑏 = 11
3𝑎 + 𝑏 = 11 … … . (1) 5𝑎 − 2𝑏 = 11 … … … (2)
𝑆𝑜𝑙𝑣𝑖𝑛𝑔 𝐸𝑞𝑢𝑎𝑡𝑖𝑜𝑛𝑠 𝑎 = 3, 𝑏=2
27

Topic: Differentiation
FORMULAE
𝑑 𝑑
(𝑥 𝑛 ) = n𝑥 𝑛−1 (𝑒 𝑥 ) = 𝑒 𝑥
𝑑𝑥 𝑑𝑥
𝑑 𝑑 1
(𝑎 𝑥 ) = 𝑎 𝑥 𝑙𝑜𝑔𝑎 (logx) =
𝑑𝑥 𝑑𝑥 𝑥
𝑑 𝑑 𝑑
(sinx) = cosx (cosx) = – sinx) (tanx) = 𝑠𝑒𝑐 2 𝑥
𝑑𝑥 𝑑𝑥 𝑑𝑥
𝑑 𝑑 𝑑
(Cotx) = −𝑐𝑜𝑠𝑒𝑐 2 𝑥 (secx) = secxtan x (Cosec) = −cosecxcotx
𝑑𝑥 𝑑𝑥 𝑑𝑥
𝑑 1 𝑑 1
(𝑠𝑖𝑛−1 𝑥, 𝑐𝑜𝑠 −1 𝑥) = ± (𝑡𝑎𝑛 𝑥, 𝑐𝑜𝑡 −1 𝑥) = ±
−1
𝑑𝑥 √1−𝑥 2 𝑑𝑥 1+𝑥 2
𝑑 1
(𝑠𝑒𝑐 −1 𝑥, 𝑐𝑜𝑠𝑒𝑐 −1 𝑥) = ±
𝑑𝑥 𝑥√𝑥 2 − 1
𝑑𝑢 𝑑𝑣
𝑑 𝑑𝑣 𝑑𝑢 𝑑 𝑢 𝑣 −𝑢
𝑑𝑥 𝑑𝑥
Product Rule (𝑢𝑣) = 𝑢 +𝑣 Quotient Rule ( )=
𝑑𝑥 𝑑𝑥 𝑑𝑥 𝑑𝑥 𝑣 𝑣2
𝑑𝑦 𝑑𝑦 𝑑𝑢 𝑑𝑦 𝑑𝑦 𝑑𝑢 𝑑𝑣
Chain Rule = × and = × ×
𝑑𝑥 𝑑𝑢 𝑑𝑥 𝑑𝑥 𝑑𝑢 𝑑𝑣 𝑑𝑥

Key Factor & common mistake


1) Read and identify the question properly
2) Avoid the mistakes while applying Chain Rule, Log functions
𝑑𝑦
𝑑𝑦 𝑑𝑡
3) In parametric functions = 𝑑𝑥 where t, is a parameter
𝑑𝑥
𝑑𝑡
𝑑 𝑑𝑦 𝑑2𝑦 𝑑𝑦
4) In higher derivatives diff w.r.t x i.e
𝑑𝑥
(𝑑𝑥 ) =𝑑𝑥 2 ≠ (𝑑𝑥 ) 2
5) In inverse function taking care of sign, square, square root etc
6) Generally to forget differentiable function only show continuous function
7) log(uv) = logu + logv and not log (u +v) = logu + logv

1 MARK QUESTIONS
1). Differentiate w.r.t x: sin (𝑥 2 + 10)
𝑑𝑦 𝑑
Sol: = cos(𝑥 2 + 10) (𝑥 2 + 10) = 2𝑥cos𝑥 2 + 10)
𝑑𝑥 𝑑𝑥
𝑑𝑦
2). Find 𝑠𝑖𝑛2 𝑥 + 𝑐𝑜𝑠 2 𝑦 = 1
𝑑𝑥

𝑑 𝑑
Solution: (𝑠𝑖𝑛2 𝑥 + 𝑐𝑜𝑠 2 𝑦) = (1)
𝑑𝑥 𝑑𝑥
𝑑𝑦 𝑑𝑦 𝑠𝑖𝑛2𝑥
2 sin x cos x + 2 cos y (-siny) =0 =
𝑑𝑥 𝑑𝑥 𝑠𝑖𝑛2𝑦

3
3). Find the derivative of 𝑒 𝑥
28

𝑑𝑦 3 𝑑 𝑑𝑦 3
Solution: = 𝑒𝑥 (𝑥 3 ) = 3𝑥 2 𝑒 𝑥
𝑑𝑥 𝑑𝑥 𝑑𝑥

𝑑𝑦
4). Find 𝑖𝑓 𝑥𝑦 = 𝑒 𝑥−𝑦
𝑑𝑥

𝑑𝑦 𝑑
Solution: 𝑥 + 𝑦 = 𝑒 𝑥−𝑦 (𝑥 − 𝑦)
𝑑𝑥 𝑑𝑥
𝑑𝑦 𝑑𝑦 𝑑𝑦 𝑥𝑦−𝑦 𝑦(𝑥−1)
𝑥 + 𝑦 = 𝑒 𝑥−𝑦 (1 − ) ∴ = =
𝑑𝑥 𝑑𝑥) 𝑑𝑥 𝑥+𝑥𝑦 𝑥(1+𝑦)

5). Find the second order derivative of logx


𝑑𝑦 1 𝑑 𝑑𝑦 𝑑 1 1
Solution: = ( )= ( )=−
𝑑𝑥 𝑥 𝑑𝑥 𝑑𝑥 𝑑𝑥 𝑥 𝑥2

𝑑2𝑦
6). Find 𝑜𝑓𝑥 2 +3x+2
𝑑𝑥 2
𝑑𝑦 𝑑 2𝑦
Solution: Differentiate w.r.t x = 2𝑥 + 3 ∴ =2
𝑑𝑥 𝑑𝑥 2

2 MARKS QUESTIONS
𝑑𝑦 𝑒 𝑥 −1
1). Find 𝑤ℎ𝑒𝑛 𝑦 =
𝑑𝑥 𝑒 𝑥 −1
𝑑 (𝑒𝑥 −1) 𝑑
𝑑𝑦 (𝑒 𝑥 +1) −𝑒 𝑥 −1) (𝑒 𝑥 −1)
𝑑𝑥 𝑑𝑥
Solution: Using Quotient formula =
𝑑𝑥 (𝑒 𝑥 +1)2
𝑑𝑦 2𝑒 𝑥
=( 𝑥
𝑑𝑥 𝑒 + 1)2
𝑑𝑦
2). If y = 2logx—3sinx+𝑒 𝑥 ; find
𝑑𝑥
𝑑𝑦 𝑑 𝑑 𝑑 𝑑𝑦 1
Solution: =2 (𝑙𝑜𝑔𝑥) − 3 (𝑠𝑖𝑛𝑥) + (𝑒 𝑥 ) = 2 − 3𝑐𝑜𝑠𝑥 + 𝑒 𝑥
𝑑𝑥 𝑑𝑥 𝑑𝑥 𝑑𝑥 𝑑𝑥 𝑥

𝑑𝑦 1
3). Find when y= (2𝑢3 + 1)and u= 2
𝑑𝑥
𝑥3
−5 𝑑𝑦
𝑑𝑦 𝑑𝑢 2 𝑑𝑦 −4
Solution: = 6𝑢2 , = 𝑥 3 = 𝑑𝑢
𝑑𝑥 = -- , x≠0
𝑑𝑢 𝑑𝑥 3 𝑑𝑥 𝑥3
𝑑𝑢
2
4). Differentiate w.r.t x, 𝑒 −𝑥 sin (log 𝑥)
𝑑𝑦 2 𝑑 𝑑 2
Solution: = 𝑒 −𝑥 (sin(log 𝑥)) + sin(log 𝑥) (𝑒 −𝑥 )
𝑑𝑥 𝑑𝑥 𝑑𝑥
𝑑𝑦 2 𝑐𝑜𝑥(𝑙𝑜𝑔𝑥)
= 𝑒 −𝑥 ( − 2𝑥𝑠𝑖𝑛(𝑙𝑜𝑔𝑥)
𝑑𝑥 𝑥
29

𝜋 𝑥
5). Differentiate w.r.t. x: log{tan ( + )}
4 2
𝑑𝑦 1 𝑑 𝜋 𝑥
Solution: Let the function be y = 𝜋 𝑥 (tan ( + )
𝑑𝑥 tan ( + ) 𝑑𝑥 4 2
4 2

𝑑𝑦 1 1 1 𝑑𝑦
= 𝜋 𝑥) = 𝜋 2𝑥 = = 𝑠𝑒𝑐𝑥
𝑑𝑥 𝑠𝑖𝑛{2( + )} sin ( + ) 𝑐𝑜𝑠𝑥 𝑑𝑥
4 2 2 𝑥

𝑑
6). Prove that: (𝑐𝑜𝑡 −1 (tan 𝑥) = −1)
𝑑𝑥
𝑑𝑦 1 𝑑
Solution: Diff w.r.t x − (tan x)
𝑑𝑥 1 + 𝑡𝑎𝑛2 𝑥 𝑑𝑥
𝑑𝑦 1
=− 𝑠𝑒𝑐 2 𝑥 = −1
𝑑𝑥 𝑠𝑒𝑐 2𝑥
2𝑥
7). Differentiate w. r. t x: 𝑠𝑖𝑛−1 ( ), |𝑥| ≪ 1
1 + 𝑥2
2𝑥
Solution: Let it be y = 𝑠𝑖𝑛−1 ( ), |𝑥|
1+𝑥 2

let x = tan𝜃
y = 𝑠𝑖𝑛−1 (sin2𝜃)
𝑑𝑦 2
y = 2𝜃, y =2𝑡𝑎𝑛−1 x ∴ =
𝑑𝑥 1+𝑥 2
𝑑𝑦 𝑦
8). If y = xsiny, prove that 𝑥 =
𝑑𝑥 1−𝑥 cos 𝑦
𝑑𝑦 𝑑𝑦 𝑑𝑦 𝑠𝑖𝑛𝑦
Solution: Diff w.r.t x = 𝑥 cos 𝑦 + 𝑠𝑖𝑛𝑦 =
𝑑𝑥 𝑑𝑥 𝑑𝑥 1−𝑥𝑐𝑜𝑠𝑦
𝑑𝑦 𝑥𝑠𝑖𝑛𝑦 𝑦
∴𝑥 = = -
𝑑𝑥 1−𝑥𝑐𝑜𝑠𝑦 1−𝑥𝑐𝑜𝑠𝑦

9). Differentiate w. r .t x: cos(𝑥 𝑥 )


Solution: let the function be y, u=𝑥 𝑥
y = cosu,
𝑑𝑦 𝑑𝑢
=-sinu, =𝑥 𝑥 (1+logx)
𝑑𝑢 𝑑𝑥
𝑑𝑦 𝑑𝑦 𝑑𝑢
= × = − sin𝑥 𝑥 (1+sinx)𝑥 𝑥
𝑑𝑥 𝑑𝑢 𝑑𝑥

𝑥
10). Diff w. r.t, y = 𝑥 𝑥
Solution: Taking log both sides log y = 𝑥 𝑥 logx diff w. r .t, x
30

1 𝑑𝑦 𝑥𝑥
= + logx𝑥 𝑥 (1 + log x), value of y goes up of other side
𝑦 𝑑𝑥 𝑥

3 MARKS QUESTIONS (Average and Hot Question)


𝑑 𝑥 𝑎2 𝑥
1). Prove: ( √𝑎2 − 𝑥 2 + 𝑠𝑖𝑛−1 ) = √𝑎2 − 𝑥 2
𝑑𝑥 2 2 2

Solution: Apply product rule and inverse formula:


1 1 𝑎2 1 1
x × (−2𝑥) + √𝑎2 − 𝑥 2 ) +
2 2√𝑎2 −𝑥 2 2 2 𝑎
√1−𝑥2
𝑎

−𝑥 2 +𝑎2 +𝑎2 −𝑥 2
= √𝑎2 _𝑥 2
2√𝑎2 −𝑥 2
𝑑𝑦 𝑠𝑖𝑛2 (𝑎+𝑦)
2). If siny = x sin (a + y) then prove that =
𝑑𝑥 𝑠𝑖𝑛𝑎
𝑠𝑖𝑛𝑦
Solution: x= by QUOTIENT FORMULA
sin (𝑎+𝑦)
𝑑𝑦
= (sin(a + y) cosy — siny cos(a + y) = 𝑠𝑖𝑛2 (𝑎 + 𝑦)
𝑑𝑥
𝑑𝑦 𝑠𝑖𝑛2 (𝑎+𝑦)
=
𝑑𝑥 sin (𝑎+𝑦−𝑦)

𝑑𝑦 𝐶𝑂𝑆 2 (𝑎+𝑦)
3). If cosy —xcos(a + y) with cosa ≠ ±1 𝑡ℎ𝑒𝑛 𝑃𝑟𝑜𝑣𝑒 𝑇ℎ𝑎𝑡 =
𝑑𝑥 𝑠𝑖𝑛𝑎
𝑑2𝑦 𝑑𝑦
Hence show sin 𝑎 +sin2(a+y) =0
𝑑𝑥 2 𝑑𝑥

𝑑𝑦 1−𝑦 2
Solution: If √1 − 𝑥 2 +√1 − 𝑦 2 = a (x—y), show that =√
𝑑𝑥 1−𝑥 2

Putting x= sin 𝛼 and y = sin𝛽 then we get


cos 𝛼 + cos 𝛽 = 𝑎 (sin 𝛼 − sin 𝛽)
𝛼+𝛽) 𝛼−𝛽 𝛼−𝛽 𝛼−𝛽
2 cos ( ) cos ( )= 2a cos( ) sin( )
2 2 2 2

𝛼 − 𝛽 = 2𝑐𝑜𝑡 −1 𝑎 𝑠𝑖𝑛−1 𝑥 − 𝑠𝑖𝑛−1 𝑦 = 2𝑐𝑜𝑡 −1 𝑎


𝑑𝑦 1−𝑦 2
=√
𝑑𝑥 1−𝑥 2

𝑑𝑦 𝜋
4). If x = cost (3—2𝑐𝑜𝑠 2 𝑡𝑎𝑛𝑑 𝑦 = 𝑠𝑖𝑛𝑡(3 − 2𝑠𝑖𝑛2 𝑡) Then find 𝑎𝑡 𝑡 =
𝑑𝑥 4
𝑑𝑥
ANS: DIFF both sides = 3𝑠𝑖𝑛𝑡(𝑐𝑜𝑠2𝑡) … … . . .1)
𝑑𝑡
𝑑𝑦
And =3costcos2t ………………….2) from 1 ,2
𝑑𝑡
𝑑𝑦 𝜋
= 𝑐𝑜𝑡𝑡 𝑎𝑡 is = 1
𝑑𝑥 4
31

𝑑𝑦 𝜋 𝜋
5). If x= a sin2t (1 + cos2t) and y=bcos2t(1—cos2t), find 𝑎𝑡 𝑡 = 𝑎𝑛𝑑 𝑡 =
𝑑𝑥 4 3
𝑑𝑥 𝑑𝑦
ANS: Diff w.r.t x = 2𝑎(cos4t+cos2t) and =2b (sin4t—sin2t)
𝑑𝑡 𝑑𝑡
𝑑𝑦 𝑏 𝑠𝑖𝑛4𝑡−𝑠𝑖𝑛2𝑡 𝜋 𝑏 𝜋 𝑏 √3
So, = { } at = and =
𝑑𝑥 𝑎 𝑐𝑜𝑠4𝑡 4 𝑎 3 𝑎
𝑑𝑦
6). 𝐹𝑖𝑛𝑑 , 𝐼𝑓 (𝑥 2 + 𝑦 2 )2 = 𝑥𝑦
𝑑𝑥
𝑑𝑦
Solution: Diff w. r. t x ∶ 4𝑦(𝑥 2 + 𝑦 2 ) − 𝑥) = 𝑦 − 4𝑥(𝑥 2 + 𝑦 2 )
𝑑𝑥

𝑑𝑦 𝑦 − 4𝑥(𝑥 2 + 𝑦 2 )
=
𝑑𝑥 4𝑦(𝑥 2 + 𝑦 2 ) − 𝑥
1−𝑥 𝑥+2
7). Diff w. r. t. x: f(x) = 𝑡𝑎𝑛−1 ( ) − 𝑡𝑎𝑛−1 (1−2𝑥)
1+𝑥
𝑎−𝑏)
Solution: Using 𝑡𝑎𝑛−1 𝑎 − 𝑡𝑎𝑛−1 𝑏 = 𝑡𝑎𝑛−1 ( )
1+𝑎𝑏

𝑡𝑎𝑛−1 1 − 𝑡𝑎𝑛− 𝑥 − 𝑡𝑎𝑛−1 𝑥 − 𝑡𝑎𝑛−1 2


𝑑𝑓 −2
Diff we get =
𝑑𝑥 1+𝑥 2

√1+𝑥 2 −1
8). Differentiate𝑡𝑎𝑛−1 ( )w.r.t. to 𝑡𝑎𝑛−1 𝑥, 𝑤ℎ𝑒𝑛 𝑥 ≠ 0(2013 F)
𝑥

Solution: Let the first function be u and the other be v


Put x = tan𝜃 so 𝑡𝑎𝑛−1 𝑥 = 𝜃
𝑠𝑒𝑐𝜃−1 𝜃 1
U= 𝑡𝑎𝑛−1 { } , u=𝑡𝑎𝑛−1 (𝑡𝑎𝑛 )= 𝑡𝑎𝑛−1 𝑥
𝑡𝑎𝑛𝜃 2 2
𝑑𝑢 1
= ……….1)
𝑑𝑥 2(1+𝑥 2 )
𝑑𝑣 1 𝑑𝑢 1
= ………………2) =
𝑑𝑥 1+𝑥 2 𝑑𝑣 2
𝑑𝑦
9). If 𝑥 𝑦 + 𝑦 𝑥 = 𝑎𝑏 , then find
𝑑𝑥

Solution: Let u=𝑥 𝑦 = 𝑢 𝑎𝑛𝑑𝑦 𝑥 = 𝑣 𝑢 + 𝑣 = 𝑎𝑏 diff w.r.t.x


𝑑𝑢 𝑑𝑣
+ = 0 taking log both sides
𝑑𝑥 𝑑𝑥
𝑑𝑢 𝑦 𝑑𝑦 𝑑𝑣 𝑥 𝑑𝑦
=𝑥 𝑦 { + 𝑙𝑜𝑔𝑥 } … … . . .1) and = 𝑦𝑥 { + 𝑙𝑜𝑔𝑦)} … … .2)
𝑑𝑥 𝑥 𝑑𝑥 𝑑𝑥 𝑦 𝑑𝑥

Adding 1 and 2 and simplify get


32

𝑑𝑦 𝑦𝑥 𝑦−1 + 𝑦 𝑥 𝑙𝑜𝑔𝑦
=− 𝑦
𝑑𝑥 𝑥 𝑙𝑜𝑔𝑥 + 𝑥𝑦 𝑥−1
𝑑2𝑦 𝑑𝑦
10). If 𝑒 𝑦 (𝑥 + 1) = 1 𝑡ℎ𝑒𝑛 𝑠ℎ𝑜𝑤 𝑡ℎ𝑎𝑡 = ( )2
𝑑𝑥 2 𝑑𝑥
𝑑𝑦 𝑑𝑦 −𝑒 𝑦
Solution: Differentiate w.r.t. x(x+1)𝑒 𝑦 = −𝑒 𝑦 =
𝑑𝑥 𝑑𝑥 (𝑥+1)𝑒 𝑦

𝑑𝑦 −1 𝑑2𝑦 1 𝑑𝑦
= (𝑥+1) again, diff get = (1+𝑥)2 = ( )2
𝑑𝑥 𝑑𝑥 2 𝑑𝑥

𝑑2𝑦 𝑑𝑦
11). If log y = 𝑡𝑎𝑛−1 𝑥, 𝑡ℎ𝑒𝑛 𝑠ℎ𝑜𝑤 𝑡ℎ𝑎𝑡 (1+𝑥 2 ) + (2x—1) =0
𝑑𝑥 2 𝑑𝑥
1 𝑑𝑦 1
Differentiating w.r.t x =
𝑦 𝑑𝑥 1+𝑥 2
𝑑𝑦
so (1+𝑥 2 ) =y again diff
𝑑𝑥
𝑑2𝑦 𝑑𝑦
(1+𝑥 2) ) + (2𝑥 − 1) =0
𝑑𝑥 2 𝑑𝑥
𝑑2𝑦
12). If 𝑥 𝑚 𝑦 𝑛 = (𝑥 + 𝑦)𝑚+𝑛 prove =0
𝑑𝑥 2

Solution: Taking log both sides m logx+ n logy = (m + n) log(x + y)


𝑚 𝑛 𝑑𝑦 𝑚+𝑛 𝑑𝑦
Diff + = {1 + }
𝑥 𝑦 𝑑𝑥 𝑥+𝑦 𝑑𝑥
𝑑𝑦 𝑦
After simplifying get = again diff
𝑑𝑥 𝑥
𝑥𝑑𝑦
𝑑2𝑦 −𝑦 0
𝑑𝑥
= = =0
𝑑𝑥 2 𝑥2 𝑋2

𝑚𝑥 𝑑2𝑦 𝑑𝑦
13). If y = A 𝑒 + 𝐵𝑒 𝑛𝑥 , show that − (𝑚 + 𝑛) + 𝑚𝑛𝑦 = 0
𝑑𝑥 2 𝑑𝑥

More Questions for Practice:


𝟏−𝒄𝒐𝒔𝒙
1. Differentiate: 𝒕𝒂𝒏−𝟏 ( ) with respect to x.
𝒔𝒊𝒏𝒙

𝒙
𝟏−𝒄𝒐𝒔𝒙 𝒅𝒚 𝒅 𝟐𝒄𝒐𝒔𝟐
Sol. Let 𝒚 = 𝒕𝒂𝒏−𝟏 ( ) = 𝒕𝒂𝒏 −𝟏
( 𝒙
𝟐
)
𝒙
𝒔𝒊𝒏𝒙 𝒅𝒙 𝒅𝒙 𝟐𝒔𝒊𝒏 𝒄𝒐𝒔
𝟐 𝟐

𝒙
𝒅𝒚 𝒅 𝒄𝒐𝒔 𝒅 𝒙
−𝟏
= 𝒕𝒂𝒏 ( 𝟐
𝒙 ) = 𝒕𝒂𝒏−𝟏 (𝒄𝒐𝒕 )
𝒅𝒙 𝒅𝒙 𝒔𝒊𝒏 𝒅𝒙 𝟐
𝟐

𝒅 𝝅 𝒙 𝒅 𝝅 𝒙 −𝟏
𝒕𝒂𝒏−𝟏 [𝒕𝒂 𝒏 ( − )] = ( − 𝟐) = .
𝒅𝒙 𝟐 𝟐 𝒅𝒙 𝟐 𝟐

2. Show that the function 𝑓(𝑥) = |𝑥 − 3|, 𝑥 ∈ 𝑅, 𝑖𝑠 𝑐𝑜𝑛𝑡𝑖𝑛𝑢𝑜𝑢𝑠 𝑏𝑢𝑡 𝑛𝑜𝑡 𝑑𝑖𝑓𝑓𝑒𝑟𝑒𝑛𝑡𝑖𝑎𝑏𝑙𝑒 𝑎𝑡 𝑥 = 3.
33

𝑥 − 3 𝑖𝑓𝑓 𝑖𝑠 𝑥 ≥ 3
Sol. 𝑓(𝑥) = |𝑥 − 3| = { } 𝑊𝑒 ℎ𝑎𝑣𝑒 𝑓(3) = |3 − 3| =0.
−(𝑥 − 3)𝑖𝑓 𝑓 𝑖𝑠 𝑥 < 3

𝑓(𝑥) = 𝑓(3 + ℎ) = 𝑙𝑖𝑚ℎ→0 (3 + ℎ) − 3 = 0

𝑓(𝑥) = 𝑓(3 − ℎ) = 𝑙𝑖𝑚ℎ→0 − (3 − ℎ − 3) = 0

𝑓(𝑥) = 𝑓(𝑥) = 𝑓(3) So 𝑓(𝑥) is contiuous at 𝑥 = 3.


𝑓(3 + ℎ) − 𝑓(3) (3 + ℎ − 3) − 0 ℎ
Now, 𝑅𝐻𝐷 = 𝑙𝑖𝑚ℎ→0 = 𝑙𝑖𝑚ℎ→0 = 𝑙𝑖𝑚ℎ→0 = 1
ℎ ℎ ℎ

𝑓(3) − 𝑓(3 − ℎ) 0 − (3 − ℎ − 3) ℎ
𝐿𝐻𝐷 = 𝑙𝑖𝑚ℎ→0 = 𝑙𝑖𝑚ℎ→0 = 𝑙𝑖𝑚ℎ→0 = −1
−ℎ −ℎ −ℎ

Since 𝐿𝐻𝐷 ≠ 𝑅𝐻𝐷 so 𝑓(𝑥) 𝑖𝑠 𝑛𝑜𝑡 𝑑𝑖𝑓𝑓𝑒𝑟𝑒𝑛𝑡𝑖𝑎𝑏𝑙𝑒 𝑎𝑡 𝑥 = 3.


𝑑2𝑦 𝜋
3. If 𝑥 = 𝑎 𝑠𝑒𝑐 3 𝜃, 𝑦 = 𝑎 𝑡𝑎𝑛3 𝜃, 𝑡ℎ𝑒𝑛 𝑓𝑖𝑛𝑑 2
at 𝜃 = .
𝑑𝑥 4

𝑑𝑥 𝑑𝑦
Sol. = 𝑎. 3𝑠𝑒𝑐 2 𝜗, 𝑠𝑒𝑐𝜃𝑡𝑎𝑛𝜃 and = 𝑎. 3𝑡𝑎𝑛2 𝜗, 𝑠𝑒𝑐 2 𝜗
𝑑𝜃 𝑑𝜃

𝑑𝑦
𝑑𝑦 𝑑𝜃 𝑎.3𝑡𝑎𝑛2 𝜗,𝑠𝑒𝑐 2 𝜗 𝑡𝑎𝑛𝜃
= 𝑑𝑥 = = 𝑠𝑖𝑛𝜃
𝑑𝑥 𝑎.3𝑠𝑒𝑐 2 𝜗,𝑠𝑒𝑐𝜃𝑡𝑎𝑛𝜃 𝑠𝑒𝑐𝜃
𝑑𝜃

𝑑2𝑦 𝑑𝜃 𝑐𝑜𝑠𝜃 1
2
= 𝑐𝑜𝑠𝜃 = 3
= 𝑐𝑜𝑠 4 𝜃𝑐𝑜𝑡𝜃
𝑑𝑥 𝑑𝑥 3𝑎𝑠𝑒𝑐 𝜃𝑡𝑎𝑛𝜃 3𝑎
𝑑2𝑦 𝜋 1 𝜋 𝜋 1 1 1
(𝑎𝑡 𝑥 = ) = 𝑐𝑜𝑠 4 𝑐𝑜𝑡 = ( )4 . 1 = .
𝑑𝑥 2 4 3𝑎 4 4 3𝑎 √2 12𝑎

𝑑2𝑦 𝑑𝑦
4. If 𝑦 = (𝑠𝑖𝑛−1 𝑥)2 , prove that (1 − 𝑥 2 ) −𝑥 − 2 = 0.
𝑑𝑥 2 𝑑𝑥

𝑑𝑦 1 𝑑𝑦 2 4(𝑠𝑖𝑛−1 𝑥)2 4𝑦 𝑑𝑦 2
Sol. = 2(𝑠𝑖𝑛−1 𝑥). ; ( ) = = ;( ) (1 − 𝑥 2 ) = 4𝑦 .
𝑑𝑥 √1−𝑥 2 𝑑𝑥 (1−𝑥 2 ) 1−𝑥 2 𝑑𝑥

𝑑𝑦 𝑑 2 𝑦 𝑑𝑦 2 𝑑𝑦
Again diff. with respect to x, we get 2 (1 − 𝑥 2 ) + ( ) (−2𝑥) =4
𝑑𝑥 𝑑𝑥 2 𝑑𝑥 𝑑𝑥

𝑑𝑦 𝑑2𝑦 𝑑𝑦 𝑑𝑦 𝑑2𝑦 𝑑𝑦
2
𝑑𝑥
((1 − 𝑥 2 ) 𝑑𝑥 2
− 𝑥.
𝑑𝑥
) = 4 𝑑𝑥 = (1 − 𝑥 2 )
𝑑𝑥 2
−𝑥
𝑑𝑥
− 2 = 0.

𝑑𝑦 𝑙𝑜𝑔𝑥
5. 𝐼𝑓 𝑥 𝑦 =𝑒 𝑥−𝑦 , prove that = .
𝑑𝑥 (1+𝑙𝑜𝑔𝑥)2

Sol. 𝑥 𝑦 = 𝑒 𝑥−𝑦 Taking log on both sides, 𝑦𝑙𝑜𝑔𝑥 = (𝑥 − 𝑦)𝑙𝑜𝑔𝑒


𝑥
𝑦(𝑙𝑜𝑔𝑥 + 1) = 𝑥 𝑜𝑟 𝑦 =
𝑙𝑜𝑔𝑥 + 1
34

𝒅𝒚 𝒍𝒐𝒈𝒙 + 𝟏 −𝟏 𝒍𝒐𝒈𝒙
Diff wrt x , we get = =
𝒅𝒙 (𝟏 + 𝒍𝒐𝒈𝒙)𝟐 (𝟏 + 𝒍𝒐𝒈𝒙)𝟐

𝑑𝑦 2𝑥+1
6. Find , 𝑖𝑓 𝑦 = 𝑠𝑖𝑛−1 ( )
𝑑𝑥 1+4 𝑥

2𝑥+1 2𝑡𝑎𝑛 𝜃
Sol. 𝑠𝑖𝑛𝑦 = (
1+4 𝑥
) . Let 2𝑥 = 𝑡𝑎𝑛𝜃, 𝑠𝑖𝑛𝑦 = 1 + 𝑡𝑎𝑛2 𝜃 or 𝑠𝑖𝑛𝑦 = 𝑠𝑖𝑛2𝜃

𝑑𝑦 2 𝑑 2 2𝑥+1 𝑙𝑜𝑔2
𝑦 = 2𝑡𝑎𝑛−1 2𝑥 . Diff. wrt x, we get = . 2𝑥 = 2𝑥 𝑙𝑜𝑔2 = .
𝑑𝑥 1+(2𝑥 ) 𝑑𝑥
2 1+4 𝑥 1 + 4𝑥

𝑑2𝑦 𝑑𝑦
7. 𝐼𝑓 𝑦 = 3𝑒 2𝑥 + 2𝑒 3𝑥 , prove that −5 + 6𝑦 = 0.
𝑑𝑥 2 𝑑𝑥

𝑑𝑦
Sol. Given, 𝑦 = 3𝑒 2𝑥 + 2𝑒 3𝑥 . = 6𝑒 3𝑥 + 6𝑒 3𝑥 = 6(𝑒 2𝑥 + 𝑒 3𝑥 )
𝑑𝑥

𝑑2𝑦
So, = 12𝑒 3𝑥 + 18𝑒 3𝑥 = 6(2𝑒 2𝑥 + 3𝑒 3𝑥 )
𝑑𝑥 2

𝑑2𝑦 𝑑𝑦
−5 + 6𝑦 = 6(2𝑒 2𝑥 + 3𝑒 3𝑥 )− 30(𝑒 2𝑥 + 𝑒 3𝑥 )+ 6(3𝑒 2𝑥 + 2𝑒 3𝑥 )= 0.
𝑑𝑥 2 𝑑𝑥

𝑑𝑦
8. 𝐹𝑖𝑛𝑑 , 𝑖𝑓 𝑦 𝑥 + 𝑥 𝑦 + 𝑥 𝑥 = 𝑎𝑏
𝑑𝑥

Sol. Given 𝑦 𝑥 + 𝑥 𝑦 + 𝑥 𝑥 = 𝑎𝑏 Put 𝑦 𝑥 = 𝑢, 𝑥 𝑦 = 𝑣 𝑎𝑛𝑑 𝑥 𝑥 = 𝑤

𝑠𝑜 𝑡ℎ𝑎𝑡 𝑢 + 𝑣 + 𝑤 = 𝑎𝑏
𝑑𝑢 𝑑𝑣 𝑑𝑤
Diff. wrt x, we get + + = 0.
𝑑𝑥 𝑑𝑥 𝑑𝑥

Take 𝑦 𝑥 = 𝑢. 𝐴𝑝𝑝𝑙𝑦 𝑙𝑜𝑔 𝑙𝑜𝑔 𝑜𝑛 𝑏𝑜𝑡ℎ 𝑠𝑖𝑑𝑒𝑠, 𝑙𝑜𝑔𝑢 = 𝑥𝑙𝑜𝑔𝑦


1 𝑑𝑢 1 𝑑𝑦 𝑑𝑢 𝑥 𝑑𝑦
Diff. wrt x, we get = 𝑥. + 𝑙𝑜𝑔𝑦. 1 ; = 𝑦 𝑥 (. + 𝑙𝑜𝑔𝑦)
𝑢 𝑑𝑥 𝑦 𝑑𝑥 𝑑𝑥 𝑦 𝑑𝑥

𝑑𝑣 𝑦 𝑑𝑦 𝑑𝑤
Similarly, = 𝑥 𝑦 (. + 𝑙𝑜𝑔𝑥 ) 𝑎𝑛𝑑 = 𝑥 𝑥 (1 + 𝑙𝑜𝑔𝑥)
𝑑𝑥 𝑥 𝑑𝑥 𝑑𝑥

𝑑𝑢 𝑑𝑣 𝑑𝑤 𝑑𝑦 −(𝑦 𝑥 𝑙𝑜𝑔𝑦 + 𝑦.𝑦 𝑥 − 1 + 𝑥 𝑥 (1+𝑙𝑜𝑔𝑥)


Hence, using + + = 0 , 𝑤𝑒 𝑔𝑒𝑡 = .
𝑑𝑥 𝑑𝑥 𝑑𝑥 𝑑𝑥 𝑥.𝑦 𝑥−1 + 𝑥 𝑦 𝑙𝑜𝑔𝑥

9. Find 𝑓 ′ (𝑥)𝑖𝑓 𝑓(𝑥) = 𝑠𝑖𝑛𝑥 𝑠𝑖𝑛𝑥 for all 0 < 𝑥 < 𝜋.

Sol. Let y = 𝑠𝑖𝑛𝑥 𝑠𝑖𝑛𝑥 Taking log on both sides 𝒍𝒐𝒈𝒚 = 𝒍𝒐𝒈 (𝒔𝒊𝒏𝒙)𝒔𝒊𝒏𝒙 = 𝒔𝒊𝒏𝒙 𝒍𝒐𝒈(𝒔𝒊𝒏𝒙)
𝟏 𝒅𝒚 𝒅 𝟏 𝒅
Diff. w.r.to x, = 𝒅𝒙 𝒔𝒊𝒏𝒙 𝒍𝒐𝒈(𝒔𝒊𝒏𝒙) = 𝒄𝒐𝒔𝒙 𝒍𝒐𝒈(𝒔𝒊𝒏𝒙) + 𝒔𝒊𝒏𝒙. 𝒔𝒊𝒏𝒙 𝒅𝒙 𝒔𝒊𝒏𝒙
𝒚 𝒅𝒙

= 𝒄𝒐𝒔𝒙 𝒍𝒐𝒈(𝒔𝒊𝒏𝒙) + 𝒄𝒐𝒔𝒙 = [𝟏 + 𝒍𝒐𝒈 (𝒔𝒊𝒏𝒙)] 𝒄𝒐𝒔𝒙


35

𝒅𝒚
; = 𝒚 {(𝟏 + 𝒍𝒐𝒈 (𝒔𝒊𝒏𝒙)) 𝒄𝒐𝒔𝒙} = (𝟏 +𝒍𝒐𝒈 𝒍𝒐𝒈 (𝒔𝒊𝒏𝒙))𝒔𝒊𝒏𝒙𝒔𝒊𝒏𝒙 𝒄𝒐𝒔𝒙.
𝒅𝒙

10. Differentiate 𝑠𝑖𝑛2 𝑥 𝑤. 𝑟. 𝑡. 𝑒 𝑐𝑜𝑠𝑥 .

𝒅𝒖 𝒅𝒖 𝒅𝒗
Sol. 𝐿𝒆𝒕 𝒖(𝒙) = 𝒔𝒊𝒏𝟐 𝒙 𝒂𝒏𝒅 𝒗(𝒙) = 𝒆𝒄𝒐𝒔𝒙. Now = 𝒅𝒙 / 𝒅𝒙.
𝒅𝒗

𝒅𝒖 𝒅𝒗
= 𝟐𝒔𝒊𝒏𝒙𝒄𝒐𝒔𝒙 𝒂𝒏𝒅 = 𝒆𝒄𝒐𝒔𝒙 (−𝒔𝒊𝒏𝒙) = −(𝒔𝒊𝒏𝒙)𝒆𝒄𝒐𝒔𝒙
𝒅𝒙 𝒅𝒙

𝒅𝒖 𝟐𝒔𝒊𝒏𝒙 𝒄𝒐𝒔𝒙 −𝟐𝒄𝒐𝒔𝒙


So = −(𝒔𝒊𝒏𝒙)𝒆𝒄𝒐𝒔𝒙 =
𝒅𝒗 𝒆𝒄𝒐𝒔𝒙

Chapter - 6 – APPLICATIONS OF DERIVATIVE

1) The side of equilateral triangle is increasing at the rate 2cm / sec at what rate area
will increase when the side is 20 cm
√3
Ans: Let x be the side of equilatral triangle We have A = x2
4

𝑑𝐴 √3 𝑑𝑥 √3 𝑑𝑥
= (2𝑥) = 𝑥
𝑑𝑡 4 𝑑𝑡 2 𝑑𝑡

(dA/dt ) at x = 20 = 20√3 cm2/sec

2) The radius of balloon increases at the rate 10cm/sec at what rate S A of balloon
increases when radius is 15 cm

Ans: let r be the radius of sphere S = 4𝜋𝑟 2

ds/dt = 8πr dr/dt

( ds/dt)at r = 15 = 80(15) π =1200π cm2/sec

3) Sand is pouring from pipe at the rate 12cm3/sec and forms in to cone on ground in
such a way that height is always one sixth of radius of cone how fast height changes
at height is 4 cm
1
Ans: let r and h be the radius and height of cone V = 𝜋𝑟 2 ℎ
3
𝜋
V = 36ℎ2 ℎ 𝑑ℎ/𝑑𝑡 since (r = 6h) V = 12πh3
3

𝑑𝑣 1 𝑑𝑣 1 1
𝑑𝑡
=
3𝜋ℎ2
( 𝑑𝑡 ) at h = 4 = 3𝜋42 = 48𝜋 cm/sec

4) A particle moves along the curve 6y = x3 + 2 find the point on the curve at which y
coordinate is changing 8 times as fast as x coordinate
36

𝑑𝑦 𝑑𝑥
Let required point be P (x, y) Given: =8
𝑑𝑡 𝑑𝑡

𝑑𝑦 𝑑𝑥 𝑑𝑥 𝑑𝑥
Now 6y = x3 + 2 6 = 3x2 6(8 ) = 3x2 3x2 = 48 x = ±4
𝑑𝑡 𝑑𝑡 𝑑𝑡 𝑑𝑡

At x = 4 y = 11 and at x =-4 y = -31/5

INCREASING AND DECREASING FUNCTION


A function f(x) is increasing in (a, b)
if x1 < x2 ⇒ f(x1) ≤ f(x2), ∀ 𝒙𝟏 , 𝒙𝟐 ∈ (a, b)
A function f(x) is strictly increasing in (a, b)
if x1 < x2 ⇒ f(x1) < f(x2), ∀ 𝒙𝟏 , 𝒙𝟐 ∈ (a, b)
A function f(x) is increasing in (a, b) if ∀ 𝒙 ∈ (a, b), f’(x) > 0
A function f(x) is decreasing in (a, b)
if x1 < x2 ⇒ f(x1) ≥ f(x2), ∀ 𝒙𝟏 , 𝒙𝟐 ∈ (a, b)
A function f(x) is strictly decreasing in (a, b)
if x1 < x2 ⇒ f(x1) > f(x2), ∀ 𝒙𝟏 , 𝒙𝟐 ∈ (a, b)
A function f(x) is decreasing in (a, b) if ∀ 𝒙 ∈ (a, b), f’(x) < 0
QUESTIONS
1. Show that the function f(x) = e2x is strictly increasing on R.
ANS: f’(x) > 0, , ∀ 𝒙 ∈ 𝑹
𝝅
2. Show that the function f(x) = cos 2x is strictly decreasing on [𝟎, ]
𝟐
’ 𝝅
ANS: f (x) < 0, f (x) is strictly decreasing in(𝟎, )
𝟐
TWO AND THREE MARKS Questions
1). Find the intervals in which the function f given by f(x) = (x + 1)3 (x – 3)3 is
increasing or decreasing.
ANS: f(x) = (x + 1)3 (x – 3)3
f’ ( x ) = ( x + 1 )3 3 ( x - 3)2 + ( x + 1 )2 3 ( x - 3)3
= (x + 1)2 3 ( x - 3)2 [ x + 1 + x -3]
= 6 (x + 1)2 ( x - 3)2[ x -1 ] >0 , when x > 1
< 0, when x < 1
f(x) is increasing when x > 1 f(x) is decreasing when x < 1
37

2. Find the least value of a such that the function f given by f(x) = x2 + ax + 1 is
strictly increasing on (1, 2).
ANS: f(x) = x2 + ax + 1 f’ (x ) = 2x + a
Given f(x) is strictly increasing in (1, 2)
⇒ f(x) is increasing in [1,2]
⇒f’ ( x ) = 2x + a ≥ 0 , ∀ 𝒙 ∈ [1, 2]
⇒ 2+a≥0
⇒ a≥–2
The least value of a = – 2
3. Show that the function f(x) = cosx is neither increasing nor decreasing in (0, 2𝝅).
ANS: f(x) = cos x
(i)∀ 𝒙 ∈ (𝟎, 𝝅 ) ,sin x > 0 ,
⇒ f’ ( x ) = – sin x < 0
⇒f(x) is strictly decreasing in (𝟎, 𝝅 )
(ii) ∀ 𝒙 ∈ (𝝅, 𝟐𝝅 ) ,sin x < 0,
⇒ f’ ( x ) = – sin x> 0
⇒f(x) is strictly increasing in (𝝅, 𝟐𝝅 )
Therefore from (i) & (ii) we get
f(x) =cos x, ∀ 𝒙 ∈ (𝟎, 𝟐𝝅 ) is neither increasing nor decreasing

ONE MARK Questions


1. Show that the function f(x) = 6x + 5 is strictly increasing on R.
ANS: f(x) = 6x + 5 𝒇, (𝒙) = 𝟔 > 0 , ∀ 𝑥 ∈ 𝑅
2. Show that the function f(x) = x3 – 3x2 + 4x is increasing on R.
ANS: f(x) = x3 – 3x2 + 4x f’(x) = 3x2 -6x +4
= (3( x – 1 ) 2 + 1 ) > 0 , ∀ 𝒙 ∈ 𝑹
3. Find the intervals in which the function f given by f(x) = x2 - 4x + 6 is
(i) strictly increasing (ii) strictly decreasing

2
ANS: f(x) = x - 4x + 6 f (x) = 2 (x – 2 ) > 0 , when x> 2,
f’ (x) = 2 (x – 2 ) < 0 , when x < 2,
(i) strictly increasing in ( 2 , ∞ ) ,
(ii) strictly decreasing in ( −∞, 𝟐 )
4. Find the intervals in which the function f given by f(x) = x3 -6 x2 + 9x + 15 is
(i) strictly increasing (ii) strictly decreasing
3 2
ANS: f(x) = x -6 x + 9x + 15 f’( x ) = 3x2 – 12x + 9 = 3 (x-1) (x-3)
f’( x ) =0 ⇒ x=1 ,x= 3

−∞, 1 3 +∞,
38

interval Sign of f’( x ) ↑ 𝒐𝒓 ↓


( −∞, 1) + ↑
(1, 3) – ↓
( 3, ∞) + ↑
(i) f(x) is strictly increasing in( −∞, 1) ∪( 3, ∞)
(ii) f(x) is strictly decreasing in (1,3)
5. Find the intervals in which the function f given by f(x) = 2X 3 – 9x2 + 12x + 15 is
(i) increasing (ii) decreasing
ANS: f(x) = 2X3 – 9x2 + 12x + 15 f’( x ) = 6x2 – 18x + 12 = 6 (x – 1) (x – 2)
f’( x ) = 0 ⇒ x = 1, x = 2

interval Sign of f’( x ) ↑ 𝒐𝒓 ↓


( −∞, 1] + ↑
[1, 2] – ↓
[ 2, ∞) + ↑

(i) f(x) is increasing in ( −∞, 1] ∪[2, ∞)


(ii) f(x) is decreasing in [1, 2]
6. Find the intervals in which the function f given by f(x) = 2x2 -3x is
(i) strictly increasing (ii) strictly decreasing
ANS: f(x) = 2x2 – 3x f’(x) = 4x – 3
f’(x)> 0, when 4x – 3>0 ⇒ x > 3/4
f’(x)< 0, when 4x – 3 < 0 ⇒ x < 3/4

interval Sign of f’( x ) ↑ 𝒐𝒓 ↓


( −∞, 𝟑/𝟒) - ↓
(3/4, ∞) + ↑

(i) f(x) is strictly decreasing in( - ∞ , 𝟑/𝟒),


(ii) f(x) is strictly increasing in (𝟑/𝟒), ∞)
7. Find the intervals in which the function f given by f(x) = 6 – 9x – x2 is
(i) strictly increasing (ii) strictly decreasing
ANS: f(x) = 6 – 9x – x 2
f’(x) = – 9 – 2x
39

f’(x)> 0, when – (9 + 2x ) > 0 ⇒ (𝟗 + 𝟐𝐱 ) < 𝟎 ⇒ x < -9/2


f’(x)< 0, when – (9 + 2x ) < 0 ⇒ (𝟗 + 𝟐𝐱 ) > 𝟎 ⇒ x > -9/2

interval Sign of f’( x ) ↑ 𝒐𝒓 ↓


( −∞, −𝟗/𝟐) – ↓
(-9/2, ∞) + ↑

(i) f(x) is strictly increasing in ( - ∞ , −𝟗/𝟐),


(ii) f(x) is strictly decreasing in (-9/2, ∞)
THREE MARKS or FIVE MARKS Questions
𝒙
1. Find the intervals in which the function f given by f(x) = is
𝟏 + 𝒙𝟐
(i) increasing (ii) decreasing
𝒙 (𝟏 + 𝒙𝟐 ).𝟏 − 𝒙.𝟐𝒙 (𝟏 − 𝒙𝟐 ) (𝟏 + 𝐱) (𝟏 − 𝐱)
ANS: f(x) = f’( x ) = 𝟐 = 𝟐 = 𝟐
𝟏 + 𝒙𝟐 (𝟏 + 𝒙𝟐 ) (𝟏 + 𝒙𝟐 ) (𝟏 + 𝒙𝟐 )
f’( x ) = 0 ⇒ (1+x) (1-x) =0
⇒ x = – 1, x = 1

−∞, -1 1 +∞,

Interval Sign of f’( x ) ↑ 𝒐𝒓 ↓


( −∞, −1] – ↓
[-1, 1] + ↑
[ 1, ∞) – ↓

(i) f(x) is decreasing in( −∞, − 1] ∪[1, ∞)


(ii) f(x) is increasing in [-1, 1]
2. Find the intervals in which the function f(x) = sin x +cos x in [0 , 2 𝝅].
(i) strictly increasing (ii) strictly decreasing
ANS: f(x) = sin x + cos x is [0 , 2 𝝅]. f’( x ) = cos x – sin x
f’( x ) =0 ⇒ tan x = 1 , ∀ 𝒙 ∈ [𝟎, 𝟐𝝅 ]
𝝅 𝟓𝝅
⇒ x= ,
𝟒 𝟒

𝟎 𝝅 𝟓𝝅
𝟒 𝟐𝝅
𝟒
40

Interval Sign of f’( x ) ↑ 𝒐𝒓 ↓


𝝅 + ↑
[𝟎, ]
𝟒
𝝅 𝟓𝝅 – ↓
[ , ]
𝟒 𝟒
𝟓𝝅 + ↑
[ , 𝟐𝝅]
𝟒
𝝅 𝟓𝝅
(i) f(x) is strictly increasing in [𝟎, ] ∪ [ , 𝟐𝝅]
𝟒 𝟒
𝝅 𝟓𝝅
(ii) f(x) is strictly decreasing in [ , ]
𝟒 𝟒
𝒙
3. Find the intervals in which the function f given by f(x) = log (1+ x) – ,
𝟏+𝒙

x ≠ -1 is (i) increasing (ii) decreasing


𝒙 (𝟏+𝒙)−𝟏
ANS: f(x) = log (1+x) – = log (1+x) –
𝟏+𝒙 𝟏+𝒙
𝟏
= log (1 + x) – 1 +
𝟏+𝒙
𝟏 𝟏 𝒙
f ’( x ) = − (𝟏+𝒙)𝟐
= (𝟏+𝒙)𝟐
𝟏+𝒙

here, (1+ x )2> 0 ∀ 𝒙 ∈R - {- 1}


(i) f(x) is increasing in( 0, ∞), (ii) f(x) is decreasing in( −∞, 𝟎)
𝟏
4. Find the intervals in which the function f given by f(x) = x3 – is
𝒙𝟑

(i) increasing (ii) decreasing


𝟏
ANS: given f(x) = x3 –
𝒙𝟑

’ 2 𝟑 (𝒙𝟔 −𝟏)𝟑 𝟑(𝒙𝟐 − 𝟏)(𝒙𝟒 + 𝒙𝟐 + 𝟏)


f ( x ) = 3x − (𝒙)𝟒
= =
𝒙𝟒 𝒙𝟒

f ’( x ) = 0 ⇒ ( x2 – 1 ) = 0 ⇒ (1 + x) (x – 1) = 0 ⇒ x = – 1, x = 1

−∞, -1 1 +∞,

Interval Sign of f’( x ) ↑ 𝒐𝒓 ↓


( −∞, −1] – ↓
[– 1 , 1] + ↑
[ 1, ∞) – ↓
41

(i) f(x) is increasing in ( −∞, −1] ∪ [1, ∞) (ii) f(x) is decreasing in [-1,1]
5. Find the intervals in which the function f given by f(x) = sin x - cos x
is [0, 2 𝝅]. (i) strictly increasing (ii) strictly decreasing

ANS: f(x) = sin x – cos x f ( x ) = cos x +sin x
f’( x ) = 0 ⇒ tan x = – 1 , ∀ 𝒙 ∈ [𝟎, 𝟐𝝅 ]
𝟑𝝅 𝟕𝝅
⇒x= ,
𝟒 𝟒

𝟎 𝟑𝝅 𝟕𝝅
𝟐𝝅
𝟒 𝟒
Interval Sign of f’( x ) ↑ 𝒐𝒓 ↓
𝟑𝝅 + ↑
[𝟎, ]
𝟒
𝟑𝝅 𝟕𝝅 – ↓
[ , ]
𝟒 𝟒
𝟕𝝅 + ↑
[ , 𝟐𝝅]
𝟒

𝟑𝝅 𝟕𝝅
(i) f(x) is strictly increasing in[𝟎, ]∪[ , 𝟐𝝅]
𝟒 𝟒
𝟑𝝅 𝟕𝝅
(ii) f(x) is strictly decreasing in [ , ]
𝟒 𝟒
𝟒 𝐬𝐢𝐧 𝜽 𝝅
6. Prove that 𝒚 = – 𝜽 is an increasing function in [𝟎, ]
𝟐 + 𝐜𝐨𝐬 𝜽 𝟐
𝟒 𝐬𝐢𝐧 𝜽
ANS: Given: 𝒚 = –𝜽
𝟐 + 𝐜𝐨𝐬 𝜽
𝒅𝒚 (𝟐 + 𝒄𝒐𝒔 𝜽)𝟒𝒄𝒐𝒔 𝜽 − 𝟒 𝐬𝐢𝐧 𝜽(𝜽 − 𝐬𝐢𝐧 𝜽)
= −𝟏
𝒅𝒙 (𝟐 + 𝐜𝐨𝐬 𝜽)𝟐
𝟖𝒄𝒐𝒔 𝜽 + 𝟒𝒄𝒐𝒔𝟐 𝜽 + 𝟒𝒔𝒊𝒏𝟐 𝜽 − (𝟐 + 𝐜𝐨𝐬 𝜽)𝟐
= (𝟐 + 𝐜𝐨𝐬 𝜽)𝟐
𝟖𝒄𝒐𝒔 𝜽 + 𝟒 − 𝟒 − 𝒄𝒐𝒔𝟐 𝜽 − 𝟒𝒄𝒐𝒔 𝜽
= (𝟐+𝐜𝐨𝐬 𝜽)𝟐
𝟒𝒄𝒐𝒔 𝜽 − 𝒄𝒐𝒔𝟐 𝜽 𝒄𝒐𝒔 𝜽(𝟒 − 𝐜𝐨𝐬 𝜽)
= (𝟐 + 𝐜𝐨𝐬 𝜽)𝟐
= (𝟐 + 𝐜𝐨𝐬 𝜽)𝟐
𝒅𝒚 𝝅
>0 𝜽 ∈ [𝟎, ] ⇒ 𝒄𝒐𝒔 𝜽 > 0 ⇒ 4 - 𝒄𝒐𝒔 𝜽>0 as -1≤ 𝒄𝒐𝒔 𝜽 ≤ 𝟏
𝒅𝒙 𝟐

More Questions & Solutions


𝟑
1. Find the intervals in which the function f given by f(x) = x4 – 4x3 – 45x2 + 51 is
𝟐
(i) strictly increasing (ii) strictly decreasing
𝟑
ANS: f(x) = x4 – 4x3 – 45x2 + 51
𝟐

f (x ) = 6X3 – 12 x2 – 90x = 6x (x + 3) (x – 5)
42

f’( x ) = 0 ⇒ x = 0, x = – 3, x = 5
interval Sign of f’( x ) ↑ 𝒐𝒓 ↓
( −∞, – 3 ) – ↓
(–3, 0) + ↑
(0, 5) – ↓
( 5, ∞) + ↑

(i) strictly increasing in ( –3, 0) ∪( 5, ∞ ),


(ii) (ii) strictly decreasing in (−∞, –3) ∪( 0, 5 )
2. Show that the function f(x) = x3 – 3x2 + 4x is increasing on R.
ANS; f(x) = x3 – 3x2 + 4x f’ (x) = 3x2 – 6x + 4
= [3 ( x – 1 ) 2 + 1 ] > 0 , ∀ 𝒙 ∈ 𝑹
3. f’( x ) = [ x (x – 2) ] (2x – 2) = 4x (x – 1) (x – 2)
f’( x ) = 0 ⇒ x = 0, x = 1, x = 2

−∞, 0 1 2 +∞,
interval Sign of1f’( x ) ↑ 𝒐𝒓 ↓
( −∞, 0] – ↓
[0, 1] + ↑
[1, 2] – ↓
[ 2, ∞) + ↑

f(x) is increasing in [0, 1] ∪ [2 , ∞)


4. Find the intervals in which the function f given by f(x) = 3x4 – 4x3 – 12x2 + 5 is
(i) strictly increasing (ii) strictly decreasing
ANS: f(x) = 3x4 – 4x3 – 12x2 + 5
f’( x ) = 12X3 – 12 x2 – 24x = 12x (x + 1) (x – 2)
f’( x ) = 0 ⇒ x = 0, x = –1, x = 2

−∞, -1 0 2 +∞,

Interval Sign of f’( x ) ↑ 𝒐𝒓 ↓


( −∞, –1) – ↓
(–1, 0) + ↑
(0, 2) – ↓
( 2, ∞) + ↑
43

(i) f(x) is strictly increasing in ( −𝟏, 0) ∪ ( 2, ∞)


(ii) f(x) is strictly decreasing in ( −∞, −1) ∪ (0, 𝟐)
5. Find the intervals in which the function f given by f(x) = (X+1)3 (X-3)3is
(i) increasing (ii) decreasing
ANS: f(x) = (x + 1)3 (x – 3)3 f’( x ) = 6 (x + 1)2 (x – 3)2 (x – 1)
f’( x ) >0 ⇒ (x – 1) > 0 , [ (x + 1)2 (x – 3)2> 0 , V ∈ 𝑹
⇒x>1 f’( x ) =0 ⇒ x = –1, x = –3, x = 1

interval Sign of f’( x ) ↑ 𝒐𝒓 ↓


( −∞, –1) – ↓
(1, 3) + ↑
(–1,1) – ↓
( 3, ∞) + ↑

(i) f(x) is increasing in(1,3) ∪ (3, ∞)


(ii) f(x) is decreasing in ( −∞, −1) ∪ (-1,1)
6. Find the intervals in which the function f given by f(x) =X3 -6 x2 +9x + 15 is
(i) strictly increasing (ii) strictly decreasing
ANS: (x) = x – 6x + 9x + 15
3 2
f’( x ) = 3x2 – 12x + 9 = 3 (x – 1) (x – 3)
f’( x ) = 0 ⇒ x = 1, x = 3

interval Sign of f’( x ) ↑ 𝒐𝒓 ↓


( −∞, 1) + ↑
(1, 3) – ↓
( 3, ∞) + ↑
(i) f(x) is strictly increasing in( −∞, 1) ∪ ( 3, ∞)
(ii) f(x) is strictly decreasing in (1,3)
7. Find the intervals in which the function f given by f(x) = 2x3 – 9x2 + 12x + 15 is
(i) increasing (ii) decreasing
ANS: f(x) = 2x – 9x + 12x + 15
3 2

f’( x ) = 6x2 – 18x + 12 = 6 (x – 1) (x – 2) f’( x ) = 0 ⇒ x = 1, x = 2

−∞, 1 2 +∞,

interval Sign of f’( x ) ↑ 𝒐𝒓 ↓


(−∞, 1] + ↑
[1, 2] – ↓
[ 2, ∞) + ↑
44

(i) f(x) is increasing in(−∞, 1] ∪ [2 , ∞)


(ii) f(x) is decreasing in [1, 2]
𝒙
8. Find the intervals in which the function f given by f(x) = is
𝟏 + 𝒙𝟐
(i) increasing (ii) decreasing
𝒙
ANS: Given f(x) =
𝟏+𝒙𝟐
(𝟏 + 𝒙𝟐 ).𝟏 − 𝒙 . 𝟐𝒙 (𝟏 − 𝒙𝟐 ) (𝟏 + 𝐱) (𝟏 − 𝐱)
f’( x ) = 𝟐 = 𝟐 = 𝟐
(𝟏 + 𝒙𝟐 ) (𝟏 + 𝒙𝟐 ) (𝟏 + 𝒙𝟐 )
(𝟏 + 𝒙𝟐 ).𝟏 − 𝒙 . 𝟐𝒙 (𝟏 − 𝒙𝟐 ) (𝟏 + 𝐱) (𝟏 − 𝐱)
f’( x ) = 𝟐 = 𝟐 = 𝟐
(𝟏 + 𝒙𝟐 ) (𝟏 + 𝒙𝟐 ) (𝟏 + 𝒙𝟐 )
f’( x ) = 0 ⇒ (1 + x) (1 – x) = 0
⇒ x = –1, x = 1

−∞, -1 1 +∞,
interval Sign of f’( x ) ↑ 𝒐𝒓 ↓
( −∞, −1] – ↓
[–1,1] + ↑
[ 1, ∞) – ↓

(i) f(x) is decreasing in (−∞, − 1] ∪ [1, ∞)


(ii) f(x) is increasing in [–1, 1]
9. Find the intervals in which the function f(x) = sin x + cos x in [0, 2 𝝅].
(i) strictly increasing (ii) strictly decreasing
ANS: f(x) = sin x +cos x in [0, 2 𝝅]
f’( x ) = cos x – sin x
𝝅 𝟓𝝅
f’( x ) =0 ⇒ tan x = 1 , ∀ 𝒙 ∈ [𝟎, 𝟐𝝅 ] ⇒x= ,
𝟒 𝟒

𝟎 𝝅 𝟓𝝅
𝟒 𝟐𝝅
𝟒
interval Sign of f’( x ) ↑ 𝒐𝒓 ↓
𝝅 + ↑
[𝟎, ]
𝟒
𝝅 𝟓𝝅
[ , ] – ↓
𝟒 𝟒
𝟓𝝅 + ↑
[ , 𝟐𝝅]
𝟒

𝝅 𝟓𝝅 𝝅 𝟓𝝅
(i) f(x) is strictly inc in[𝟎, ] ∪ [ , 𝟐𝝅] (ii) f(x) is strictly dec in [ , ]
𝟒 𝟒 𝟒 𝟒
45

10. Find the intervals in which the function f(x) = (x + 2) e-x is increasing or
decreasing ?
ANS: f(x) = (x+2) e-x f’( x ) = – {x + 2 ) e-x + e-x = – {x + 1 ) e-x
Here e-x > 0 , ∀ 𝒙 ∈R
Hence f’( x ) ≥ 0, when (x + 1) ≤ 0 Ie, when x ≤ –1
Hence f(x) is increasing in (– ∞ , −𝟏],
f’( x ) ≤ 0, when (x + 1) ≥ 0 i. e, when x ≥ –1

−∞ -1 +∞
+ ∞,
interval Sign of f’( x ) ↑ 𝒐𝒓 ↓
( −∞, −1] + ↑
[ –1, ∞) – ↑

(i) f(x) is increasing in ( – ∞ , −𝟏],


(ii) (i) f(x) is decreasing in [−𝟏, ∞)
𝒙
11. Find the intervals in which the function f given by f(x) = log ( 1 + x) – ,
𝟏+𝒙
x ≠ –1 is(i) increasing (ii) decreasing

𝒙
ANS: f(x) = log ( 1 + x) –
𝟏+𝒙
𝒙 (𝟏 + 𝒙 ) − 𝟏 𝟏
f(x) = log (1 + x) – = log (1 + x) – = log (1 + x) – 1 +
𝟏+𝒙 𝟏+𝒙 𝟏+𝒙
’ 𝟏 𝟏 𝒙
f(x)= − (𝟏 + 𝒙)𝟐
= (𝟏
𝟏+𝒙 + 𝒙)𝟐
here, (1 + x )2 > 0 ∀ 𝒙 ∈R – { –1}
(i) f(x) is increasing in ( 0, ∞),
(ii) f(x) is decreasing in (−∞, 𝟎)
𝟏
12. Find the intervals in which the function f given by f(x) = x3 – is
𝒙𝟑
(i) increasing (ii) decreasing
𝟏
ANS: f(x) = x3 –
𝒙𝟑
𝟑 (𝒙𝟔 − 𝟏)𝟑
f’( x ) = 3x2 − (𝒙)𝟒
=
𝒙𝟒
𝟑 (𝒙𝟐 − 𝟏)(𝒙𝟒 + 𝒙𝟐 + 𝟏)
=
𝒙𝟒
46

f ’( x ) = 0 ⇒ ( x2 – 1 ) = 0 ⇒ (1 + x) (x – 1) = 0
⇒ x = –1, x = 1

−∞, -1 1 +∞,

interval Sign of f’( x ) ↑ 𝒐𝒓 ↓


( −∞, −1] – ↓
[–1, 1] + ↑
[ 1, ∞) – ↓
(i) f(x) is increasing in (−∞, −1] ∪ [1, ∞)
(ii) f(x) is decreasing in [–1, 1]
13. Find the intervals in which the function f given by f(x) = sin x – cos x
is [0, 2 𝝅]. (i) strictly increasing (ii) strictly decreasing

ANS: f(x) = sin x – cos x f ( x ) = cos x + sin x
𝟑𝝅 𝟕𝝅
f’( x ) = 0 ⇒ tan x = –1 , ∀ 𝒙 ∈ [𝟎, 𝟐𝝅 ] ⇒ x = ,
𝟒 𝟒

𝟕𝝅
𝟎 𝟑𝝅 𝟐𝝅
𝟒
𝟒

Interval Sign of f’( x ) ↑ 𝒐𝒓 ↓


𝟑𝝅 + ↑
[𝟎, ]
𝟒
[
𝟑𝝅
,
𝟕𝝅
]] – ↓
𝟒 𝟒

𝟕𝝅 + ↑
[ , 𝟐𝝅]
𝟒

𝟑𝝅 𝟕𝝅
(i) f(x) is strictly increasing in [𝟎, ] ∪[ , 𝟐𝝅]
𝟒 𝟒
𝟑𝝅 𝟕𝝅
(ii) f(x) is strictly decreasing in [ , ]
𝟒 𝟒
𝟒 𝐬𝐢𝐧 𝜽 𝝅
14. Prove that 𝒚 = – 𝜽 is an increasing function in [𝟎, ]
𝟐 + 𝐜𝐨𝐬 𝜽 𝟐
𝟒 𝐬𝐢𝐧 𝜽 𝟒 𝐬𝐢𝐧 𝜽
ANS: Given: 𝒚 = 20. 𝒚= –𝜽
𝟐 + 𝐜𝐨𝐬 𝜽 𝟐 + 𝐜𝐨𝐬 𝜽
𝒅𝒚 (𝟐 + 𝒄𝒐𝒔 𝜽)𝟒𝒄𝒐𝒔 𝜽 − 𝟒 𝐬𝐢𝐧 𝜽(𝜽 − 𝐬𝐢𝐧 𝜽)
= −𝟏
𝒅𝒙 (𝟐 + 𝐜𝐨𝐬 𝜽)𝟐
47

𝟖𝒄𝒐𝒔 𝜽 + 𝟒𝒄𝒐𝒔𝟐 𝜽 + 𝟒𝒔𝒊𝒏𝟐 𝜽 − (𝟐 + 𝐜𝐨𝐬 𝜽)𝟐


= (𝟐 + 𝐜𝐨𝐬 𝜽)𝟐
𝟖𝒄𝒐𝒔 𝜽 + 𝟒 − 𝟒 − 𝒄𝒐𝒔𝟐 𝜽 − 𝟒𝒄𝒐𝒔 𝜽
= (𝟐 + 𝐜𝐨𝐬 𝜽)𝟐
𝟒𝒄𝒐𝒔 𝜽 − 𝒄𝒐𝒔𝟐 𝜽 𝒄𝒐𝒔 𝜽(𝟒 − 𝐜𝐨𝐬 𝜽)
= (𝟐 + 𝐜𝐨𝐬 𝜽)𝟐
= (𝟐 + 𝐜𝐨𝐬 𝜽)𝟐
𝒅𝒚 𝝅
>0 𝜽 ∈ [𝟎, ] ⇒ 𝒄𝒐𝒔 𝜽 > 0 ⇒ 4 - 𝒄𝒐𝒔 𝜽>0 as -1≤ 𝒄𝒐𝒔 𝜽 ≤ 𝟏
𝒅𝒙 𝟐
MAXIMUM AND MINIMUM: (Second Derivative Test )

Let f be a continuous function in an open interval I.

f ( x 0 ) is said to be Local Maximum at x 0 if


f ' ’ (x)
(i). f ‘ ( x 0 ) = 0, (ii). f ‘’( x 0 ) is – ve . – ve

The point is x0 is called Point of Maximum.


x0

Let f be a continuous function in an open interval I.

f ( x 0 ) is said to be Local Minimum at x 0 if


f ’ ‘ (x)
(i). f ‘ ( x 0 ) = 0, (ii). f ‘’( x 0 ) is + ve . +ve

The point is x0 is called Point of Minimum.


x0

1. Find the absolute maximum and minimum values, if any, of f(x)= x-2, x𝝴[2,3]
Sol : Given f(x) = x – 2 on [2, 3] f’(x) = 1 on [2,3]
clearly, there is no value of x 𝝴 [2,3] for which f’(x) = 0
also f’(x) is defined for all x 𝝴 [2,3].
So, the possible points for absolute maximum value and absolute minimum values
are 2 and 3 respectively.
Now f(2) = 2 – 2 = 0 f(3) = 3 – 2 = 1
Hence, absolute maximum value is 1 which occurs at x = 3 and
absolute minimum value of f(x) is 0 which occurs at 2.
2). Find the absolute max and min values of f(x) = 2𝒙𝟑 –15𝒙𝟐 + 36x + 1, x 𝝴 [1, 5].
Sol: Given that f(x) = 2𝒙𝟑 – 15𝒙𝟐 + 36x + 1, x 𝝴 [ 1, 5].
f’(x)= 6𝒙𝟐 − 𝟑𝟎𝒙 + 𝟑𝟔 = 6(x – 3) (x -2).
48

Now, f’(x) = 0 6(x – 3) (x – 2) = 0 x = 2, 3 𝝴 [1, 5]


Also f’(x) is defined for all x𝝴 [1, 5].
So, the possible points for absolute maximum and absolute minimum are 2,3,1,5.
Now f(2) = 29. f(3) = 28. f(1) = 24. f(5) = 56.
So, absolute maximum value is 56 and absolutely minimum value is 24.
3). Find the local maxima and local minima of the function f(x) = 𝒆𝒙 .
Sol: f(x) = 𝒆𝒙 f’(x)= 𝒆𝒙
to find local maxima and local minima
f’(x)= 0 𝒆𝒙 = 0 No solution
Hence f(x) = 𝒆𝒙 has no local maximum or local minimum values.
THREE marks questions:
1. Show that given function does not have local maxima or local minima points and
values. f (x)= 𝒙𝟑 + 18𝒙𝟐 + 𝟏𝟒𝟒𝒙 + 𝟓
Sol: f(x) = 𝒙𝟑 + 18𝒙𝟐 + 𝟏𝟒𝟒𝒙 + 𝟓
f’(x) = 3𝒙𝟐 + 36𝒙 + 144
For local maxima and local minima put
f’(x) =0 3𝒙𝟐 + 36𝒙 + 144 = 0
Here, D = 𝒃𝟐 -4ac
D=144 – 192 = – 48 < 0 Hence f’(x) has no solution.

2. The length x of a rectangle is decreasing at the rate of 5 cm/ min. and the width y is
increasing at the rate of 4 cm /min. When x = 8 cm and y = 6 cm, find the change
of area of rectangle.
𝒅𝒙 𝒅𝒚
Sol: Given = – 5 cm/min and = 4cm/min
𝒅𝒕 𝒅𝒕

Let A be the area of rectangle


Then A = xy Differentiating w.r.to t,
𝒅𝑨 𝒅𝒚 𝒅𝒙
= x. + y.
𝒅𝒕 𝒅𝒕 𝒅𝒕

= x . 4 + y. (–5)
= 4x – 5y
49

At x = 8 cm and y = 6 cm
𝒅𝑨
= 4(8) – 5 (6) = 32 – 30 = 2 𝒄𝒎𝟐 /min.
𝒅𝒕

1. Of all rectangles inscribed in a circle show that square has maximum


area.
A B
SOL: 𝑳𝒆𝒕 𝑨𝑩𝑪𝑫 𝒃𝒆 𝒕𝒉𝒆 𝒓𝒆𝒄𝒕𝒂𝒏𝒈𝒍𝒆,
y
𝒍𝒆𝒏𝒈𝒕𝒉 = 𝒙, 𝑩𝒓𝒆𝒂𝒅𝒕𝒉 = 𝒚 𝒙𝟐 + 𝒚𝟐 = (𝟐𝒓)𝟐 x B
D C

 𝒚𝟐 = (𝟐𝒓)𝟐 − 𝒙𝟐 , 𝒚 = √𝟒𝒓𝟐 − 𝒙𝟐 …..(1)


𝑨𝒓𝒆𝒂 = 𝒍𝒃 = 𝒙√𝟒𝒓𝟐 − 𝒙𝟐  𝑨 = 𝒙𝟐 (𝟒𝒓𝟐 − 𝒙𝟐 )  𝑨 = 𝟒𝒓𝟐 𝒙𝟐

𝑨′ = 𝟖𝒓𝟐 𝒙 − 𝟒𝒙𝟑  𝑭𝒐𝒓 𝑴𝒂𝒙 𝒐𝒓 𝑴𝒊𝒏 𝑨′ = 𝟎

𝟖𝒓𝟐 𝒙 = 𝟒𝒙𝟑 → 𝟐𝒓𝟐 = 𝒙𝟐 𝒙 = √𝟐 𝒓

𝑨′′ = 𝟖𝒓𝟐 − 𝟏𝟐𝒙𝟐  𝑨′′(√𝟐 𝒓) = 𝟖𝒓𝟐 − 𝟏𝟐(𝟐𝒓𝟐 ) = −𝒗𝒆

𝑻𝒉𝒆 𝒂𝒓𝒆𝒂 𝒐𝒇 𝒕𝒉𝒆 𝒓𝒆𝒄𝒕𝒂𝒏𝒈𝒍𝒆 𝒊𝒔 𝑴𝒂𝒙 𝒘𝒉𝒆𝒏 𝒙 = √𝟐 𝒓,

 𝒚 = √𝟒𝒓𝟐 − 𝒙𝟐 , 𝒚= √𝟒𝒓𝟐 − 𝟐𝒓𝟐 𝒚= √𝟐 𝒓


𝒙 = 𝒚, 𝑨𝑩𝑪𝑫 𝒊𝒔 𝒂 𝒔𝒒𝒖𝒂𝒓𝒆. 𝑻𝒉𝒆 𝒂𝒓𝒆𝒂 𝒊𝒔 𝑴𝒂𝒙 𝒘𝒉𝒆𝒏 𝒊𝒕 𝒊𝒔 𝒔𝒒𝒖𝒂𝒓𝒆.
2. A wire of length of 28 cm is to be cut into two pieces one of the two
pieces is to made into square and other into a circle. What should be the
lengths of two pieces so that combined area of the square and circle is
minimum?

SOL: 𝑮𝒊𝒗𝒆𝒏 𝑳𝒆𝒏𝒈𝒕𝒉 𝒐𝒇 𝒕𝒉𝒆 𝒘𝒊𝒓𝒆 = 𝟐𝟖 𝒄𝒎

𝑳𝒆𝒕 𝒕𝒉𝒆 𝒍𝒆𝒏𝒈𝒕𝒉 𝒐𝒇 𝒘𝒊𝒓𝒆 𝒖𝒔𝒆𝒅 𝒇𝒐𝒓 𝒄𝒊𝒓𝒄𝒍𝒆 = 𝒙

2𝝅r=x 4a=28 – x

𝑨𝟏 = 𝝅𝒓𝟐 𝑨𝟐 = 𝒂𝟐
x
50

𝒙 𝒙𝟐 𝒙𝟐
𝟐 𝝅𝒓 = 𝒙  𝒓 = 𝟐
, 𝑨𝒓𝒆𝒂 𝒐𝒇 𝑪𝒊𝒓𝒄𝒍𝒆 = 𝑨𝟏 = 𝝅𝒓 = 𝝅 ( )=
𝟐𝝅 𝟒 𝝅𝟐 𝟒𝝅

𝑳𝒆𝒕 𝒍𝒆𝒏𝒈𝒕𝒉 𝒐𝒇 𝒕𝒉𝒆 𝒘𝒊𝒓𝒆 𝒖𝒔𝒆𝒅 𝒇𝒐𝒓 𝒔𝒒𝒖𝒂𝒓𝒆 = 𝟐𝟖 − 𝒙,

𝟐𝟖 − 𝒙 𝟐𝟖 − 𝒙 𝟐
𝟒𝒂 = 𝟐𝟖 − 𝒙  𝒂 = 𝟐
, 𝑨𝒓𝒆𝒂 𝒐𝒇 𝑺𝒒𝒖𝒂𝒓𝒆 = 𝑨𝟐 = 𝒂 = ( )
𝟒 𝟒
𝒙𝟐 𝟐𝟖−𝒙 𝟐 𝟐𝒙 𝟐𝟖− 𝒙 −𝟏
𝑺𝒖𝒎 𝒐𝒇 𝒂𝒓𝒆𝒂𝒔 = 𝑨 = 𝑨𝟏 + 𝑨𝟐 = +( ) , 𝑨′ = + 𝟐( )( )
𝟒𝝅 𝟒 𝟒𝝅 𝟒 𝟒

𝒙 𝟐𝟖 − 𝒙 −𝟏 𝒙 𝟐𝟖 − 𝒙 – 𝟏
A′ = +( )( ) 𝑭𝒐𝒓 𝑴𝒂𝒙 𝒐𝒓 𝑴𝒊𝒏𝑨′ = 𝟎, +( ) ( ) = 𝟎,
𝟐𝝅 𝟒 𝟐 𝟐𝝅 𝟒 𝟐
𝒙 𝟐𝟖 − 𝒙 𝟏 𝟐𝟖𝝅
=( ) ( )  𝟒𝒙 = 𝟐𝟖𝝅 − 𝝅𝒙,  𝒙(𝝅 + 𝟒) = 𝟐𝟖𝝅  𝒙 =
𝟐𝝅 𝟒 𝟐 𝝅+𝟒
𝟏 𝟏 𝟐𝟖𝝅 𝟐𝟖𝝅
𝑨′′ = + ( ), 𝑨′′ ( ) = +𝒗𝒆  𝑺𝒖𝒎 𝒐𝒇 𝒂𝒓𝒆𝒂𝒔 𝒊𝒔 𝑴𝒊𝒏 𝒂𝒕 𝒙 =
𝟐𝝅 𝟖 𝝅+𝟒 𝝅+𝟒
𝟐𝟖𝝅
𝑳𝒆𝒕 𝒕𝒉𝒆 𝒍𝒆𝒏𝒈𝒕𝒉 𝒐𝒇 𝒘𝒊𝒓𝒆 𝒖𝒔𝒆𝒅 𝒇𝒐𝒓 𝒄𝒊𝒓𝒄𝒍𝒆 = 𝒙 = cm
𝝅+𝟒
𝟐𝟖𝝅
𝑳𝒆𝒏𝒈𝒕𝒉 𝒐𝒇 𝒕𝒉𝒆 𝒘𝒊𝒓𝒆 𝒖𝒔𝒆𝒅 𝒇𝒐𝒓 𝒔𝒒𝒖𝒂𝒓𝒆 = 𝟐𝟖 − 𝒙 = 𝟐𝟖 −
𝝅+𝟒
𝟐𝟖𝝅 + 𝟏𝟏𝟐 − 𝟐𝟖𝝅 𝟏𝟏𝟐
= =
𝝅+𝟒 𝝅+𝟒
3(a). Find the volume of the largest cone that can be inscribed in a sphere of
radius R.

3(b). Show that the height of the largest cone that can be inscribed in a sphere
of radius R is 4R /3.

3(c). Show that volume of the largest cone that can be inscribed in a sphere of
radius R is 8/27 volume of sphere.

SOL: 𝑮𝑰𝑽𝑬𝑵 𝒕𝒉𝒆 𝒓𝒂𝒅𝒊𝒖𝒔 𝒐𝒇 𝒕𝒉𝒆 𝑺𝒑𝒉𝒆𝒓𝒆 = 𝑹

𝑳𝒆𝒕 𝒓𝒂𝒅𝒊𝒖𝒔 𝒐𝒇 𝒕𝒉𝒆 𝑪𝒐𝒏𝒆 = 𝒓, 𝒉𝒆𝒊𝒈𝒉𝒕 = 𝒉 R

𝑹𝟐 = 𝒓𝟐 + (𝒉 − 𝑹)𝟐  𝒓𝟐 = 𝑹𝟐 − (𝒉 − 𝑹)𝟐 = 𝟐𝑹𝒉 − 𝒉𝟐


h-R
𝟏 𝝅
𝑽𝒐𝒍 𝒐𝒇 𝑪𝒐𝒏𝒆 = 𝑽 = 𝝅𝒓𝟐 𝒉 = (𝟐𝑹𝒉 − 𝒉𝟐 )𝒉
𝟑 𝟑
r
𝝅
= (𝟐𝑹𝒉𝟐 − 𝒉𝟑 )
𝟑
51

𝝅 𝝅 𝝅
𝑽′ = 𝟑 [(𝟒𝑹𝒉 − 𝟑𝒉𝟐 ) = 𝟑 [𝒉(𝟒𝑹 − 𝟑𝒉)] 𝑭𝒐𝒓 𝑴𝒂𝒙 𝒐𝒓 𝑴𝒊𝒏 𝑽′ = 𝟎, [𝒉(𝟒𝑹 − 𝟑𝒉)] = 𝟎
𝟑

𝟒𝑹
 𝒉 = 𝟎 𝑹𝒆𝒋𝒆𝒄𝒕𝒆𝒅 (𝒐𝒓) 𝟒𝑹 − 𝟑𝒉 = 𝟎 𝒉= 𝟑

𝝅 𝟒𝑹 𝟒𝑹
𝑽′′ = (𝟒𝑹 − 𝟔𝒉),  𝑽′′ ( ) = −𝒗𝒆  𝑽𝒐𝒍 𝒐𝒇 𝒄𝒐𝒏𝒆 𝑴𝒂𝒙 𝒂𝒕, 𝒉 =
𝟑 𝟑 𝟑

𝝅 𝝅 𝟒𝑹 𝟏𝟔𝑹𝟐 𝟒𝑹
𝑽𝒐𝒍 𝒐𝒇 𝒄𝒐𝒏𝒆 = 𝑽 = [𝟐𝒉𝑹 − 𝒉𝟐 ]𝒉 = [𝟐 𝑹− ]
𝟑 𝟑 𝟑 𝟗 𝟑

𝝅 𝟐𝟒𝑹𝟐 − 𝟏𝟔𝑹𝟐 𝟒𝑹 𝝅 𝟖𝑹𝟐 𝟒𝑹 𝟑𝟐𝝅𝑹𝟑


= ( ) = ( ) =
𝟑 𝟗 𝟑 𝟑 𝟗 𝟑 𝟖𝟏

𝟑𝟐𝝅𝑹𝟑
𝟒𝝅𝑹𝟑 𝑽𝒐𝒍 𝒐𝒇 𝑪𝒐𝒏𝒆 𝟖
𝑽𝒐𝒍𝒖𝒎𝒆 𝒐𝒇 𝒔𝒑𝒉𝒆𝒓𝒆 = , = 𝟖𝟏
𝟒𝝅𝑹𝟑
=
𝟑 𝑽𝒐𝒍 𝒐𝒇 𝒔𝒑𝒉𝒆𝒓𝒆 𝟐𝟕
𝟑

𝟖
𝑽𝒐𝒍 𝒐𝒇 𝑪𝒐𝒏𝒆 = 𝑽𝒐𝒍 𝒐𝒇 𝒔𝒑𝒉𝒆𝒓𝒆
𝟐𝟕

4. An open box with square base is to be made out of given quantity of a sheet
of area a2. Find the maximum volume of box. (Show that maximum volume of
the box is a 3 / 6√3)

SOL: 𝑮𝒊𝒗𝒆𝒏 𝒂𝒓𝒆𝒂 𝒐𝒇 𝒄𝒂𝒓𝒅𝒃𝒐𝒂𝒓𝒅 = 𝒂𝟐


𝑳𝒆𝒕 𝒍𝒆𝒏𝒈𝒕𝒉 𝒐𝒇 𝒕𝒉𝒆 𝒐𝒑𝒆𝒏 𝒃𝒐𝒙 𝒘𝒊𝒕𝒉 𝒔𝒒𝒖𝒂𝒓𝒆 𝒃𝒂𝒔𝒆 = 𝒙 𝑳𝒆𝒕 𝒉𝒆𝒊𝒈𝒉𝒕 = 𝒚.

𝒂𝟐 − 𝒙𝟐 𝒂𝟐 𝒙
𝑨𝒓𝒆𝒂 𝒐𝒇 𝒕𝒉𝒆 𝒄𝒂𝒓𝒅𝒃𝒐𝒂𝒓𝒅 = 𝒂𝟐 = 𝒙𝟐 + 𝟒𝒙𝒚, 𝒚 = = −
𝟒𝒙 𝟒𝒙 𝟒

𝒂𝟐 𝒙 𝒂𝟐 𝒙 𝒙𝟑
𝟐 𝟐
𝑽𝒐𝒍𝒖𝒎𝒆 𝒐𝒇 𝒕𝒉𝒆 𝒃𝒐𝒙 = 𝑽 = 𝒙 𝒚 = 𝒙 ( − )= −
𝟒𝒙 𝟒 𝟒 𝟒

𝒂𝟐 𝒙 𝒙𝟑 𝒂𝟐 𝟑𝒙𝟐
𝑽= − , 𝑽′ = − , 𝑭𝒐𝒓 𝑴𝒂𝒙 𝒐𝒓 𝑴𝒊𝒏 𝑽′ = 𝟎,
𝟒 𝟒 𝟒 𝟒

𝒂𝟐 𝟑𝒙𝟐 𝒂𝟐 𝟑𝒙𝟐 𝒂
− = 𝟎, = , 𝟑𝒙𝟐 = 𝒂𝟐 𝒙=
𝟒 𝟒 𝟒 𝟒 √𝟑
−𝟔𝒙 𝒂 𝒂
𝑽′′ = , 𝑽′′ ( ) = −𝒗𝒆, 𝑽𝒐𝒍 𝒊𝒔 𝑴𝒂𝒙 𝒂𝒕 𝒙 =
𝟒 √𝟑 √𝟑
𝟏 𝟐 𝟏 𝒂 𝒂 𝟑
𝑴𝒂𝒙 𝒗𝒐𝒍 𝒐𝒇 𝒕𝒉𝒆 𝒃𝒐𝒙 = 𝑽 = ( 𝒂 𝒙 − 𝒙𝟑 ) = ( 𝒂 𝟐 ( ) − ( ) )
𝟒 𝟒 √𝟑 √𝟑
52

𝟏 𝒂𝟑 𝒂𝟑 𝟏 𝟑𝒂𝟑 − 𝒂𝟑 𝟐𝒂𝟑 𝒂𝟑
𝑽= ( − )= ( )= =
𝟒 √ 𝟑 𝟑√ 𝟑 𝟒 𝟑√ 𝟑 𝟏𝟐√𝟑 𝟔√𝟑
5. Prove that semi-vertical angle of right circular cone of maximum
volume and of given slant height is tan –1 √2.

SOL: 𝑳𝒆𝒕 𝑹𝒂𝒅𝒊𝒖𝒔 𝒐𝒇 𝒕𝒉𝒆 𝑪𝒐𝒏𝒆 = 𝒓, 𝑯𝒆𝒊𝒈𝒉𝒕 = 𝒉, 𝒔𝒍𝒂𝒏𝒕 𝒉𝒆𝒊𝒈𝒉𝒕 = 𝒍,

𝟏 𝟐
𝒍𝟐 = 𝒓𝟐 + 𝒉𝟐  𝒓𝟐 = 𝒍𝟐 − 𝒉𝟐 , 𝑽𝒐𝒍 𝒐𝒇 𝒄𝒐𝒏𝒆 = 𝑽 = 𝝅𝒓 𝒉
𝟑
𝝅 𝟐 𝝅 𝝅 𝟐
𝑽= (𝒍 − 𝒉𝟐 )𝒉 = (𝒍𝟐 𝒉 − 𝒉𝟑 )  𝑽′ = (𝒍 − 𝟑𝒉𝟐 )
𝟑 𝟑 𝟑
𝝅 𝟐
𝑭𝒐𝒓𝑴𝒂𝒙 𝒐𝒓 𝑴𝒊𝒏 𝑽′ = 𝟎, (𝒍 − 𝟑𝒉𝟐 ) = 𝟎, 𝒍𝟐 = 𝟑𝒉𝟐 𝒍 = √𝟑𝒉
𝟑

𝝅
𝑽′′ = (−𝟔𝒉)  𝑽′′ (√𝟑𝒉) = −𝒗𝒆,
𝟑

𝑽𝒐𝒍 𝒐𝒇 𝒄𝒐𝒏𝒆 𝒊𝒔 𝒎𝒂𝒙 𝒊𝒇, 𝒍 = √𝟑𝒉, 𝒍𝟐 = 𝟑𝒉𝟐 𝒔𝒖𝒃 𝒊𝒏 𝒍𝟐 = 𝒓𝟐 + 𝒉𝟐

𝒓
𝟑𝒉𝟐 = 𝒓𝟐 + 𝒉𝟐  𝟐𝒉𝟐 = 𝒓𝟐  √𝟐 =
𝒉
 𝒕𝒂𝒏 = √𝟐   = 𝒕𝒂𝒏−𝟏 √𝟐

6. Show that height of cylinder of maximum volume that can be inscribed in a


sphere of R is equal to 2R /√3. Also find the maximum volume.

SOL: 𝑮𝑰𝑽𝑬𝑵 𝒕𝒉𝒆 𝒓𝒂𝒅𝒊𝒖𝒔 𝒐𝒇 𝒕𝒉𝒆 𝑺𝒑𝒉𝒆𝒓𝒆 = 𝑹

𝑳𝒆𝒕 𝒓𝒂𝒅𝒊𝒖𝒔 𝒐𝒇 𝒕𝒉𝒆 𝑪𝒚𝒍𝒊𝒏𝒅𝒆𝒓 = 𝒓, 𝒉𝒆𝒊𝒈𝒉𝒕 = 𝒉


𝟒𝑹𝟐 − 𝒉𝟐
𝟐
𝟒𝑹 = 𝟒𝒓 + 𝒉𝟐 𝟐
𝒓 =
𝟐
𝟒
𝟒𝑹𝟐 − 𝒉𝟐
𝟐
𝝅
𝑽𝒐𝒍 𝒐𝒇 𝑪𝒚𝒍𝒊𝒏𝒅𝒆𝒓 = 𝑽 = 𝝅𝒓 𝒉 = 𝝅 ( )𝒉 = (𝟒𝑹𝟐 𝒉 − 𝒉𝟑 )
𝟒 𝟒
53

𝝅
𝑽′ = (𝟒𝑹𝟐 − 𝟑𝒉𝟐 ) 𝑭𝒐𝒓𝑴𝒂𝒙 𝒐𝒓 𝑴𝒊𝒏 𝑽′ = 𝟎
𝟒

𝝅 𝟐𝑹
 (𝟒𝑹𝟐 − 𝟑𝒉𝟐 ) = 𝟎 𝟒𝑹𝟐 = 𝟑𝒉𝟐 𝒉= ,
𝟒 √𝟑
𝝅 𝟐𝑹 𝟐𝑹
𝑽′′ = (−𝟔𝒉)  𝑽′′ ( ) = −𝒗𝒆, 𝑽𝒐𝒍 𝒐𝒇 𝒄𝒚𝒍𝒊𝒏𝒅𝒆𝒓 𝒊𝒔 𝒎𝒂𝒙 𝒂𝒕 𝒉 =
𝟑 √𝟑 √𝟑
𝝅 𝟐
𝟐𝑹 𝟖𝑹𝟑 𝝅 𝟐𝟒𝑹𝟑 − 𝟖𝑹𝟑
𝑽𝒐𝒍 𝒐𝒇 𝒄𝒚𝒍𝒊𝒏𝒅𝒆𝒓 = (𝟒𝑹 ( ) − )= ( )
𝟒 √𝟑 𝟑√ 𝟑 𝟒 𝟑√ 𝟑

𝝅 𝟏𝟔𝑹𝟑 𝟒𝝅𝑹𝟑
= ( )=
𝟒 𝟑√𝟑 𝟑√𝟑
7. Show that height of a cylinder of given surface and greater volume is equal
to the diameter of the base (h = 2r).

SOL: Let radius of the CLOSED Cylinder = r, Height = h

𝑺−𝟐𝝅𝒓𝟐
𝑺𝒖𝒓𝒇𝒂𝒄𝒆 𝒂𝒓𝒆𝒂 = 𝑺 = 𝟐𝝅𝒓𝒉 + 𝟐𝝅𝒓𝟐  𝒉 = ….(1)
𝟐𝝅𝒓

𝑺 − 𝟐𝝅𝒓𝟐
𝟐 𝟐
𝑺𝒓
𝑽𝒐𝒍 𝒐𝒇 𝑪𝒚𝒍𝒊𝒏𝒅𝒆𝒓 = 𝑽 = 𝝅𝒓 𝒉 = 𝝅𝒓 ( )= − 𝝅𝒓𝟑
𝟐𝝅𝒓 𝟐

𝑺
𝑽′ = − 𝟑𝝅𝒓𝟐 , 𝑭𝒐𝒓 𝑴𝒂𝒙𝒐𝒓 𝑴𝒊𝒏𝑽′ = 𝟎
𝟐

𝑺 √𝑺
 𝟐
= 𝟑𝝅𝒓𝟐  𝑺 = 𝟔𝝅𝒓𝟐 𝒓=
√𝟔𝝅

√𝑺 √𝑺
𝑽′′ = −𝟔𝝅𝒓  𝑽′′ ( ) = −𝒗𝒆, 𝑽𝒐𝒍 𝒊𝒔 𝑴𝒂𝒙 𝒓 = 𝒐𝒓 𝑺 = 𝟔𝝅𝒓𝟐 sub (1)
√𝟔𝝅 √𝟔𝝅

𝑺 − 𝟐𝝅𝒓𝟐 𝟔𝝅𝒓𝟐 − 𝟐𝝅𝒓𝟐 𝟒𝝅𝒓𝟐


𝒉=  𝒉 = = = 𝟐𝒓  𝒉 = 𝟐𝒓
𝟐𝝅𝒓 𝟐𝝅𝒓 𝟐𝝅𝒓

8. A window in form of rectangle is surmounted by a semicircular opening. The


total perimeter is 20 cm. Find dimensions of window to emit maximum light
through the whole opening.

SOL: 𝑳𝒆𝒕 𝒓𝒂𝒅𝒊𝒖𝒔 𝒐𝒇 𝒕𝒉𝒆 𝒔𝒆𝒎𝒊 − 𝒄𝒊𝒓𝒄𝒍𝒆 = 𝒓, 𝑳𝒆𝒏𝒈𝒕𝒉 𝒐𝒇 𝒓𝒂𝒄𝒕𝒂𝒏𝒈𝒍𝒆 = 𝟐𝒓,


54

𝑩𝒓𝒆𝒂𝒅𝒕𝒉 = 𝒙, 𝑮𝒊𝒗𝒆𝒏 𝑷𝒆𝒓𝒊𝒎𝒊𝒕𝒆𝒓 𝒐𝒇 𝑾𝒊𝒏𝒅𝒐𝒘 = 𝟐𝟎 𝒄𝒎.


𝟐𝟎 − 𝟐𝒓 − 𝝅𝒓 𝟏
𝟐𝒓 + 𝟐𝒙 + 𝝅𝒓 = 𝟐𝟎 𝒙= = (𝟐𝟎 − 𝟐𝒓 − 𝝅𝒓)
𝟐 𝟐
𝝅𝒓𝟐 𝟐𝟎 − 𝟐𝒓 − 𝝅𝒓 𝝅𝒓𝟐
𝑨𝒓𝒆𝒂 𝒐𝒇 𝑾𝒊𝒏𝒅𝒐𝒘 = 𝑨 = (𝟐𝒓)𝒙 + = (𝟐𝒓) ( )+
𝟐 𝟐 𝟐
𝝅𝒓𝟐 𝝅𝒓𝟐
𝑨 = 𝟐𝟎𝒓 − 𝟐𝒓𝟐 − 𝝅𝒓𝟐 + 𝑨 = 𝟐𝟎𝒓 − 𝟐𝒓𝟐 −  𝑨′ = 𝟐𝟎 − 𝟒𝒓 − 𝝅𝒓
𝟐 𝟐
𝟐𝟎
𝑭𝒐𝒓 𝑴𝒂𝒙 𝒐𝒓 𝑴𝒊𝒏 𝑨′ = 𝟎  𝟐𝟎 − 𝒓(𝟒 + 𝝅) = 𝟎, 𝒓=
𝝅+𝟒
𝟐𝟎 𝟐𝟎
𝑨′′ = −𝟒 − 𝝅  𝑨′′ ( ) = −𝒗𝒆, 𝑨𝒓𝒆𝒂 𝒊𝒔 𝒎𝒂𝒙 𝒂𝒕 𝒓 =
𝝅+𝟒 𝝅+𝟒
𝟐𝟎 𝟒𝟎
𝑹𝒂𝒅𝒊𝒖𝒔 𝒐𝒇 𝒘𝒊𝒏𝒅𝒐𝒘 = 𝒓 = , 𝒍𝒆𝒏𝒈𝒕𝒉 = 𝟐𝒓 =
𝝅+𝟒 𝝅+𝟒
𝟏 𝟐𝟎 𝟐𝟎 𝟐𝟎
𝑩𝒓𝒆𝒂𝒅𝒕𝒉 = 𝒙 = (𝟐𝟎 − 𝟐 ( )−𝝅( )) =
𝟐 𝝅+𝟒 𝝅+𝟒 𝝅+𝟒

7 – INDEFINITE INTEGRALS

Integration is the inverse process of differentiation. In the differential calculus, we are given
a function and we have to find the derivative or differential of this function, but in the integral
calculus, we are to find a function whose differential is given. Thus, integration is a process
which is the inverse of differentiation.
Then, ∫f(x) dx = F(x) + C, these integrals are called indefinite integrals or general integrals. C
is an arbitrary constant by varying which one gets different anti-derivatives of the given
function.
Properties of Indefinite Integral
(i) ∫[f(x) + g(x)] dx = ∫f(x) dx + ∫g(x) dx
(ii) For any real number k, ∫k f(x) dx = k∫f(x)dx.
55

Basic Formulae
56

(a). Integration using Trigonometric Identities

1) 2 sin A . cos B = sin( A + B) + sin( A – B)

2) 2 cos A . sin B = sin( A + B) – sin( A – B)


3) 2 cos A . cos B = cos (A + B) + cos(A – B)
4) 2 sin A . sin B = cos(A – B) – cos (A + B)
5) 2 sin A cos A = sin 2A
6) cos2 A – sin2 A = cos 2A
7) sin2 A = (1 − cos2A)/2 sin2 A + cos2 A = 1
8) sin3A = 3sinA − 4sin3A cos3A = 4cos3A – 3cosA

(b). Integration by Substitutions


(c). Integration by Partial Fractions
A rational function is ratio of two polynomials of the form p(x)q(x), where p(x) and q(x) are
polynomials in x and q(x) ≠ 0. If degree of p(x) > degree of q(x), then we may divide p(x) by q(x) so
that p(x)q(x) = t(x) + p1(x) q(x), where t(x) is a polynomial in x which can be integrated easily and
degree of p1(x) is less than the degree of q(x) . p1(x)q(x) can be integrated by expressing p1(x)q(x) as
the sum of partial fractions of the following type:

where x2 + bx + c cannot be factorised further.

(d). Integration by Parts


For a given functions f(x) and q(x), we have
∫[f(x) q(x)] dx = f(x)∫g(x)dx – ∫{f'(x) ∫g(x)dx} dx
Here, we can choose the first function according to its position in ILATE, where
I = Inverse trigonometric function etc..

Note: (i) Keep in mind, ILATE is not a rule as all questions of integration by parts cannot be done by
above method.

Integrals of the types can be transformed into standard


𝑑
form by expressing px + q = A 𝑑𝑥 (ax2 + bx + c) + B = A (2ax + b) + B, where A and B are
determined by comparing coefficients on both sides.
57

MCQ (1 Mark Questions)

cos(log 𝑥)
1) ∫ 𝑑𝑥=
𝑥
a) cos(log 𝑥) + c b) sin(log 𝑥) + c c) −cos(log 𝑥) + c d) −sin(log 𝑥) + c
cos 𝑥 − sin 𝑥
2) ∫ 𝑑𝑥 = a) √sin 𝑥 + cos 𝑥 + c b) 2√sin 𝑥 + cos 𝑥 + c
√sin 𝑥 + cos 𝑥
c) - √sin 𝑥 + cos 𝑥 + c d) - 2√sin 𝑥 + cos 𝑥 + c

1 1 5𝑥 5𝑥
3) ∫ 𝑑𝑥 = a) sin−1 ( ) + c b) sin−1 ( ) + c
√9−25 𝑥2 5 3 3
1 5𝑥 5𝑥
c) − sin−1 ( ) + c d) − sin−1 ( ) + c
5 3 3

4) ∫ log 𝑥 𝑑𝑥= a) log x + c b) 1/x + c

c) x(log x + 1) + c d) x(log x - 1) + c
𝑥−1 𝑒𝑥 𝑒𝑥
5) ∫ 𝑒𝑥 ( 2 ) 𝑑𝑥 = a) 𝑥𝑒 𝑥 + c b) – 𝑥𝑒 𝑥 + c c) +c d) − +c
𝑥 𝑥 𝑥

1 1 𝑥+1 1 𝑥+1
6) ∫ 𝑑𝑥 = a) − tan−1 ( )+𝑐 b) tan−1 ( )+𝑐
𝑥 2 +2𝑥+2 2 2 2 2
c) − tan−1 (𝑥 + 1) + 𝑐 d) tan−1 (𝑥 + 1) + 𝑐
1
7) To solve the ∫ 𝑑𝑥, the substitution to be done is put t =
𝑥.log 𝑥√.log(log 𝑥)

a) log x b) log(log x) c) xlog x d) x


𝑥𝑒𝑥 (𝑥+2)
8) ∫ 𝑑𝑥 = a) 𝑡𝑎𝑛(𝑥 2 𝑒 𝑥 ) + 𝑐 b) −𝑡𝑎𝑛(𝑥 2 𝑒 𝑥 ) + 𝑐
𝑠𝑖𝑛2 (𝑥2 𝑒𝑥 )

c) 𝑐𝑜𝑡(𝑥 2 𝑒 𝑥 ) + 𝑐 d) −𝑐𝑜𝑡(𝑥 2 𝑒 𝑥 ) + 𝑐

9) To solve the ∫ 𝑒𝑥 sin(2𝑥) 𝑑𝑥, the first function to be taken is


a) 𝑒 𝑥 b) sin(2𝑥) c) any one can be taken d) data incorrect.
1
10) To solve the ∫ 𝑑𝑥, the substitution to be done is put t =
𝑥 − √𝑥

a) √𝑥 b) 𝑥 c) 𝑥 − √𝑥 d) None of these
𝜋
𝜋 𝜋
11) ∫
0
2 𝑐𝑜𝑠 2 𝑥 𝑑𝑥 = a)
2
b)
4
c)  d) 0

𝜋/3 𝜋 𝜋 𝜋
12) ∫
𝜋/6
𝑠𝑖𝑛2 𝑥 𝑑𝑥 = a)
12
b)
6
c)
3
d) 0
58

𝜋/2 𝑐𝑜𝑠 2023 𝑥 𝜋 𝜋


13) ∫0 𝑑𝑥 a) b) c)  d) 0
𝑐𝑜𝑠 2023 𝑥 + 𝑠𝑖𝑛2023 𝑥 2 4
𝜋/3 1 𝜋 𝜋 𝜋
14) ∫𝜋/6 𝑑𝑥 a) b) c) d) 0
1+√tan𝑥 12 6 3

6 √x 3
15) ∫0 𝑑𝑥 a) b) 3 c) 2 d) 0
√x + √6 − x 2

𝜋 4 + 5 sin 𝑥
16) ∫−𝜋 log | | 𝑑𝑥 a) 1 b) 3 c) 2 d) 0
4 − 5 sin 𝑥

𝜋/2 sin 𝑥 − cos 𝑥 𝜋 𝜋


17) ∫0 𝑑𝑥 a) b) c)  d) 0
cos 𝑥 + sin 𝑥 2 4
𝝅
𝟒 𝒅𝒙
18) ∫ 𝝅 𝟏+𝐜𝐨𝐬 𝟐𝒙
is equal to a) 1 (b) 2 (c) 3 (d) 4

𝟒
8
19) ∫−8|𝑥 | 𝑑𝑥 = a) 8 b) 4 c) 64 d) 32

2
20) The value of ∫−2( 𝑥 3 + 1 ) dx is (a) 2 (b) 3 (c) 4 (d) 5

2 or 3 marks questions
Evaluate the following integrals w.r.t. x.

1 1 1
1) 2) 3) 4) √9 + 16𝑥 2
9+ 16𝑥 2 √16 − 25𝑥 2 √4𝑥 2 − 9

1 1 (𝑥 + 1)(𝑥 + log 𝑥) 3
5) √7 − 6𝑥 − 𝑥 2 6) 7) 8)
𝑥2 + 4𝑥 − 5 √5 − 4𝑥 − 𝑥2 𝑥

sin 𝑥 + cos 𝑥 1 𝑒 𝑥 (𝑥+1)


9) 10) 11) 12) 𝑥 2 . 𝑐𝑜𝑠𝑥
sin 𝑥 − cos 𝑥 𝑥.log 𝑥.log(log 𝑥) 𝑐𝑜𝑠 2 (𝑥𝑒 𝑥 )

𝑥− 1
13) 𝑥 3 . 𝑒 𝑥 14) 𝑥. tan−1 𝑥 15) cos −1 𝑥 16) 𝑒 𝑥 [(1 + 𝑥)3
]

2 + sin 2𝑥 𝑥 𝑥3
17) 𝑒 𝑥 [ 1 + ] 18) (𝑥 − 19) 𝑥 2+
cos 2𝑥 1)(𝑥 − 2)(𝑥 − 3) 4𝑥 + 4

𝑥
20) (𝑥+1)2(𝑥+2)
59

INTEGRATION AND APPLICATION


𝒙𝟐
1. (a). Evaluate:∫ 𝒅𝒙 1 (b). Evaluate:∫ 𝒔𝒊𝒏𝟒𝒙 𝒄𝒐𝒔𝟕𝒙 𝒅𝒙
𝟏 + 𝒙𝟑
𝟏 𝟐𝒙
2. (a). Evaluate:∫𝟎 𝒅𝒙 2 .(b). Evaluate:∫ 𝒆𝒙 [𝒕𝒂𝒏𝒙 + 𝒍𝒐𝒈(𝒔𝒆𝒄𝒙)] 𝒅𝒙
𝟏+𝒙𝟐
𝟓 𝒅𝒙 𝒔𝒊𝒏√𝒙
3. (a). Evaluate: ∫ 3. (b). Evaluate:∫ 𝒅𝒙
√𝒙+𝟑 − √𝒙−𝟐 √𝒙
𝟏 𝒙𝟑 − 𝒙𝟐 + 𝒙 − 𝟏
4. (a). Evaluate: ∫ 𝒅𝒙 4. (b). Evaluate:∫ 𝒅𝒙
𝟏 + 𝒕𝒂𝒏𝒙 𝒙−𝟏
𝝅
𝒆𝟒𝒙 𝟑𝒙𝟐 𝒔𝒊𝒏𝒙
5. (a). If ∫(𝒆𝒂𝒙 + 𝒃𝒙)𝒅𝒙 = + , 𝑓𝑖𝑛𝑑 𝑎, 𝑏. 5.(b). Evaluate: ∫𝟎𝟐 𝒅𝒙
𝟒 𝟐 𝟏+𝒄𝒐𝒔𝟐 𝒙
𝒅𝒙
6. (a). Evaluate: ∫ 𝒙𝒍𝒐𝒈(𝟏 + 𝒙)𝒅𝒙 6. (b). Evaluate: ∫ 𝒅𝒙
√𝟑−𝒙+𝒙𝟐
𝟏 𝟏
7. (a). Evaluate: ∫ 𝒅𝒙 7. (b). Evaluate: ∫ 𝒅𝒙
𝟏 +𝒙𝟒 𝒙 𝟑+ 𝒙 𝟐+ 𝒙 + 𝟏
𝟏 𝒙
8. (a). Evaluate: ∫ 𝒅𝒙 8. (b). Evaluate: ∫ (𝒙−𝟏)𝟐(𝒙+𝟐) 𝒅𝒙
𝒄𝒐𝒔(𝒙−𝒂)𝒄𝒐𝒔(𝒙−𝒃)
𝝅 𝜋
√𝒄𝒐𝒕𝒙
9. (a). Evaluate:∫𝟎𝟐 𝒅𝒙 9. (b). Evaluate: ∫0 log (1 + 𝑡𝑎𝑛𝑥) 𝑑𝑥
4
√ 𝒄𝒐𝒕𝒙 +√𝒕𝒂𝒏𝒙
𝝅 𝝅
𝒙
10. (a). Evaluate: ∫ 𝟐
−𝝅 𝒙𝟑 𝒄𝒐𝒔𝟐𝒙 𝒅𝒙 10. (b). Evaluate:∫𝟎𝟐 𝒅𝒙
𝒔𝒊𝒏𝒙 + 𝒄𝒐𝒔𝒙
𝟐
𝝅
𝒙 𝟐 +𝟏
11. (a).Evaluate:∫ 𝑑𝑥 11. (b). Evaluate:∫𝟎 𝐥𝐨𝐠 (𝒔𝒊𝒏𝒙) 𝒅𝒙
𝟐
𝒙 𝟒 + 𝒙 𝟐 +𝟏

12. (a). Evaluate:∫ 𝒔𝒊𝒏𝒙. 𝒔𝒊𝒏𝟐𝒙. 𝒔𝒊𝒏𝟑𝒙 𝒅𝒙 12. (b). Evaluate:∫ 𝒔𝒊𝒏𝟒 𝒙 𝒅𝒙
𝝅
𝒔𝒊𝒏𝒙 𝒄𝒐𝒔𝒙
13. (a). Evaluate: Evaluate: ∫𝟎 𝟒 𝒅𝒙 13. (b). Evaluate :∫ √𝒕𝒂𝒏𝒙 𝒅𝒙
𝒄𝒐𝒔 𝟒 𝒙 + 𝒔𝒊𝒏 𝟒 𝒙
𝝅 𝟑
14. (a). Evaluate:∫𝟎 |𝒄𝒐𝒔𝒙| 𝒅𝒙 14.(b). Evaluate: ∫𝟎 (|𝒙 − 𝟏| + |𝒙 − 𝟐|) 𝒅𝒙
𝑎 𝑎 𝟐
15. Prove that:∫0 𝑓(𝑥)𝑑𝑥 = ∫0 𝑓(𝑎 − 𝑥)𝑑𝑥. Use it to evaluate: ∫𝟎 𝒙√𝟐 − 𝒙 𝒅𝒙
𝒔𝒊𝒏 𝟖 𝒙 − 𝒄𝒐𝒔 𝟖 𝒙 𝟔𝒙 + 𝟕
16. (a). Evaluate:∫ 𝑑𝑥 16.(b). Evaluate:∫ 𝑑𝑥
𝟏−𝟐 𝒔𝒊𝒏 𝟐 𝒙 𝒄𝒐𝒔 𝟐 𝒙 √( 𝒙 − 𝟓 )( 𝒙 − 𝟒 )

𝟏 𝟐𝒙
17. (a). Evaluate: ∫ 𝒙 𝒔𝒊𝒏−𝟏 𝒙 𝒅𝒙 17.(b). Evaluate: ∫𝟎 𝒔𝒊𝒏−𝟏 ( ) 𝒅𝒙
𝟏 + 𝒙𝟐
𝟑 𝟒
18. (a). Evaluate as limit of a sum:(a).∫𝟎 (𝟐𝒙𝟐 + 𝒙 + 𝟓)𝒅𝒙 18.(b).∫𝟏 (𝒙𝟐 + 𝒙)𝒅𝒙
𝝅
𝟏 𝐥𝐨𝐠 (𝟏+𝒙) 𝒙
19. (a). Evaluate:∫𝟎 𝒅𝒙 19. (b). Evaluate:∫𝟎 𝟐 𝒅𝒙
𝟏 + 𝒙𝟐 𝟏 + 𝒔𝒊𝒏𝒙 + 𝒄𝒐𝒔𝒙
60

𝝅
𝝅 𝒙 𝒕𝒂𝒏𝒙
20. (a). Evaluate:∫𝟎𝟐(√𝒕𝒂𝒏𝒙 + √𝒄𝒐𝒕𝒙) 𝒅𝒙 20.(b). Evaluate:∫𝟎 𝒅𝒙
𝒔𝒆𝒄𝒙 + 𝒕𝒂𝒏𝒙
𝟏
21. (a). Evaluate: ∫ √𝒙𝟐 + 𝟒𝒙 + 𝟏 𝒅𝒙 21 (b). Evaluate: ∫𝟎 𝒙(𝟏 − 𝒙)𝒏 𝒅𝒙
𝟏 𝟏
22. (a). Evaluate: ∫ 𝑑𝑥 22. (b). Evaluate: ∫ 𝑑𝑥
𝒙 𝟒 −𝟏 𝟏− 𝒙 𝟑
𝟏 𝟐𝒙
23. (a). Evaluate: ∫ 𝑑𝑥 23. (b). Evaluate: ∫ 𝒆 𝒄𝒐𝒔𝟑𝒙 𝒅𝒙
𝒙( 𝒙 𝒏 + 𝟏)
𝟏
24. (a). Evaluate: ∫ 𝑑𝑥 24. (b). Evaluate: ∫(𝒔𝒊𝒏−𝟏 𝒙)𝟐 𝒅𝒙
𝒙 (𝟏 +𝒍𝒐𝒈𝒙)
𝟏 √𝟏−√𝒙
25. (a). Evaluate: ∫
√𝒔𝒊𝒏 𝟑 𝒙 𝐬𝐢𝐧 ( 𝒙 + 𝜶)
𝒅𝒙 25. (b).Evaluate: ∫ 𝑑𝑥
√𝟏+√𝒙
𝝅
26. (a).Evaluate:∫ 𝒆𝟐𝒙 𝑺𝒊𝒏𝟑𝒙 𝒅𝒙 26.(b).Evaluate:∫𝟎𝟐(𝟐 𝒍𝒐𝒈𝒔𝒊𝒏𝒙 − 𝒍𝒐𝒈𝒔𝒊𝒏𝟐𝒙) 𝒅𝒙
𝜋 𝝅
𝒄𝒐𝒔 𝟐 𝒙 𝒔𝒊𝒏𝒙 + 𝒄𝒐𝒔𝒙
27. (a). Evaluate: ∫02 𝑑𝑥 27. (b). Evaluate:∫𝟎𝟒 𝒅𝒙
𝒄𝒐𝒔 𝟐 𝒙 + 𝟒 𝒔𝒊𝒏 𝟐 𝒙 𝟗 + 𝟏𝟔 𝒔𝒊𝒏𝟐𝒙

28. (a). Evaluate: ∫ 𝒔𝒊𝒏−𝟏 𝒙 𝒅𝒙 28. (b). Evaluate: ∫ 𝒙 𝒕𝒂𝒏−𝟏 𝒙 𝒅𝒙


( 𝟐𝒙 − 𝟑 ) 𝟓𝒙 + 𝟑
29. (a). Evaluate: ∫ (𝒙 𝟐 𝒅𝒙 29. (b). Evaluate: ∫ 𝑑𝑥
−𝟏 )( 𝟐𝒙 + 𝟑 ) √𝒙 𝟐 + 𝟒𝒙 + 𝟏𝟎

30. Find area bounded between curves y 2 = 4x and x 2 = 4y


31. Find area bounded between y 2 = 4x and x = 1 and x = 3
32. Find the area of the region: {( x, y ) : y2 ≤ 4x , 4x2 + 4y2 ≤ 9 }
𝒙𝟐 𝒚𝟐
33. Find the area of the smaller region bounded by the ellipse 𝟐
+ = 𝟏 and the
𝒂 𝒃𝟐
𝒙 𝒚
line + = 𝟏
𝒂 𝒃

34. Using integration find the area of the region given by:
{(x, y) : 0 ≤ y ≤ x2 , 0 ≤ y ≤ x+1, 0 ≤ x ≤2}
35. Using integration, find the area of the triangular region whose vertices are
(a). (1, 0), (2, 2) and (3, 1) (b). A (2, 0), B (4, 5) and C (6, 3)
36. Using integration, find the area bounded by the lines:
(a). x + 2y = 2, y – x =1 and 2x + y = 7. (b). y =2x + 1, y = 3x + 1 and x = 4
37. Find the area of the region bounded by the parabola y = x 2 and 𝑦 = |x| .
38. Find the area bounded by curves (x – 1)2 + y2 = 1 and x2 + y2 = 1
39. Find the area of the smaller region bounded by x2 +y2 = 1 and x + y = 1
1
40. Sketch the graph of: y = |x + 3| and evaluate ∫−4|x + 3| dx.
61

CHAPTER 8 - APPLICATIONS OF INTEGRALS


The circle with centre at (0,0) and radius a

𝒙𝟐 + 𝒚𝟐 = 𝒂𝟐

S.N Figure About the figure Equation

2 Parabola: Vertex:(0, 0) Y2 = 4ax


Axis of symmetry: x axis
Directrix: x =-a
Focus: (a, 0)
Latus rectum: 4a

3 Parabola: Vertex:(0,0) Y2 = - 4ax


Axis of symmetry: x axis
Directrix: x = a
Focus: (-a, 0)
Latus rectum: 4a

PARABOLA
4 Parabola: Vertex:(0, 0) X2 = 4ay

Axis of symmetry: y axis

Directrix: y = - a

Focus: (0, a)

Latus rectum: 4a

5 Parabola: Vertex:(0, 0) X2 = - 4ay

Axis of symmetry: y axis

Directrix: y = a

Focus: (0, - a)

Latus rectum: 4a

.
62

PROPERTIES OF ELLIPSE

S.N Figure About the figure Equation

6 Ellipse:Vertex:(0, 0) 𝑥2 𝑦2
+ =1
𝑎2 𝑏 2
Major axis: x axis
Here a2 > b2
2
Directrix: x = a /c

Focus: (±c, 0)

Latusrectum: 2b2/a

Eccentricity: e = c/a

7. Ellipse:Vertex:(0, 0) 𝑥2 𝑦2
+ =1
𝑏 2 𝑎2
Major axis: y axis
Here a2 < b2
2
Directrix: y = a /c

Focus: (0, ± c)

Latus rectum: 2b2/a

Eccentricity: e = c/a

LINE

8 Straight line: 𝑦2 − 𝑦1
𝑦2 − 𝑦1
Passing through the points(x1,y1) = (𝑥
𝑥2 − 𝑥1 2
and (x2,y2)
− 𝑥1 )
𝑦2 − 𝑦1
Slope of line:
𝑥2 −𝑥1

Some special figure and their equation:


1 𝑎2 𝑥
Some important formulae: ∫ √𝑎2 − 𝑥 2 𝑑𝑥 = 2 𝑥√𝑎2 − 𝑥 2 + 𝑠𝑖𝑛−1 𝑎
2

Sketching of some important function or equations:

1. 𝑎𝑥 + 𝑏𝑦 = 𝑐
2. y = x2
3. y = |x| etc.
63

Area as a Definite Integral:

(a) Let f(x) be a continuous function defined on [a, b]. then, the area bounded by the curve y=f(x),
𝑏 𝑏
the x-axis and the ordinates x = a and x = b is given by ∫𝑎 |𝑓(𝑥)|𝑑𝑥 𝑂𝑟 ∫𝑎 |𝑦 |𝑑𝑥

Remark: we take absolute value if curve lie below the x axis.


(b) Let f(y) be a continuous function defined on [a,b]. then, the area bounded by the curve x=f(y),
𝑏
the y-axis and the ordinates y=a and y=b is given by ∫𝑎 |𝑓(𝑦)|𝑑𝑦 𝑂𝑟 ∫ |𝑥| 𝑑𝑦

(c) Remark: we take absoulute value if curve lie left of y axis.

(d) The area of the region between the curve and the X-axis, we split the figure into two parts,

Here, the value of the function is positive above the X-axis and negative below the X-axis.
Total area under the curve = A1 + |A2|
𝑏
: 𝐴 = ∫𝑎 [𝑓(𝑥) − 𝑔(𝑥)]𝑑𝑥

Find the values of x, say x=a and x=b within which the approximating rectangle can move
𝑏 𝑏
horizontally in the given region and form the integral∫𝑎 |𝑓(𝑥)|𝑑𝑥 𝑂𝑟 ∫𝑎 |𝑦 |𝑑𝑥
64

5 Marks questions
1) Using integrals find d the area enclosed by the parabola 3x 2 = 4y and the line 2y = 3x + 12

2) Using integrals find the area lying in the first quadrant and bounded by the circle x2 + y2 = 4 and
the lines x = 0 and x = 2

3) Using integrals find the area of the region enclosed by x2 + y2 ≤ 8, x ≥ √2 and x ≤ 2.

4) Using integrals find the area of ellipse 4𝑥 2 + 16𝑦 2 = 64.

5) Using integrals find the area of region bounded by parabola 𝑥 2 = 𝑦 and the curve 𝑦 = |𝑥|.

6) Using integrals find the area of the smaller part of the circle 𝒙𝟐 + 𝒚𝟐 = 𝒂𝟐 cut off by the
𝒂
line x =
√𝟐

7) Using integration, find the area of the region: {(𝑥, 𝑦): 0 ≤ 2𝑦 ≤ 𝑥 2 , 0 ≤ 𝑦 ≤ 𝑥 ,0 ≤ 𝑥 ≤ 3}


9 – DIFFERENTIAL EQUATIONS

Basic Concepts
Definition : An equation involving derivative (derivatives) of the dependent variable with respect to
independent variable (variables) is called a differential equation.

Order of a differential equation:


Order of a differential equation is defined as the order of the highest order derivative of the dependent
variable with respect to the independent variable involved in the given differential equation.

𝟒
𝒅𝟑 𝒚 𝒅𝟐 𝒚
Consider the following differential equations: ( ) + 𝒙𝟐 ( ) =𝟓
𝒅𝒙𝟑 𝒅𝒙𝟐

The equation involves the highest derivative of third order. Therefore, the order of this differential equation
is 3.

Degree of a differential equation:


The degree of a differential equation is defined only when a differential equation is polynomial equation in
derivatives, i.e., y′, y″, y″′ etc.

By the degree of a differential equation, when it is a polynomial equation in derivatives, we mean the
highest power (positive integral index) of the highest order derivative involved in the given differential
equation.

𝟐
𝒅𝟑 𝒚 𝒅𝟐 𝒚 𝒅𝒚
Consider the following differential equation: 𝒅𝒙𝟑
+ 𝟐 (𝒅𝒙𝟐 ) − 𝒅𝒙 + 𝒚 = 𝟎

We observe that equation is a polynomial equation in y″′, y″ and y′. Degree of such differential equations can
be defined. In view of the above definition, one may observe that above differential equation is of degree
one.
65

𝒅𝒚 𝟐 𝒅𝒚
Consider the following differential equation: (𝒅𝒙) + 𝒅𝒙
− 𝐬𝐢𝐧𝟐 𝒚 = 𝟎

We observe that above equation is a polynomial equation in y′ (not a polynomial in y though). Degree of such
differential equations can be defined. One may observe that above differential equation is of degree two.

𝒅𝒚 𝒅𝒚
Consider the following differential equation: + 𝐬𝐢𝐧 ( ) = 𝟎
𝒅𝒙 𝒅𝒙

We observe that the above equation is not a polynomial equation in y′ and degree of such a differential
equation cannot be defined

• Order and degree (if defined) of a differential equation are always positive integers.

General and Particular Solutions of a Differential Equation


𝒅𝟐 𝒚
consider the differential equation +𝒚=𝟎
𝒅𝒙𝟐

The solution of this differential equation is a function φ that will satisfy it i.e., when the function φ is
substituted for the unknown y (dependent variable) in the given differential equation, L.H.S. becomes equal
to R.H.S.

General solution : The solution which contains arbitrary constants is called the general solution (primitive)
of the differential equation.

Particular solution : The solution free from arbitrary constants i.e., the solution obtained from the general
solution by giving particular values to the arbitrary constants is called a particular solution of the differential
equation

Methods of Solving First Order, First Degree Differential Equations

(A) Differential equations with variables separable


𝒅𝒚
A first order-first degree differential equation is of the form : = 𝐹(𝑥, 𝑦)
𝒅𝒙

𝒅𝒚
= 𝑔 (𝑥). ℎ(𝑦)
𝒅𝒙
1
If ℎ (𝑦) ≠ 0, separating the variables, can be rewritten as 𝑑𝑦 = 𝑔(𝑥) 𝑑𝑥
ℎ(𝑦)
1
Integrating both sides of, we get ∫ ℎ(𝑦) 𝑑𝑦 = ∫ 𝑔(𝑥) 𝑑𝑥

Thus, above provides the solutions of given differential equation in the form 𝐻(𝑦) = 𝐺(𝑥) + 𝐶

1
Here, 𝐻 (𝑦) and 𝐺 (𝑥) are the anti derivatives of and 𝑔 (𝑥) respectively and C is the arbitrary constant.
ℎ(𝑦)
66

(B) Homogeneous differential equations


Homogeneous function: A function 𝐹(𝑥, 𝑦) is said to be a homogeneous function of degree 𝑛 if
𝐹(𝜆𝑥, 𝜆𝑦) = 𝜆𝑛 𝐹(𝑥, 𝑦) for any nonzero constant 𝜆.

𝒅𝒚
A differential equation of the form 𝒅𝒙 = 𝑭 (𝒙, 𝒚) is said to be homogenous if 𝑭(𝒙, 𝒚) is a homogenous
function of degree zero.

𝑑𝑦 𝑦
To solve a homogeneous differential equation of the type = 𝐹 (𝑥, 𝑦) = 𝑔 ( )
𝑑𝑥 𝑥

𝑑𝑦 𝑑𝑣
We make the substitution 𝑦 = 𝑣 . 𝑥 and 𝑑𝑥
= 𝑣 + 𝑥 𝑑𝑥

𝑑𝑥 𝑥
If the homogeneous differential equation is in the form 𝑑𝑦
= 𝐹 (𝑥, 𝑦) = 𝑔 (𝑦)

𝑑𝑥 𝑑𝑣
We make the substitution 𝑥 = 𝑣 . 𝑦 and 𝑑𝑦
= 𝑣 + 𝑦 𝑑𝑦

(C) Linear differential equations


𝑑𝑦
A differential equation of the from 𝑑𝑥
+𝑃𝑦 =𝑄

where, P and Q are constants or functions of 𝑥 only, is known as a first order linear differential equation.

𝑑𝑥
Another form of first order linear differential equation is +𝑃𝑥 = 𝑄
𝑑𝑦

where, P and Q are constants or functions of 𝑦 only, is known as a first order linear differential equation.

Steps involved to solve first order linear differential equation: 𝒅𝒚


𝒅𝒙
+𝑷𝒚=𝑸

where, P and Q are constants or functions of x only, is known as a first order linear differential equation.

𝑑𝑦
(i) Write the given differential equation in the form + 𝑃 𝑦 = 𝑄 , where P, Q are constants or
𝑑𝑥
functions of x only.
(ii) Find the Integrating Factor (I.F) = 𝑒 ∫ 𝑝𝑑𝑥
(iii) Write the solution of the given differential equation as 𝑦 (𝐼. 𝐹) = ∫ Q (I. F) 𝑑𝑥 + 𝐶

𝒅𝒙
Steps involved to solve first order linear differential equation: 𝒅𝒚
+𝑷𝒙= 𝑸

where, P and Q are constants or functions of y only, is known as a first order linear differential equation.

𝒅𝒙
(iv) Write the given differential equation in the form + 𝑷 𝒙 = 𝑸 , where P, Q are constants or
𝒅𝒚
functions of y only. Find the Integrating Factor (I.F) = 𝑒 ∫ 𝑝𝑑𝑦
(v) Write the solution of the given differential equation as 𝑥 (𝐼. 𝐹) = ∫ Q (I. F) 𝑑𝑦 + 𝐶
67

MCQ ON DIFFERENTIAL EQUATIONS:


𝟒
𝒅𝟑 𝒚 𝒅𝟐 𝒚
1. The order of the differential equation (𝒅𝒙𝟑 ) + 𝒙𝟐 (𝒅𝒙𝟐 ) = 𝟓 is
(A) 1 (B) 2 (C) 3 (D) 4
𝒅𝒚 𝟐
2. If the order and degree of the differential equation 𝒙 (𝒅𝒙) + 𝒅𝒚 + 𝟗 = 𝒚𝟐 are m and n
( )
𝒅𝒙
respectively , then the value of (𝒎𝟑 − 𝒏) is:
(A) 1 (B) 0 (C) −𝟏 (D) −𝟐
𝒅𝒚 𝟐 𝒅𝒚
3. The order of the differential equation (𝒅𝒙) + − 𝐬𝐢𝐧𝟐 𝒚 = 𝟎 is
𝒅𝒙
(A) 1 (B) 2 (C) 3 (D) not defined
𝒅𝒚 𝟐 𝒅𝒚
4. If the order and degree of the differential equation (𝒅𝒙) + − 𝐬𝐢𝐧𝟐 𝒚 = 𝟎 are 𝒎 and
𝒅𝒙
𝒏 respectively , then the value of (𝒎𝟐 + 𝒏) is:
(A) 1 (B) 2 (C) 3 (D) not defined
𝒅𝒚 𝒅𝒚
5. The order of the differential equation 𝒅𝒙 − 𝐬𝐢𝐧 (𝒅𝒙) = 𝟎 is
(A) 1 (B) 2 (C) 3 (D) not defined
𝒅𝒚 𝒅𝒚
6. The degree of the differential equation − 𝐬𝐢𝐧 (𝒅𝒙) = 𝟎 is
𝒅𝒙
(A) 1 (B) 2 (C) 3 (D) not defined
𝒅𝒚
𝒅𝟐 𝒚
7. The degree of the differential equation +𝒆 𝒅𝒙 = 𝟎 is
𝒅𝒙𝟐
(A) 1 (B) 2 (C) 3 (D) not defined
𝟑
𝒅𝒚 𝟐 𝟐 𝒅𝟐 𝒚
8. If the order and degree of the differential equation [𝟏 + (𝒅𝒙) ] = 𝒅𝒙𝟐 are 𝒎 and
𝒏 respectively , then the value of (𝒎𝟐 − 𝟐𝒏) is:
(A) 0 (B) 2 (C) 3 (D) not defined
𝒅𝟐 𝒚
9. If the order and degree of the differential equation 𝐥𝐨𝐠 𝒆 (𝟏 + 𝒅𝒙𝟐 ) = 𝒙 are 𝒎 and
𝒏 respectively , then the value of (𝒎𝟐 − 𝟐𝒏) is:
(A) 0 (B) 2 (C) 3 (D) not defined
𝒅 𝒅𝒚 𝟓
10. The order of the differential equation {(𝒅𝒙) } = 𝒙 is:
𝒅𝒙
(A) 0 (B) 2 (C) 3 (D) not defined

11. The number of arbitrary constants in the general solution of a differential equation of fourth order
are
(A) 0 (B) 2 (C) 3 (D) 4
12. The number of arbitrary constants in the particular solution of a differential equation of third
order are:
(A) 3 (B) 2 (C) 1 (D) 0
𝒅𝒚 𝒙+𝒚
13. The general solution of the differential equation 𝒅𝒙 = 𝒆 is
(A) 𝑒 𝑥 + 𝑒 −𝑦 = C (B) 𝑒 𝑥 + 𝑒 𝑦 = C (C) 𝑒 −𝑥 + 𝑒 𝑦 = C (D) 𝑒 −𝑥 + 𝑒 −𝑦 = C
𝑑𝑥 𝑥
14. A homogeneous differential equation of the from 𝑑𝑦 = ℎ (𝑦) can be solved by making the
substitution.
(A) 𝑦 = 𝑣𝑥 (B) 𝑣 = 𝑦𝑥 (C) 𝑥 = 𝑣𝑦 (D) 𝑥 = 𝑣
68

15. Which of the following is a homogeneous differential equation?


(A) (4𝑥 + 6𝑦 + 5) 𝑑𝑦 – (3𝑦 + 2𝑥 + 4) 𝑑𝑥 = 0 (B) (𝑥𝑦) 𝑑𝑥 – (𝑥 3 + 𝑦 3 ) 𝑑𝑦 = 0
(C) (𝑥 3 + 2𝑦 2 ) 𝑑𝑥 + 2𝑥𝑦 𝑑𝑦 = 0 (D) 𝑦 2 𝑑𝑥 + (𝑥 2 – 𝑥𝑦 – 𝑦 2 ) 𝑑𝑦 = 0
𝒅𝒚
16. The Integrating Factor of the differential equation 𝑥 𝒅𝒙 − 𝒚 = 2𝑥 2 is
1
(A) 𝑒 −𝑥 (B) 𝑒 −𝑦 (C) 𝑥 (D) 𝑥
𝑑𝑥
17. The Integrating Factor of the differential equation (1 − 𝑦 2 ) 𝑑𝑦 + 𝑦𝑥 = 𝑎𝑦, (−1 < 𝑦 < 1) is
1 1 1 1
(A) 𝑦2 −1 (B) (C) 1−𝑦2 (D)
√𝑦 2 −1 √1−𝑦 2

y dx− x dy
18. The general solution of the differential equation = 0 is
𝑦
(A) 𝑥𝑦 = 𝐶 (B) 𝑥 = 𝐶𝑦 2 (C) 𝑦 = 𝐶𝑥 (D) 𝑦 = 𝐶𝑥 2
𝒅𝒙
19. The general solution of a differential equation of the type is 𝒅𝒚 + 𝑷 𝒙 = 𝑸 is
(A) 𝑦 (𝑒 ∫ 𝑝𝑑𝑦 ) = ∫(Q 𝑒 ∫ 𝑃 𝑑𝑦 ) 𝑑𝑦 + 𝐶 (B) 𝑦 (𝑒 ∫ 𝑝𝑑𝑥 ) = ∫(Q 𝑒 ∫ 𝑃 𝑑𝑥 ) 𝑑𝑥 + 𝐶
(C) 𝑥 (𝑒 ∫ 𝑝𝑑𝑦 ) = ∫(Q 𝑒 ∫ 𝑃 𝑑𝑦 ) 𝑑𝑦 + 𝐶 (D) 𝑥 (𝑒 ∫ 𝑝𝑑𝑥 ) = ∫(Q 𝑒 ∫ 𝑃 𝑑𝑥 ) 𝑑𝑥 + 𝐶
20. The general solution of the differential equation 𝑒 𝑥 𝑑𝑦 + (𝑦 𝑒 𝑥 + 2𝑥) 𝑑𝑥 = 0 is
(A) 𝑥 𝑒 𝑦 + 𝑥 2 = 𝐶 (B) 𝑥 𝑒 𝑦 + 𝑦 2 = 𝐶 (C) 𝑦 𝑒 𝑥 + 𝑥 2 = 𝐶 (D) 𝑦 𝑒 𝑦 + 𝑥 2 = 𝐶

SHORT ANSWER QUESTIONS [ 3 MARKS]


𝑑𝑦
1. Solve the differential equation: 𝑑𝑥 = 1 + 𝑥 2 + 𝑦 2 + 𝑥 2 𝑦 2 , given that 𝑦 = 1 𝑤ℎ𝑒𝑛 𝑥 = 0.
𝑦 𝑦 𝑦 𝑦
2. Solve the differential equation: (𝑥 cos 𝑥 + 𝑦 sin 𝑥 ) 𝑦𝑑𝑥 = (𝑦 sin 𝑥 − 𝑥 cos 𝑥 ) 𝑥𝑑𝑦
𝑑𝑦
3. Solve the differential equation: 2𝑥 2 𝑑𝑥 − 2𝑥𝑦 + 𝑦 2 = 0
𝑑𝑦
4. Solve the differential equation: (𝑥 + 1) 𝑑𝑥 = 2𝑒 −𝑦 − 1; 𝑦 = 0 𝑤ℎ𝑒𝑛 𝑥 = 0
𝑑𝑦
5. Solve the differential equation: 𝑥𝑦 𝑑𝑥 = (𝑥 + 2)(𝑦 + 2); 𝑦 = −1 𝑤ℎ𝑒𝑛 𝑥 = 1
𝑥 𝑥
𝑥
6. Solve the differential equation : (1 + 2𝑒 𝑦 ) 𝑑𝑥 + 2𝑒 𝑦 (1 − 𝑦) 𝑑𝑦 = 0
𝑦
7. Solve the differential equation: 𝑦𝑑𝑥 + 𝑥 log (𝑥 ) 𝑑𝑦 − 2𝑥𝑑𝑦 = 0
8. Solve the differential equation : sec 2 𝑥 tan 𝑦 𝑑𝑥 + sec 2 𝑦 tan 𝑥 𝑑𝑦 = 0
9. Solve the differential equation :(𝑒 𝑥 + 𝑒 −𝑥 )𝑑𝑦 − (𝑒 𝑥 − 𝑒 −𝑥 )𝑑𝑥 = 0
10. Solve the differential equation :𝑒 𝑥 tan 𝑦 𝑑𝑥 + (1 − 𝑒 𝑥 ) sec 2 𝑦 𝑑𝑦 = 0
𝑑𝑦
11. Solve the differential equation : 𝑑𝑥 + 𝑦 = 1, (𝑦 ≠ 1)
𝑑𝑦
12. Solve the differential equation : cos (𝑑𝑥 ) = 𝑎, (𝑎 ∈ 𝑅); 𝑦 = 1 𝑤ℎ𝑒𝑛 𝑥 = 0
𝑑𝑦 𝑦 𝑦
13. Solve the differential equation : 𝑑𝑥 = 𝑥 + tan 𝑥
14. Solve the differential equation :(𝑥 2 + 3𝑥𝑦 + 𝑦 2 )𝑑𝑥 − 𝑥 2 𝑑𝑦 = 0
15. Solve the differential equation :(𝑥√𝑥 2 + 𝑦 2 − 𝑦 2 )𝑑𝑥 + 𝑥𝑦 𝑑𝑦 = 0
16. Solve the differential equation: 𝑥 𝑑𝑦 − 𝑦 𝑑𝑥 = √𝑥 2 + 𝑦 2 𝑑𝑥
𝑑𝑦
17. Find the general solution of differential equation : (𝑥 + 3𝑦 2 ) 𝑑𝑥 = 𝑦 (𝑦 > 0)
18. Solve the differential equation: 𝑦 𝑑𝑥 − 𝑥 𝑑𝑦 = 𝑥 2 𝑦 𝑑𝑥
𝑑𝑦
19. Solve the differential equation: 2𝑥 𝑑𝑥 + 𝑦 = 6𝑥 3
69

LONG ANSWER TYPE QUESTIONS [ 5 MARKS]


1. Find the particular solution of differential equation : ( tan−1 𝑦 − 𝑥)𝑑𝑦 = (1 + 𝑦 2 )𝑑𝑥 , given
when x = 0 , y = 0
2. Find the particular solution of differential equation cos 𝑦 𝑑𝑥 + (1 + 𝑒 −𝑥 ) sin 𝑦 𝑑𝑦 = 0, given
𝜋
that 𝑦 = when 𝑥 = 0.
4

3. Find the particular solution of differential equation (𝑥 2 − 𝑦𝑥 2 )𝑑𝑦 + (𝑦 2 + 𝑥 2 𝑦 2 ) 𝑑𝑥 = 0, given


that 𝑦 = 1 when 𝑥 = 1.
𝑑𝑦
4. Find the particular solution of differential equation (𝑥 + 1) 𝑑𝑥 = 2𝑒 −𝑦 − 1, given tha 𝑦 = 0 when

𝑥 = 0.
𝑑𝑦 𝑦 𝑦
5. Show that the differential equation 𝑥 𝑑𝑥 sin 𝑥 + 𝑥 − 𝑦 sin (𝑥 ) = 0 is homogeneous. Find the
𝜋
particular solution of this differential equation, given tha 𝑥 = 1 when 𝑦 = 2 .
𝑑𝑦 𝑦 2 +𝑦+1
6. Show that the general solution of the differential equation 𝑑𝑥 + 𝑥 2 +𝑥+1 = 0 is given by

(𝑥 + 𝑦 + 1) = 𝐴 (1 – 𝑥 – 𝑦 – 2𝑥𝑦), where A is parameter.


7. Find a particular solution of the differential equation (𝑥 – 𝑦) (𝑑𝑥 + 𝑑𝑦) = 𝑑𝑥 – 𝑑𝑦, given that
𝑦 = – 1, when 𝑥 = 0.
8. Prove that (𝑥 2 − 𝑦 2 ) = 𝑐(𝑥 2 + 𝑦 2 )2 is the general solution of differential equation
(𝑥 3 − 3𝑥𝑦 2 )𝑑𝑥 = (𝑦 3 − 3𝑦𝑥 2 )𝑑𝑦 , where c is a parameter.
𝑦 𝑦
9. Solve differential equation (𝑥 𝑑𝑦 − 𝑦 𝑑𝑥)𝑦 sin 𝑥 + (𝑦 𝑑𝑥 + 𝑥 𝑑𝑦)𝑥 cos 𝑥
𝑑𝑦 𝜋
10. Solve differential equation 𝑑𝑥 − 3 𝑦 cot 𝑥 = sin 2𝑥, given that 𝑦 = 2, when 𝑥 = 2 .

ANSWERS – DE - (MCQ)
1. (C) 3 2. (C) −𝟏 3. (A) 1 4. (C) 3 5. (A) 1 6. (D) not defined
7. (D) not defined 8. (A) 0 9. (B) 2 10. (B) 2 11. (D) 4 12. (D) 0
13. (A) 𝑒 𝑥 + 𝑒 −𝑦 = C 14. (C) 𝑥 = 𝑣𝑦 15. (D) 𝑦 2 𝑑𝑥 + (𝑥 2 – 𝑥𝑦 – 𝑦 2 ) 𝑑𝑦 = 0
1 1
16. (C) 17. (D) 2
18. (C) 𝑦 = 𝐶𝑥 19. (C) 𝑥 (𝑒 ∫ 𝑝𝑑𝑦 ) = ∫(Q 𝑒 ∫ 𝑃 𝑑𝑦 ) 𝑑𝑦 + 𝐶
𝑥 √1−𝑦

SHORT ANSWER QUESTIONS – DE - [ 3 MARKS ]

𝑥3 𝜋 𝑦 2𝑥
1. tan−1 𝑦 = 𝑥 + +4 2. xycos 𝑥 = k 3. 𝑦 = log 𝑥+𝑐 4. (𝑥 + 1)(2 − 𝑒 𝑦 ) = 1
3
𝑥
𝑥
5. 𝑦 − 2𝑙𝑜𝑔|𝑦 + 2| = 𝑥 + 2𝑙𝑜𝑔𝑥 − 2] 6. x+2y𝑒 𝑦 =k 7. 1+log 𝑦=k.
8. tan 𝑥 tan 𝑦 = 𝐶 9.𝑦 = log(𝑒 𝑥 + 𝑒 −𝑥 ) + 𝐶 10. tan 𝑦 = 𝐶(1 − 𝑒 𝑥 ) 11. 𝑦 = 1 + 𝐴𝑒 −𝑥
𝑦−2 𝑦 𝑥 𝑦2
12. cos ( )=𝑎 13. sin 𝑥 = 𝐴𝑥 14. − 𝑦+𝑥 = log 𝑥 + 𝐶 15. √1 + 𝑥 2 = − log 𝑥 + 𝐶
𝑥
𝑥2 1
6
16. 𝑦 + √𝑥 2 + 𝑦 2 = 𝐶𝑥 2 17. 𝑥 = 3𝑦 2 + 𝐶𝑦 18. 𝑦 = 𝐶x𝑒 − 2 19. 𝑦 = 7 𝑥 3 + 𝐶𝑥 −2
(3 sin 3𝑥−2 cos 3𝑥)
20. 𝑦 = + 𝐶𝑒 2𝑥 .
13
70

−𝟏 𝒚
LONG ANSWER QUESTIONS – DE - [ 5 MARKS ] 1. 𝒙 = (𝐭𝐚𝐧−𝟏 𝒚 − 𝟏) + 𝑪𝒆− 𝐭𝐚𝐧
2. 𝐬𝐞𝐜 𝒚 (𝟏 + 𝒆𝒙 ) = 𝟐√𝟐 3. 𝒙𝟐 𝒚 = 𝒙 + 𝒙𝒚 𝐥𝐨𝐠 𝒚 + 𝒚 − 𝒙𝒚 4. (𝒙 + 𝟏)(𝟐 − 𝒆𝒚 ) = 𝟏
𝒚 𝒚
5. 𝐥𝐨𝐠 𝒙 = 𝐜𝐨𝐬 7. 𝐥𝐨𝐠|𝒙 − 𝒚| = 𝒙 + 𝒚 + 𝟏 9. 𝐬𝐞𝐜 = 𝑪𝒙𝒚 10. 𝒚 = 𝟒 𝐬𝐢𝐧𝟑 𝒙 − 𝟐 𝐬𝐢𝐧𝟐 𝒙
𝒙 𝒙

Chapter 10 – VECTOR ALGEBRA


Some Basic Concepts
Vector quantity: A quantity that has magnitude as well as direction is called a vector quantity .
̅̅̅̅ or simply as 𝑎⃗ , and read as ‘vector 𝐴𝐵
Notice that a directed line segment is a vector , denoted as 𝐴𝐵 ̅̅̅̅ ’
or ‘vector 𝑎⃗ ’.

̅̅̅̅ starts is called its initial point, and the point B where it ends is called
The point A from where the vector 𝐴𝐵
its terminal point.

The distance between initial and terminal points of a vector is called the magnitude (or length) of the vector,
̅̅̅̅| or |𝑎⃗| or 𝑎 .
denoted as |𝐴𝐵

The arrow indicates the direction of the vector.

Position Vector

The vector ̅̅̅̅


𝑂𝑃 having O and P as its initial and terminal points, respectively, is called the position vector of
the point P with respect to O.

̅̅̅̅ (or 𝑟⃗ ) is given by


the magnitude of 𝑂𝑃 ̅̅̅̅| = √𝑥 2 + 𝑦 2 + 𝑧 2
| 𝑂𝑃

In practice, the position vectors of points A, B, C, etc., with respect to the origin O are denoted by 𝑎⃗, 𝑏⃗⃗, 𝑐⃗
etc., respectively.
71

Types of Vectors:
Zero Vector : A vector whose initial and terminal points coincide, is called a zero vector (or null vector), and
denoted as ⃗0⃗. Zero vector can not be assigned a definite direction as it has zero magnitude. Or, alternatively
otherwise, it may be regarded as having any direction. The vectors ̅̅̅̅
𝐴𝐴 and ̅̅̅̅
𝐵𝐵 represent the zero vector.

Unit Vector : A vector whose magnitude is unity (i.e., 1 unit) is called a unit vector. The unit vector in the
direction of a given vector 𝑎⃗ is denoted by 𝑎̂ .

Coinitial Vectors : Two or more vectors having the same initial point are called coinitial vectors.

Collinear Vectors: Two or more vectors are said to be collinear if they are parallel to the same line,
irrespective of their magnitudes and directions.

Equal Vectors: Two vectors 𝑎⃗ and 𝑏⃗⃗ are said to be equal, if they have the same magnitude and direction
regardless of the positions of their initial points, and written as 𝑎⃗ = 𝑏⃗⃗.

Negative of a Vector: A vector whose magnitude is the same as that of a given vector (say 𝐴𝐵 ̅̅̅̅, ), but
̅̅̅̅ vector is negative of
direction is opposite to that of it, is called negative of the given vector. For example, 𝐵𝐴
̅̅̅̅ vector , and written as 𝐵𝐴
the 𝐴𝐵 ̅̅̅̅= – 𝐴𝐵
̅̅̅̅.

Addition of Vectors
Triangle law of vector addition:

⃗⃗⃗⃗⃗⃗ + 𝐵𝐶
𝐴𝐵 ⃗⃗⃗⃗⃗⃗ = 𝐴𝐶
⃗⃗⃗⃗⃗⃗

parallelogram law of vector addition:

If we have two vectors 𝑎⃗ and 𝑏⃗⃗ represented by the two adjacent sides of a parallelogram in magnitude and
direction , then their sum 𝑎⃗ + 𝑏⃗⃗ is represented in magnitude and direction by the diagonal of the parallelogram
through their common point. This is known as the parallelogram law of vector addition.
72

Properties of vector addition :

• vector addition is Commutative i.e. 𝑎⃗ + 𝑏⃗⃗ = 𝑏⃗⃗ + 𝑎⃗


• vector addition is Associative i.e. ( 𝑎⃗ + 𝑏⃗⃗) + 𝑐⃗ = 𝑎⃗ +( 𝑏⃗⃗ + 𝑐⃗ )
• The associative property of vector addition enables us to write the sum of three vectors without
using brackets.
• The zero vector⃗⃗⃗0⃗ is called the additive identity for the vector addition.
Note that for any vector 𝑎⃗, we have : 𝑎⃗ + ⃗⃗⃗0⃗ = ⃗⃗⃗0⃗ + 𝑎⃗ = 𝑎⃗
Multiplication of a Vector by a Scalar

⃗⃗ be a given vector and λ a scalar. Then the product of the vector by the scalar λ, denoted as λ𝒂
Let 𝒂 ⃗⃗ , is called
the multiplication of vector by the scalar λ.

⃗⃗ has the direction same (or opposite) to that of vector according as the value of λ is positive (or
The vector λ𝒂
negative).

⃗⃗ is |λ| times the magnitude of the vector 𝒂


The magnitude of vector λ𝒂 ⃗⃗ , i.e | λ𝒂
⃗⃗|=|λ||𝒂
⃗⃗|

Negative (or additive inverse) of vector : . The vector –𝒂 ⃗⃗ is called the negative (or additive inverse) of vector
⃗⃗ and we always have
𝒂 ⃗⃗) + 𝑎⃗ = ⃗⃗⃗0⃗
⃗⃗) = ( – 𝒂
𝑎⃗ +( – 𝒂

component form of a vector: The position vector of P with reference to O is given by

𝑂𝑃 (or 𝑟⃗ )=𝑥 𝑖̂ + 𝑦 𝑗̂ + 𝑧 𝑘̂
̅̅̅̅

This form of any vector is called its component form. Here, 𝑥, 𝑦 and 𝑧 are called as the scalar components of 𝑟⃗
and , 𝑥 𝑖̂, 𝑦 𝑗̂ 𝑎𝑛𝑑 𝑧 𝑘̂ and are called the vector components of 𝑟⃗ along the respective axes.

• If 𝑎⃗ and 𝑏⃗⃗ are any two vectors given in the component form 𝑎1 𝑖̂ + 𝑎2 𝑗̂ + 𝑎3 𝑘̂ and 𝑏1 𝑖̂ + 𝑏2 𝑗̂ +
𝑏3 𝑘̂ respectively, then
• The sum (or resultant) of the vectors is given by: 𝑎⃗ + 𝑏⃗⃗=(𝑎1 + 𝑏1 )𝑖̂ + (𝑎2 + 𝑏2 ) 𝑗̂ + (𝑎3 + 𝑏3 ) 𝑘̂
• The difference of the vectors is given by : 𝑎⃗ − 𝑏⃗⃗=(𝑎1 − 𝑏1 )𝑖̂ + (𝑎2 − 𝑏2 ) 𝑗̂ + (𝑎3 − 𝑏3 ) 𝑘̂
• The vectors 𝑎⃗ and 𝑏⃗⃗ are equal if and only if: 𝑎1 = 𝑏1, 𝑎2 = 𝑏2 and 𝑎3 = 𝑏3
• The multiplication of vector 𝑎⃗ by any scalar λ is given by: λ𝒂 ⃗⃗ = (𝛌𝑎1 ) 𝑖̂ + (𝛌𝑎2) 𝑗̂ + (𝛌𝑎3 )𝑘̂
𝑎 𝑎 𝑎
• 𝑎⃗ = 𝑎1 𝑖̂ + 𝑎2 𝑗̂ + 𝑎3 𝑘̂ and 𝑏⃗⃗ = 𝑏1 𝑖̂ + 𝑏2 𝑗̂ + 𝑏3 𝑘̂ s are collinear if and only if 1 = 2 = 3
𝑏1 𝑏2 𝑏3
• If 𝑎⃗ = 𝑎1 𝑖̂ + 𝑎2 𝑗̂ + 𝑎3 𝑘̂ , then 𝑎1 , 𝑎2 , 𝑎3 are also called direction ratios of 𝑎⃗ .
Vector joining two points

If 𝑃1 (𝑥1 , 𝑦1 , 𝑧1 ) and 𝑃2 (𝑥2 , 𝑦2 , 𝑧2 ) are any two points, then the vector joining 𝑃1 and 𝑃2 is the vector

⃗⃗⃗⃗⃗⃗⃗⃗⃗⃗⃗
𝑃 ̂
1 𝑃2 = (𝑥2 − 𝑥1 )𝑖̂ + (𝑦2 − 𝑦1 ) 𝑗̂ + (𝑧2 − 𝑧1 ) 𝑘

Section formula: The position vector of the point R which divides P and Q internally in the ratio of
m : n is given by

𝑚𝑏⃗⃗ + 𝑛𝑎⃗
⃗⃗⃗⃗⃗⃗ =
𝑂𝑅
𝑚+𝑛
73

• The position vector of the point R which divides P and Q externally in the ratio of m : n is given by

𝑚𝑏⃗⃗ − 𝑛𝑎⃗
⃗⃗⃗⃗⃗⃗ =
𝑂𝑅
𝑚−𝑛

Product of Two Vectors:


⃗⃗
(A) Scalar (or dot) product of two vectors: The scalar product of two nonzero vectors 𝑎⃗ and 𝑏
denoted by 𝑎⃗ . 𝑏⃗⃗ is defined as : 𝑎⃗ . 𝑏⃗⃗ = |𝒂 ⃗⃗| cos 𝜃
⃗⃗||𝒃
where, θ is the angle between 𝑎⃗ and 𝑏⃗⃗ , 0 ≤ 𝜃 ≤ 𝜋.
• 𝐼𝑓 𝑎⃗ = 𝑎1 𝑖̂ + 𝑎2 𝑗̂ + 𝑎3 𝑘̂ and 𝑏⃗⃗ = 𝑏1 𝑖̂ + 𝑏2 𝑗̂ + 𝑏3 𝑘̂ , then
𝑎⃗ . 𝑏⃗⃗ = 𝑎1 𝑏1 + 𝑎2 𝑏2 + 𝑎3 𝑏3
• For mutually perpendicular unit vectors , and , we have
̂. 𝒌
𝒊̂. 𝒊̂ = 𝒋̂. 𝒋̂ = 𝒌 ̂ = 𝟏 and
̂=𝒌
𝒊̂. 𝒋̂ = 𝒋̂. 𝒌 ̂. 𝒊̂ = 𝟎
⃗⃗ . 𝑏⃗
𝑎
• The angle θ between two nonzero vectors 𝑎⃗ and ⃗⃗⃗⃗
𝑏 is given by 𝜽 = 𝐜𝐨𝐬 −𝟏 ( ⃗⃗ )
⃗⃗||𝒃|
|𝒂

• Let 𝑎⃗ and ⃗⃗⃗⃗


𝑏 be two nonzero vectors, then 𝑎⃗ . 𝑏⃗⃗ =0 , if and only if are perpendicular to each other.
• 𝑎⃗ . 𝑎⃗ = |𝑎⃗|2
⃗⃗ ⃗⃗
• Projection of a vector on a line: . Projection of a vector 𝑎⃗ on other vector 𝑏⃗⃗=𝑎⃗. 𝑏̂ = 𝑎⃗. (|𝒃⃗⃗|) = |𝒃⃗⃗|
𝑏 𝑎⃗⃗ . 𝑏

Vector (or cross) product of two vectors:


The vector product of two nonzero vectors 𝑎⃗ and ⃗⃗⃗⃗
𝑏 , is denoted by 𝑎⃗ × 𝑏⃗⃗ and defined as
⃗⃗ × ⃗𝒃⃗ = |𝒂
𝒂 ⃗⃗| 𝐬𝐢𝐧 𝜽 𝒏
⃗⃗||𝒃 ̂
where, θ is the angle between 𝑎⃗ and 𝑏⃗⃗ , 0 ≤ 𝜃 ≤ 𝜋. and𝑛̂ is a unit vector perpendicular to both 𝑎⃗ and ⃗⃗⃗⃗
𝑏 ,
⃗⃗⃗⃗
such that 𝑎⃗ , 𝑏 and 𝑛̂ form a right handed system,

• For mutually perpendicular unit vectors , and , we have


̂×𝒌
𝒊̂ × 𝒊̂ = 𝒋̂ × 𝒋̂ = 𝒌 ̂ = ⃗⃗⃗0 and
̂, 𝒋̂ × 𝒊̂ = −𝒌
𝒊̂ × 𝒋̂ = 𝒌 ̂ , 𝒋̂ × 𝒌
̂ = 𝒊̂ , 𝒌
̂ × 𝒋̂ = −𝒊̂ , 𝒌
̂ × 𝒊̂ = 𝒋̂, 𝒊̂ × 𝒌
̂ = −𝒋̂
1
• If 𝑎⃗ and ⃗⃗⃗⃗ ⃗⃗ × ⃗𝒃⃗|
𝑏 represent the adjacent sides of a triangle then its area =2 |𝒂
• If 𝑎⃗ and 𝑏
⃗⃗⃗⃗ represent the adjacent sides of a parallelogram, then its area=|𝒂⃗⃗ × ⃗𝒃⃗|
𝒊̂ 𝒋̂ ̂
𝒌
• If 𝑎⃗ = 𝑎1 𝑖̂ + 𝑎2 𝑗̂ + 𝑎3 𝑘 and 𝑏 = 𝑏1 𝑖̂ + 𝑏2 𝑗̂ + 𝑏3 𝑘 , then
̂ ⃗⃗ ̂ 𝒂 ⃗⃗
⃗⃗ × 𝒃 = |𝑎1 𝑎2 𝑎3 |
𝑏1 𝑏2 𝑏3
VECTOR ALGEBRA

1. Given ⃗⃗⃗⃗⃗⃗⃗
𝑨𝑩 = 𝟑𝒊̂ - 𝑱̂ - 5𝒌
̂ and coordinates of the terminal point are
(0, 1, 3). Find the coordinate of the initial point.

SOL: Given ⃗⃗⃗⃗⃗⃗⃗ ̂


𝑨𝑩 = 𝟑𝒊̂ − 𝑱̂ − 𝟓𝒌 B (0, 1, 3) ⃗⃗⃗⃗⃗⃗⃗ ̂
𝑶𝑩 = 𝑱̂ + 𝟑𝒌 ⃗⃗⃗⃗⃗⃗⃗
𝑶𝑨 = ?
⃗⃗⃗⃗⃗⃗⃗ = ⃗⃗⃗⃗⃗⃗⃗
𝑨𝑩 𝑶𝑩 − ⃗⃗⃗⃗⃗⃗⃗
𝑶𝑨 𝑶𝑨 = ⃗⃗⃗⃗⃗⃗⃗
⃗⃗⃗⃗⃗⃗⃗ ⃗⃗⃗⃗⃗⃗⃗
𝑶𝑩 − 𝑨𝑩
74

⃗⃗⃗⃗⃗⃗⃗ ̂ − ( 𝟑𝒊̂ − 𝑱̂ − 𝟓𝒌
𝑶𝑨 = 𝑱̂ + 𝟑𝒌 ̂ ) = 𝑱̂ + 𝟑𝒌
̂ − 𝟑𝒊̂ + 𝑱̂ + 𝟓𝒌
̂ − = − 𝟑𝒊̂ + 𝟐𝑱̂ + 𝟖𝒌
̂ A (–3, 2, 8)

2. If ⃗⃗| = 2, |𝒃
|𝒂 ⃗⃗  𝒃
⃗⃗| = 5 and |𝒂 ⃗⃗• 𝒃
⃗⃗| = 8, find the value of 𝒂 ⃗⃗.

⃗⃗| = 2, |𝒃
SOL: Given |𝒂 ⃗⃗  𝒃
⃗⃗| = 5 and |𝒂 ⃗⃗• 𝒃
⃗⃗| = 8, 𝒂 ⃗⃗ =?
𝟒 𝟑
⃗⃗  ⃗𝒃⃗| =
|𝒂 |𝒂 ⃗⃗| 𝑺𝒊𝒏 = 𝟖
⃗⃗ ||𝒃 (𝟐)(𝟓) 𝑺𝒊𝒏 = 𝟖 𝑺𝒊𝒏 = 𝑪𝒐𝒔 =
𝟓 𝟓
𝟑
⃗⃗ . ⃗𝒃⃗ = |𝒂
𝒂 ⃗⃗| 𝑪𝒐𝒔𝜽 = (𝟐)(𝟓) = 𝟔
⃗⃗ ||𝒃
𝟓

⃗⃗ , 𝒃
3. If 𝒂 ⃗⃗ are any two unit vectors and  is the angle between them,
𝜽 𝟏
then show that 𝑺𝒊𝒏 ( ) = ⃗⃗ − ⃗𝒃⃗|
|𝒂
𝟐 𝟐
SOL: Given |𝒂
⃗⃗| = 1, |𝒃
⃗⃗| = 1 ⃗⃗ = |𝒂
⃗⃗ . 𝒂
𝒂 ⃗⃗ |𝟐 = 𝒂𝟐
⃗⃗| Cos 0 = |𝒂
⃗⃗ ||𝒂
𝟐
⃗⃗ − ⃗𝒃⃗| = |𝒂
|𝒂 ⃗⃗ • ⃗𝒃⃗
⃗⃗| 𝟐 − 𝟐 𝒂
⃗⃗| 𝟐 + |𝒃
= (𝟏)𝟐 + (𝟏)𝟐 − 𝟐 |𝒂 ⃗⃗| 𝑪𝒐𝒔𝜽
⃗⃗ ||𝒃
= 𝟏 + 𝟏 − 𝟐 (𝟏)(𝟏) 𝑪𝒐𝒔𝜽
= 𝟐( 𝟏 − 𝑪𝒐𝒔𝜽 )
𝜽
= 𝟐 (𝟐 𝑺𝒊𝒏𝟐 )
𝟐
𝜽 𝜽 𝟏
⃗⃗ − ⃗𝒃⃗| = 𝟐 𝑺𝒊𝒏
|𝒂 𝑺𝒊𝒏 ( ) = ⃗⃗ − ⃗𝒃⃗|
|𝒂
𝟐 𝟐 𝟐

4. If 𝒂
⃗⃗ = 𝒊̂ +2 𝑱̂ – 𝒌
̂ and 𝒃
⃗⃗ =3𝑰̂ + 𝑱̂ - 𝟓𝒌
̂ find a unit vector in the direction of
⃗⃗ − ⃗𝒃⃗
𝒂
⃗⃗ = 𝒊̂ + 𝟐𝑱̂ − 𝒌
SOL: Given: 𝒂 ̂ and ⃗𝒃⃗ =𝟑𝒊̂ + 𝑱̂ − 𝟓𝒌
̂
⃗⃗ − ⃗𝒃⃗ = 𝒊̂ + 𝟐𝑱̂ − 𝒌̂
𝒂 ̂ ) = −𝟐𝒊̂ + 𝑱̂ + 𝟒𝒌̂ = 𝒄
− ( 𝟑𝒊̂ + 𝑱̂ − 𝟓𝒌 ⃗⃗
⃗𝒄
⃗⃗ ̂ ̂
−𝟐𝒊 + 𝑱 + 𝟒𝒌̂ −𝟐𝒊̂ 𝑱̂ ̂
𝟒𝒌
Unit vector in the direction 𝒄̂ = ⃗⃗⃗ = = + +
|𝒄| √𝟒+𝟏+𝟏𝟔 √𝟐𝟏 √𝟐𝟏 √𝟐𝟏

5. If the position vectors of the points A and B are 𝟐𝒊̂ + 𝟑𝑱̂ − 𝒌


̂ and 𝟑𝒊̂ +
̂ then find the vector of magnitude 6 units in the direction of ⃗⃗⃗⃗⃗⃗⃗
𝟐𝑱̂ + 𝒌 𝑨𝑩

SOL: Given ⃗⃗⃗⃗⃗⃗⃗


𝑶𝑨 = 𝟐𝒊̂ + 𝟑𝑱̂ − 𝒌 ̂ and ⃗⃗⃗⃗⃗⃗⃗ ̂
𝑶𝑩 = 𝟑𝒊̂ + 𝟐𝑱̂ + 𝒌 ⃗⃗⃗⃗⃗⃗⃗
𝑨𝑩 = ⃗⃗⃗⃗⃗⃗⃗
𝑶𝑩 − ⃗⃗⃗⃗⃗⃗⃗ ̂
𝑶𝑨 = 𝒊̂ − 𝑱̂ + 𝟐𝒌
⃗⃗⃗⃗⃗⃗⃗
Unit vector 𝑨𝑩 ⃗⃗⃗⃗⃗⃗⃗ = 𝑨𝑩 = 𝒊̂− 𝑱̂ + 𝟐𝒌̂ = 𝒊̂ − 𝑱̂ + 𝟐𝒌̂
⃗⃗⃗⃗⃗⃗⃗|
|𝑨𝑩 √𝟏 + 𝟏 + 𝟒 √𝟔 √𝟔 √𝟔
𝒊̂ ̂ ̂
The vector of magnitude 6 units in the direction of ⃗⃗⃗⃗⃗⃗⃗
𝑨𝑩 = 𝟔 (
√𝟔
− √𝑱𝟔 + 𝟐𝒌
√𝟔
)
6. If P (1, 5, 4) and Q (4, 1, -2), find the direction ratios of 𝑷𝑸
⃗⃗⃗⃗⃗⃗⃗
SOL: P (1, 5, 4) and Q (4, 1, -2), ⃗⃗⃗⃗⃗⃗⃗ ̂
𝑶𝑷 = 𝒊̂ + 𝟓𝑱̂ + 𝟒𝒌 ⃗⃗⃗⃗⃗⃗⃗ ̂
𝑶𝑸 = 𝟒𝒊̂ + 𝑱̂ − 𝟐𝒌
⃗⃗⃗⃗⃗⃗⃗
𝑷𝑸 = ⃗⃗⃗⃗⃗⃗⃗
𝑶𝑸 − ⃗⃗⃗⃗⃗⃗⃗ ̂ The direction ratios of ⃗⃗⃗⃗⃗⃗⃗
𝑶𝑷 = 𝟑𝒊̂ − 𝟒𝑱̂ − 𝟔𝒌 𝑷𝑸 = 3, – 4, – 6
75

̂ and 𝟑𝒊̂ + ̂𝑱 − 𝒌
7. Find  , if the vectors 𝟑𝒊̂ + ̂𝑱 − 𝟓𝒌 ̂ are parallel
̂ and 𝟑𝒊̂ + 𝑱̂ − 𝒌
SOL: The vectors 𝟑𝒊̂ + 𝑱̂ − 𝟓𝒌 ̂ are parallel =5

̂ and 𝟐𝒊̂ + 𝟑𝑱̂ − 𝒌


8. Find , if the vectors 𝟑𝒊̂ − 𝑱̂ − 𝟓𝒌 ̂ are perpendicular

SOL: Given: 𝟑𝒊̂ − 𝑱̂ − 𝟓𝒌


̂ and 𝟐𝒊̂ + 𝟑𝑱̂ − 𝒌
̂ are perpendicular ⃗⃗• ⃗𝒃⃗ = 𝟎
𝒂
−𝟑
3(2) –1(3) –5 (− ) = 0 5 = –3 =
𝟓

9. If 𝒂⃗⃗ = 𝟒𝒊̂ + 𝟐𝑱̂ − 𝒌


̂ and 𝒃
⃗⃗ =𝟓𝒊̂ + 𝟐𝑱̂ − 𝟑𝒌
̂ find the angle between 𝒂 ⃗⃗ and
⃗⃗ + 𝒃
⃗⃗ − ⃗𝒃⃗
𝒂

SOL: ⃗⃗ = 𝟒𝒊̂ + 𝟐𝑱̂ − 𝒌


𝒂 ̂ and ⃗𝒃⃗ = 𝟓𝒊̂ + 𝟐𝑱̂ − 𝟑𝒌̂
⃗⃗ + ⃗𝒃⃗ = 𝟗𝒊̂ + 𝟒𝑱̂ − 𝟒𝒌
𝒂 ̂= 𝒄 ⃗⃗ ⃗⃗ − ⃗𝒃⃗ = −𝒊̂ + 𝟐𝒌
𝒂 ̂ = ⃗𝒅⃗
⃗𝒄 ⃗⃗⃗⃗
⃗⃗ .𝒅 𝟗(−𝟏) + 𝟒(𝟎) − 𝟒(𝟐) ( −𝟏𝟕)
The angle between 𝒄 ⃗⃗ =
⃗⃗ 𝒂𝒏𝒅 𝒅 𝑪𝒐𝒔𝜽 = ⃗⃗⃗ ||⃗⃗⃗⃗
= √𝟖𝟏 + 𝟏𝟔 + 𝟏𝟔 √𝟏 + 𝟒
= √𝟏𝟏𝟑 √𝟓
|𝒄 𝒅|

10. Find the projection of 𝒂


⃗⃗ = 𝒊̂ + 𝑱̂ + 𝟒𝒌 ̂ on 𝒃
⃗⃗ = 2𝒊̂ + 𝑱̂ + 𝟐𝒌 ̂
SOL: Given the projection of 𝒂⃗⃗ = 𝒊̂ + 𝑱̂ + 𝟒𝒌
̂ on 𝒃⃗⃗ = 𝟐𝒊̂ + 𝑱̂ + 𝟐𝒌
̂
𝒂 ⃗⃗
⃗⃗ . 𝒃 𝟏(𝟐) + 𝟏(𝟏) + 𝟒(𝟐) 𝟏𝟏
⃗⃗ on ⃗𝒃⃗ = ⃗⃗ =
Projection of 𝒂 =
|𝒃 | √𝟒 + 𝟏 + 𝟒 𝟑

11. Find the position vector of a point R which divided the line segment joining
the points P and Q with position vectors 𝒊̂ + 𝟐𝒋̂ − 𝒌
̂ and −𝒊̂ + 𝒋̂ + 𝒌
̂ respectively
in the ration 2 : 1 i) internally ii) externally.

SOL: The position vector of a point R which divided the line segment joining
the points P and Q with position vectors 𝒊̂ + 𝟐𝒋̂ − 𝒌 ̂ and −𝒊̂ + 𝒋̂ + 𝒌 ̂ respectively in
the ration 2 : 1
𝒎𝒃⃗⃗ +𝒏 𝒂
⃗⃗ 𝟐(−𝒊̂ +𝒋̂ + 𝒌̂ ) + 𝟏 (𝒊̂ +𝟐𝒋̂ − 𝒌̂ ) ̂ + 𝒊̂ + 𝟐𝒋̂ − 𝒌̂
−𝟐𝒊̂ +𝟐𝒋̂ + 𝟐𝒌 ̂
−𝒊̂ + 𝟒𝒋̂ +𝒌
= ⃗⃗⃗⃗⃗⃗⃗
𝑶𝑹 = = = =
𝒎+𝒏 𝟐+𝟏 𝟑 𝟑

12. Show that the points A (2, 6, 3), B (1, 2, 7) and C (3, 10, -1) are
collinear.

SOL: Given points A (2, 6, 3), B (1, 2, 7) and C (3, 10, –1)
⃗⃗⃗⃗⃗⃗⃗
𝑶𝑨 = 𝟐𝒊̂ + 𝟔𝑱̂ + 𝟑𝒌 ̂ ⃗⃗⃗⃗⃗⃗⃗
𝑶𝑩 = 𝒊̂ + 𝟐𝑱̂ + 𝟕𝒌 ̂ ̂
⃗⃗⃗⃗⃗⃗⃗ = 𝟑𝒊̂ + 𝟏𝟎𝑱̂ − 𝒌
𝑶𝑪
⃗⃗⃗⃗⃗⃗⃗ = ⃗⃗⃗⃗⃗⃗⃗
𝑨𝑩 𝑶𝑩 − ⃗⃗⃗⃗⃗⃗⃗ ̂
𝑶𝑨 = −𝒊̂ − 𝟒𝑱̂ + 𝟒𝒌 ⃗⃗⃗⃗⃗⃗ = 𝑶𝑪
𝑨𝑪 ⃗⃗⃗⃗⃗⃗⃗ − ⃗⃗⃗⃗⃗⃗⃗
𝑶𝑨 = 𝒊̂ + 𝟒𝑱̂ − 𝟒𝒌̂
⃗⃗⃗⃗⃗⃗⃗
𝑨𝑩 = (−𝟏) ⃗⃗⃗⃗⃗⃗
𝑨𝑪 ⃗⃗⃗⃗⃗⃗⃗
𝑨𝑩 𝒊𝒔 𝒑𝒂𝒓𝒂𝒍𝒍𝒆𝒍 𝒕𝒐 ⃗⃗⃗⃗⃗⃗
𝑨𝑪 A, B, C are collinear

13. If 𝒂
⃗⃗ , ⃗𝒃⃗ and 𝒄
⃗⃗ are 3 unit vectors and 𝒂 ⃗⃗ = 𝟎 find 𝒂
⃗⃗ + ⃗𝒃⃗ + 𝒄 ⃗⃗ . ⃗𝒃⃗ + ⃗𝒃⃗ . 𝒄
⃗⃗ + 𝒄
⃗⃗ . ⃗⃗⃗⃗
𝒂
SOL: Given |𝒂 ⃗⃗| = 𝟏 , 𝒂
⃗⃗| = 𝟏, |𝒃| = 𝟏, |𝒄 ⃗⃗ + ⃗𝒃⃗ + 𝒄
⃗⃗ = 𝟎
2 𝟐 𝟐 𝟐
⃗⃗⃗ + ⃗𝒃⃗ + 𝒄
(𝒂 ⃗⃗) = |⃗𝒂
⃗⃗⃗| + |𝒃| + |⃗𝒄⃗| + ⃗⃗⃗ • ⃗⃗𝒃
𝟐 ⃗𝒂 ⃗⃗ • ⃗𝒄⃗ + 𝟐 ⃗𝒄⃗ • ⃗𝒂
⃗⃗ + 𝟐 ⃗⃗𝒃 ⃗⃗⃗
76

−𝟑
0= ⃗⃗ • 𝒃
𝟏+𝟏+𝟏+ 𝟐(𝒂 ⃗⃗ • 𝒄
⃗⃗ + 𝒃 ⃗⃗ • 𝒂
⃗⃗ + 𝒄 ⃗⃗ ) ⃗⃗ • 𝒃
𝒂 ⃗⃗ • 𝒄
⃗⃗ + 𝒃 ⃗⃗ • 𝒂
⃗⃗ + 𝒄 ⃗⃗ =
𝟐

14. If 𝒂
⃗⃗ , ⃗𝒃⃗ and 𝒄
⃗⃗ are three unit vectors such that 𝒂
⃗⃗ + ⃗𝒃⃗ + 𝒄
⃗⃗ = 0, prove
that 𝒂
⃗⃗ × ⃗𝒃⃗ = ⃗𝒃⃗ × 𝒄
⃗⃗ = 𝒄
⃗⃗ × 𝒂
⃗⃗.

⃗⃗ , 𝒃
SOL: Given 𝒂 ⃗⃗ and 𝒄
⃗⃗ are three vectors such that 𝒂 ⃗⃗ + 𝒄
⃗⃗ + 𝒃 ⃗⃗ = 𝟎 ⃗⃗ = −𝒄
⃗⃗ + 𝒃
𝒂 ⃗⃗
⃗⃗ 𝑿 (𝒂
𝒂 ⃗⃗ + ⃗𝒃⃗ ) = − 𝒂⃗⃗ 𝑿 𝒄⃗⃗ ⃗⃗ 𝑿 𝒂
𝒂 ⃗⃗ + 𝒂⃗⃗ 𝑿 ⃗𝒃⃗ = − 𝒂⃗⃗ 𝑿 𝒄⃗⃗ ⃗⃗ 𝑿 ⃗𝒃⃗ = 𝒄
𝒂 ⃗⃗ 𝑿 𝒂⃗⃗ ….(1)
⃗𝒃⃗ 𝑿 (𝒂⃗⃗ + ⃗𝒃⃗ ) = − ⃗𝒃⃗ 𝑿 𝒄⃗⃗ ⃗𝒃⃗ 𝑿 𝒂⃗⃗ + ⃗𝒃⃗ 𝑿 ⃗𝒃⃗ = − ⃗𝒃⃗ 𝑿 𝒄⃗⃗ ⃗⃗ 𝑿 ⃗𝒃⃗ = ⃗𝒃⃗ 𝑿 𝒄
𝒂 ⃗⃗ ….(2)
⃗⃗ 𝑿 ⃗𝒃⃗ = ⃗𝒃⃗ 𝑿 𝒄
From (1) and (2) 𝒂 ⃗⃗ = 𝒄
⃗⃗ 𝑿 ⃗⃗⃗⃗
𝒂

15. Find the area of the parallelogram with diagonals 𝟑𝒊̂ + ̂𝑱 + 𝟐𝒌 ̂ and
𝒊̂ − 𝟑𝑱̂ + 𝟒𝒌̂.
SOL: If 𝒅 ⃗⃗⃗⃗⃗𝟐 are diagonals of Parallelogram then Area = 𝟏 |𝒅
⃗⃗⃗⃗⃗𝟏 and 𝒅 ⃗⃗⃗⃗⃗𝟏 𝐗 ⃗⃗⃗⃗⃗
𝒅𝟐 |
𝟐
𝒊̂ 𝒋̂ ̂
𝒌 𝒊̂ 𝒋̂ ̂
𝒌
⃗⃗⃗⃗⃗
𝒅𝟏 𝐗 ⃗⃗⃗⃗⃗
𝒅𝟐 = |𝒂𝟏 𝒂𝟐 𝒂𝟑 | = |𝟑 𝟏 𝟐 |
𝒃𝟏 𝒃𝟐 𝒃𝟑 𝟏 −𝟑 𝟒
. = 𝒊̂(𝟒 + 𝟔) − 𝑱̂ (𝟏𝟐 − 𝟐) + 𝒌̂ (−𝟗 − 𝟏) = 𝟏𝟎𝒊̂ − 𝟏𝟎𝑱̂ − 𝟏𝟎𝒌̂
⃗⃗⃗⃗⃗𝟏 𝐗 ⃗⃗⃗⃗⃗
|𝒅 𝒅𝟐 | = √𝟏𝟎𝟎 + 𝟏𝟎𝟎 + 𝟏𝟎𝟎 = 𝟏𝟎√𝟑
𝟏
𝑨𝒓𝒆𝒂 𝒐𝒇 𝑷𝒂𝒓𝒂𝒍𝒍𝒆𝒍𝒐𝒈𝒓𝒂𝒎 = |𝒅 ⃗⃗⃗⃗⃗ 𝐗 ⃗⃗⃗⃗⃗
𝒅𝟐 | = 𝟓√𝟑 𝒔𝒒𝒖𝒂𝒓𝒆 𝒖𝒏𝒊𝒕𝒔
𝟐 𝟏
16). If 𝒂
⃗⃗ = 𝟐𝒊̂ + 𝟐𝑱̂ + 𝟑𝒌̂ ̂ and 𝒄
⃗⃗ = −𝒊̂ + 𝟐𝑱̂ + 𝒌
𝒃 ⃗⃗ = 𝟑𝒊̂ + 𝑱̂ are such that 𝒂 ⃗⃗ is
⃗⃗ + 𝝀𝒃
perpendicular to 𝒄 ⃗⃗ then find the value of 𝝀

SOL: Given 𝒂 ̂
⃗⃗ = 𝟐𝒊̂ + 𝟐𝑱̂ + 𝟑𝒌 ⃗𝒃⃗ = −𝒊̂ + 𝟐𝑱̂ + 𝒌
̂ ⃗⃗ = 𝟑𝒊̂ + ̂𝑱
𝒄

⃗⃗ = 𝟐𝒊̂ + 𝟐𝑱̂ + 𝟑𝒌
⃗⃗ + 𝝀𝒃
𝒂 ̂ + 𝝀(−𝒊̂ + 𝟐𝑱̂ + 𝒌
̂) = (𝟐 − 𝝀)𝒊̂ + (𝟐 + 𝟐𝝀)𝑱̂ + (𝟑 + 𝝀)𝒌
̂

⃗⃗ is perpendicular to 𝒄
⃗⃗ + 𝝀𝒃
𝒂 ⃗⃗, (𝒂 ⃗⃗ ) . 𝒄
⃗⃗ + 𝝀𝒃 ⃗⃗ = 𝟎

(𝟑)(𝟐 − 𝝀) + (𝟏)(𝟐 + 𝟐𝝀) + (𝟎)(𝟑 + 𝝀) = 𝟎 𝟔 − 𝟑𝝀 + 𝟐 + 𝟐𝝀 = 𝟎 𝝀=𝟖

(17). Find the unit vector perpendicular to vector 𝒂 ̂ and


⃗⃗ = 𝒊̂ − 𝟕𝑱̂ + 𝟕𝒌 ⃗𝒃⃗ = 𝟑𝒊̂ −
̂
𝟐𝑱̂ + 𝟐𝒌

SOL: Given 𝒂 ̂
⃗⃗ = 𝒊̂ − 𝟕𝑱̂ + 𝟕𝒌 ⃗𝒃⃗ = 𝟑𝒊̂ − 𝟐𝑱̂ + 𝟐𝒌
̂

⃗⃗ 𝑿 ⃗𝒃⃗
𝒂
Unit vector perpendicular to ⃗⃗ 𝒂𝒏𝒅 ⃗𝒃⃗ = 𝒏
𝒂 ̂= ⃗⃗|
⃗⃗ 𝑿 𝒃
|𝒂
77

𝒊̂ 𝒋̂ ̂
𝒌 𝒊̂ 𝒋̂ ̂
𝒌
⃗⃗ 𝐗 ⃗𝒃⃗ = |𝒂𝟏 𝒂𝟐 𝒂𝟑 | = |𝟏 −𝟕 𝟕 |
𝒂
𝒃𝟏 𝒃𝟐 𝒃𝟑 𝟑 −𝟐 𝟐
. = 𝒊̂(−𝟏𝟒 + 𝟏𝟒) − 𝑱̂ (𝟐 − 𝟐𝟏) + 𝒌̂ (−𝟐 + 𝟐𝟏) = 𝟏𝟗𝑱̂ − 𝟏𝟗𝒌
̂

⃗⃗ | = √(𝟏𝟗)𝟐 + (𝟏𝟗)𝟐 = 𝟏𝟗√𝟐


⃗⃗ 𝐗 𝒃
|𝒂

⃗𝒂⃗ 𝑿 ⃗𝒃⃗ ̂
𝟏𝟗𝑱̂ −𝟏𝟗𝒌
Unit vector perpendicular to ⃗𝒂⃗ 𝒂𝒏𝒅 𝒃 = 𝒏
⃗⃗ ̂= =
|𝒂⃗⃗ 𝑿 ⃗𝒃⃗| 𝟏𝟗√𝟐

⃗⃗⃗⃗ − ⃗⃗⃗
(18). Show that ( 𝒂 𝒃) ⃗⃗⃗⃗ + ⃗⃗⃗
𝑿 (𝒂 ⃗⃗⃗⃗ 𝐗 ⃗⃗⃗
𝒃)=𝟐 (𝒂 𝒃)

⃗⃗⃗ − ⃗⃗⃗
SOL: LHS = ( ⃗𝒂 𝒃) ⃗⃗⃗ + ⃗⃗⃗
𝑿 ( ⃗𝒂 𝒃) ⃗⃗ 𝐗 ⃗𝒃⃗ = − ⃗𝒃⃗ 𝐗 𝒂
Note: 𝒂 ⃗⃗

=(𝒂
⃗⃗⃗⃗ 𝐗 𝒂 ⃗⃗⃗⃗ 𝐗 ⃗⃗⃗
⃗⃗⃗⃗ ) + (𝒂 𝒃) − (⃗⃗⃗
𝒃𝐗𝒂 ⃗⃗⃗⃗) − (⃗⃗⃗
𝒃 𝐗 ⃗⃗⃗ ⃗⃗⃗⃗ 𝐗 ⃗⃗⃗
𝒃 ) = 𝟎 + (𝒂 ⃗⃗⃗⃗ 𝐗 ⃗⃗⃗
𝒃) + ( 𝒂 𝒃)−𝟎

⃗⃗ 𝐗 ⃗𝒃⃗ )
=𝟐(𝒂

MCQ - VECTOR ALGEBRA


1. The value of 𝑖̂. (𝑗̂ × 𝑘̂ ) + 𝑗̂. (𝑖̂ × 𝑘̂) + 𝑘̂. (𝑖̂ × 𝑗̂) is
( A) 0 (B) −1 (C) 1 (D) 3

⃗⃗ be such that |𝑎⃗| = 3 and |𝑏⃗⃗| = √2


⃗⃗ and 𝒃
2. Let the vectors 𝒂 , then 𝑎⃗ × 𝑏⃗⃗ is a unit vector , if the
3
𝜋 𝜋 𝜋 𝜋
angle between 𝑎⃗ and 𝑏⃗⃗ is ( A) (B) (C) 3 (D) 2
6 4
̂, |𝑏⃗⃗| = 5 and the angle between 𝒂
⃗⃗ = 𝒊̂ + 𝟐𝒋̂ + 𝟐𝒌
3. . If 𝒂 ⃗⃗ is 𝝅 , then the area of the triangle
⃗⃗ and 𝒃
𝟔

formed by these two vectors as two sides is


15√3 15 15
( A) 15 sq. units (B) sq. units (C) sq. units (D) sq. units
2 2 4

4. If 𝑎⃗ and ⃗⃗⃗⃗
𝑏 are two collinear vectors, then which of the following is correct:
( A) ⃗⃗⃗⃗
𝑏 = λ𝒂
⃗⃗ , for some scalar λ (B) 𝒂 ⃗⃗⃗⃗ (C) the respective components of 𝑎⃗ and ⃗⃗⃗⃗
⃗⃗ =±𝑏 𝑏 are not
proportional (D) both the vectors 𝑎⃗ and ⃗⃗⃗⃗
𝑏 have same direction, but different magnitudes.
5. If 𝑎⃗ is a nonzero vector of magnitude ‘a’ and λ a nonzero scalar, then λ is unit vector if
(A) λ = 1 (B) λ = – 1 (C) a = |λ| (D) a = 1/|λ|
√2
6. Let 𝑎⃗ and ⃗⃗⃗⃗
𝑏 the vectors be such that|𝑎⃗| = 3 and |𝑏⃗⃗| = 3 , then 𝒂
⃗⃗ × ⃗𝒃⃗ is a unit vector, if the angle

⃗⃗⃗⃗ is
between 𝑎⃗ and 𝑏 (A) π/6 (B) π/4 (C) π/3 (D) π/2
78

1 1
7. Area of a rectangle having vertices A, B, C and D with position vectors −𝑖̂ + 𝑗̂ + 4 𝑘̂, 𝑖̂ + 𝑗̂ +
2 2
1 1
4 𝑘̂ , 𝑖̂ − 2 𝑗̂ + 4 𝑘̂ , and −𝑖̂ − 2 𝑗̂ + 4 𝑘̂ , respectively is
1
(A) 2 (B) 1 (C) 2 (D) 4

8. Let 𝑎⃗ and ⃗⃗⃗⃗


𝑏 be two unit vectors and θ is the angle between them. Then ( 𝑎⃗ + 𝑏⃗⃗)is a unit vector if
𝜋 𝜋 𝜋 2𝜋
(A) θ = 4 (B) θ = 3 (C) θ = 2 (D) θ = 3

9. If θ is the angle between any two vectors 𝑎⃗ and ⃗⃗⃗⃗ ⃗⃗ . ⃗𝒃⃗| = |𝒂


𝑏 , then |𝒂 ⃗⃗ × ⃗𝒃⃗| , when θ is equal to
𝜋 𝜋
(A) θ = 0 (B) θ = 4 (C) θ = 2 (D) θ = 𝜋

10. The value of λ for which the vectors 3𝑖̂ − 6 𝑗̂ + 𝑘̂ and 2𝑖̂ − 4 𝑗̂ + 𝛌𝑘̂ are parallel , is
2 3 5 2
(A) 3 (B) 2 (C) (D)
2 5

SHORT ANSWER TYPE QUESTIONS – VECTIRS - [ 2 MARKS]

1. Find the projection of the vector 𝑖̂ + 3𝑗̂ + 7𝑘̂ on 7 𝑖̂ − 𝑗̂ + 8𝑘̂.

2. Find a unit vector in the direction of the vector 𝑎


⃗⃗⃗⃗ = 𝑖̂ + 𝑗̂ + 2𝑘̂.
3. Find a vector in the direction of the vector 𝑎
⃗⃗⃗⃗ = 𝑖̂ − 2𝑗̂ that has magnitude 7 units.

4. Find the value of  so that the vectors 𝑎


⃗⃗⃗⃗ = 2𝑖̂ + 𝑗̂ + 𝑘̂ and ⃗⃗⃗⃗
𝑏 = 𝑖̂ − 2𝑗̂ + 3𝑘̂ are perpendicular
to each other .

5. Find the angle between the vectors 𝑖̂ − 2𝑗̂ + 3𝑘̂ and 3 𝑖̂ − 2𝑗̂ + 𝑘̂ .

6. If 𝑎
⃗⃗⃗⃗ = √3 , 𝑏⃗⃗ = 2 and angle between 𝑎 ⃗⃗⃗⃗ is 600 , then find 𝑎
⃗⃗⃗⃗ and 𝑏 ⃗⃗⃗⃗.
⃗⃗⃗⃗. 𝑏

7. Find the value of  for which 𝑎 𝑏 = 𝑖̂ + 𝑗̂ + 3𝑘̂ are parallel vectors.
⃗⃗⃗⃗ = 3𝑖̂ + 2𝑗̂ + 9𝑘̂ and ⃗⃗⃗⃗
8. Find  if (2𝑖̂ + 6𝑗̂ + 14𝑘̂) × (𝑖̂ − 𝑗̂ + 7𝑘̂)=0
⃗⃗⃗⃗
√2
9. Let 𝑎 𝑏 be two vectors such that 𝑎
⃗⃗⃗⃗ and ⃗⃗⃗⃗ ⃗⃗⃗⃗ = 3 , 𝑏⃗⃗ = 3 and 𝑎
⃗⃗⃗⃗ × ⃗⃗⃗⃗
𝑏 is a unit vector. What

is angle between 𝑎 ⃗⃗⃗⃗?


⃗⃗⃗⃗ and 𝑏
10. Write the value of (𝑖̂ × 𝑗̂). 𝑘̂ + 𝑖̂. 𝑗̂ .
1 4 1
⃗⃗⃗⃗ and ⃗⃗⃗⃗
11. If vectors 𝑎 ⃗⃗⃗⃗|=2, | ⃗⃗⃗⃗
𝑏 are such that | 𝑎 𝑏 |= ⃗⃗⃗⃗ × ⃗⃗⃗⃗
and |𝑎 ⃗⃗⃗⃗ . ⃗⃗⃗⃗
𝑏 | = ,then find |𝑎 𝑏 |.
√3 √3

12. Find  when the projection of 𝑎


⃗⃗⃗⃗ = 𝑖̂ + 𝑗̂ + 4𝑘̂ on ⃗⃗⃗⃗
𝑏 = 2𝑖̂ + 6𝑗̂ + 3𝑘̂ is 4units .[CBSE 2012]

13. For what value of p , p(𝑖̂ + 𝑗̂ + 𝑘̂) is unit vector.

14. Find 𝑥
⃗⃗⃗⃗ ,if for a unit vector 𝑎
⃗⃗⃗⃗, (𝑥 ⃗⃗⃗⃗). (𝑥
⃗⃗⃗⃗ − 𝑎 ⃗⃗⃗⃗ − 𝑎
⃗⃗⃗⃗)=18
79

⃗⃗⃗⃗ = 2𝑖̂ − 𝑗̂ + 2𝑘̂ and ⃗⃗⃗⃗


15. Find unit vector in the direction of the sum of the vectors 𝑎 𝑏 = −𝑖̂ + 𝑗̂ +
3𝑘̂

16. Find the area of parallelogram whose adjacent sides are 𝑖̂ + 𝑘̂ and 2𝑖̂ + 𝑗̂ + 𝑘̂.

⃗⃗⃗⃗ + ⃗⃗⃗⃗
17. Find unit vector perpendicular to each of the vectors 𝑎 𝑏 and 𝑎⃗ − ⃗⃗⃗⃗
𝑏 , where 𝑎
⃗⃗⃗⃗ = 3𝑖̂ +
2𝑗̂ + 2𝑘̂ and 𝑏
⃗⃗⃗⃗= 𝑖̂ + 2𝑗̂ − 2𝑘̂ .

18. If 𝑎 ⃗⃗⃗⃗ = 5𝑖̂ − 𝑗̂ + 𝑘̂, then find the value of  so that 𝑎


⃗⃗⃗⃗ = 𝑖̂ − 𝑗̂ + 7𝑘̂ and 𝑏 ⃗⃗⃗⃗ and 𝑎⃗ − 𝑏
⃗⃗⃗⃗ + 𝑏 ⃗⃗⃗⃗ are
perpendicular vectors.
19. If 𝑟⃗ =𝑥𝑖̂ + 𝑦𝑗̂ + 𝑧𝑘̂, find (𝑟⃗ × 𝑖̂). (𝑟⃗ × 𝑗̂)+xy
⃗⃗⃗⃗=𝑖̂ + 𝑗̂ + 𝑘̂ , ⃗⃗⃗⃗
20. Let 𝑎 ̂ + 3𝑘̂ and 𝑐⃗⃗⃗=𝑖̂ − 2𝑗̂ + 𝑘̂ , then find a vector of magnitude 6 units
𝑏 =4𝑖̂ − 2𝑗
⃗⃗⃗⃗ − ⃗⃗⃗⃗
which is parallel to the vector 2𝑎 𝑏 + 3𝑐⃗⃗⃗.

21. Find the value of p if vectors 3 𝑖̂ − 2𝑗̂ + 3𝑘̂ and p𝑖̂ − 4𝑗̂ + 8𝑘̂ are orthogonal

LONG ANSWER TYPE QUESTIONS – VECTORS - [ 5 MARKS]

⃗⃗⃗⃗=𝑖̂ + 4 𝑗̂ + 2𝑘̂ , 𝑏
1. . Let 𝑎 ⃗⃗⃗⃗=3𝑖̂ − 2𝑗̂ + 7𝑘̂and 𝑐⃗⃗⃗=2𝑖̂ − 𝑗̂ + 4𝑘̂ .Find a vector 𝑑
⃗⃗⃗⃗ which is

perpendicular to both 𝑎 ⃗⃗⃗⃗,and 𝑐⃗⃗⃗. 𝑑


⃗⃗⃗⃗𝑎𝑛𝑑 𝑏 ⃗⃗⃗⃗ = 15.

2. If 𝑎 ⃗⃗⃗⃗ and 𝑐⃗⃗⃗ are three unit vectors such that 𝑎


⃗⃗⃗⃗, 𝑏 ⃗⃗⃗⃗ + 𝑐⃗⃗⃗ = 0
⃗⃗⃗⃗ + 𝑏 ⃗⃗⃗⃗, then find 𝑎 ⃗⃗⃗⃗ + 𝑏
⃗⃗⃗⃗ . 𝑏 ⃗⃗⃗⃗ . 𝑐⃗⃗⃗ + 𝑐⃗⃗⃗ . 𝑎
⃗⃗⃗⃗.
3. Using vectors ,find the area of a triangle having the points A (1,1,2),B(2,3,5)and C(1,5,5) as
its vertices.
4. Using vectors ,find the area of a parallelogram whose diagonals are the vectors 𝑎
⃗⃗⃗⃗=3𝑖̂ + 𝑗̂ +
4𝑘̂ and ⃗⃗⃗⃗
𝑏 =𝑖̂ − 𝑗̂ + 𝑘̂ .
⃗⃗⃗⃗ = 3𝑖̂ + 4𝑗̂ + 5𝑘̂, and ⃗⃗⃗ = 2𝑖̂ + 𝑗̂ − 4𝑘̂, then express ⃗⃗⃗ in the form ⃗⃗⃗ = ⃗⃗⃗⃗⃗⃗
5. If For 𝛼 1 + ⃗⃗⃗⃗⃗⃗
2 ,

1 is parallel to 𝛼
where ⃗⃗⃗⃗⃗⃗ 2 is perpendicular to 𝛼
⃗⃗⃗⃗ and ⃗⃗⃗⃗⃗⃗ ⃗⃗⃗⃗.

6. If 𝑎 ⃗⃗⃗⃗ and 𝑐⃗⃗⃗ are three mutually perpendicular vectors of equal magnitude, then prove that
⃗⃗⃗⃗, 𝑏
⃗⃗⃗⃗ + 𝑐⃗⃗⃗ is equally inclined to the vectors 𝑎
⃗⃗⃗⃗ + 𝑏
𝑎 ⃗⃗⃗⃗ and 𝑐⃗⃗⃗.
⃗⃗⃗⃗, 𝑏
7. If 𝑎 ⃗⃗⃗⃗ and 𝑐⃗⃗⃗ be three vectors such that 𝑎
⃗⃗⃗⃗, 𝑏 ⃗⃗⃗⃗= 3, 𝑏
⃗⃗⃗⃗= 4 and 𝑐⃗⃗⃗= 5 and each of them being
perpendicular to the sum of other two, then find 𝑎 ⃗⃗⃗⃗ + 𝑐⃗⃗⃗.
⃗⃗⃗⃗ + 𝑏
8. Find 𝑎 ⃗⃗⃗⃗, if two vectors 𝑎
⃗⃗⃗⃗ − 𝑏 ⃗⃗⃗⃗ are such that 𝑎
⃗⃗⃗⃗ and 𝑏 ⃗⃗⃗⃗= 2, 𝑏
⃗⃗⃗⃗= 3 and 𝑎 ⃗⃗⃗⃗ = 4.
⃗⃗⃗⃗ . 𝑏
9. Find the area of a triangle having the points A (1,1,1),B(1,2,3)and C(2,3,1) as its vertices.
10. Find the area of a parallelogram whose adjacent sides are given by the vectors 𝑎 ̂ + 𝑗̂ +
⃗⃗⃗⃗=3𝑖
4𝑘̂ and ⃗⃗⃗⃗
𝑏 =𝑖̂ − 𝑗̂ + 𝑘̂
80

𝜋
ANSWERS OF MCQ - VECTOR ALGEBRA : 1.(C ) 1 2. (B) 4
15
3. (D) sq. units ⃗⃗⃗⃗ = λ𝒂
4. ( A) 𝑏 ⃗⃗ , for some scalar λ 5. (D) a = 1/|λ| 6. (B) π/4
4
2𝜋 𝜋 2
7. (C) 2 8. (D) θ = 9. (B) θ = 4 10. (A) 3
3

ANSWERS – VECTORS - SHORT ANSWER TYPE QUESTIONS [ 2 MARKS]

1 1 2 7 14 5
1.
60
2. 𝑖̂ + 𝑗̂ + 𝑘̂ 3. 𝑖̂ − 𝑗̂ 4.  =
√114 √6 √6 √6 √5 √5 2
5 3
5. 𝜃 = cos−1 ( ) 6. √3 7.  = 8.  = −3
7 2
1
9. 450 10. 1 11. 1 12.  = 5 13. p=±
√3
1 5 2 2
14. 𝑥
⃗⃗⃗⃗ = 4 15. 𝑖̂ + 𝑘̂ 16. √3 sq. unit 17. 𝑖̂ − 𝑗̂ −
√26 √26 3 3
1 32
𝑘̂ 18.  = ±5 19. 0 20. 2𝑖̂ − 4𝑗̂ + 4𝑘̂ 21. p=−
3 3

ANSWERS - Vectors - LONG ANSWER TYPE QUESTIONS [ 5 MARKS]

1 −3 √61 1
(160𝑖̂ − 5𝑗̂ + 70𝑘̂ ) 2. 3. sq. units 4. √42 sq.units
3 2 2 2
−3 4 13 9
5. The required expression is : 2𝑖̂ + 𝑗̂ − 4𝑘̂=( 5 𝑖̂ − 5 𝑗̂ − 𝑘̂) + ( 5
𝑖̂ + 𝑗̂ − 3𝑘̂)
5
√21
8. √5 9. 2
10. √42

Chapter 11 – THREE - DIMENSIONAL COORDINATE GEOMETRY


Direction cosines and direction ratios of a line joining two points. Cartesian equation and
vector equation of a line, shortest distance between two lines. Angle between two lines.

Formulae/Important Points
• Distance between two given points P (x1, y1, z1) and Q (x2, y2, z2) is

|𝑃𝑄| = √(𝑥2 − 𝑥1 )2 + (𝑦2 − 𝑦1 )2 + (𝑧2 − 𝑧1 )2

• Direction ratio of line joining the points (x1, y1, z1) and (x2, y2, z2) are
𝑥2 − 𝑥1 , 𝑦2 − 𝑦1 , 𝑧2 − 𝑧1
• Let a, b, c be the direction ratio of a line whose direction cosines are l, m, n
𝑙 𝑚 𝑛
then = = and 𝑙 2 + 𝑚2 + 𝑛2 = 1 ;
𝑎 𝑏 𝑐
𝑎 𝑏 𝑐
𝑙 = ± √𝑎2 ; 𝑚 = ± √𝑎2 ; 𝑛 = ± √𝑎2
+𝑏 2 +𝑐 2 +𝑏2 +𝑐 2 +𝑏 2 +𝑐 2
81

• If a line makes angles 𝛼, 𝛽, 𝛾, with positive direction o f X , Y a n d Z - axes then


𝑙 = cos 𝛼 ; 𝑚 = 𝑐𝑜𝑠𝛽 ; 𝑛 = cos 𝛾 and 𝑐𝑜𝑠 2 𝛼 + 𝑐𝑜𝑠 2 𝛽 + 𝑐𝑜𝑠 2 𝛾 = 1
• Vector equation of a straight line passing through a fixed point with the position
vector 𝑎⃗ and parallel to given vector 𝑚
⃗⃗⃗ is 𝑟⃗ = 𝑎⃗ + 𝜆𝑚
⃗⃗⃗
• Cartesian equation of a line passing through point (x1, y1, z1) having direction
𝑥−𝑥1 𝑦−𝑦1 𝑧−𝑧1
ratio<a,b,c> is = = .
𝑎 𝑏 𝑐

• Cartesian equation of a line passing through two points (x1, y1, z1) and (x2, y2, z2) is
𝑥 − 𝑥1 𝑦 − 𝑦1 𝑧 − 𝑧1
= =
𝑥2 − 𝑥1 𝑦2 − 𝑦1 𝑧2 − 𝑧1
• Vector equation of a straight line passing through two points with position vectors 𝑎⃗
and 𝑏⃗⃗ is 𝑟⃗ = 𝑎⃗ + 𝜆(𝑏⃗⃗ − 𝑎⃗)
𝑥−𝑥1 𝑦−𝑦1 𝑧−𝑧1
• Coordinate of any point on the line = = is
𝑎 𝑏 𝑐

(𝑥1 + 𝜆𝑎, 𝑦1 + 𝜆𝑏, 𝑧1 + 𝜆𝑐), 𝜆 ∈ 𝑅


• Angle between two lines whose direction ratios are < 𝑎1 , 𝑏1 , 𝑐1 > and < 𝑎2 , 𝑏2 , 𝑐2 > is
𝑎1 𝑎2 +𝑏1 𝑏2 +𝑐1 𝑐2
𝑐𝑜𝑠𝜃 = If lines are perpendicular then 𝑎1 𝑎2 + 𝑏1 𝑏2 + 𝑐1 𝑐2 = 0
√𝑎12 +𝑏12 +𝑐12 √𝑎22 +𝑏22 +𝑐22

𝑎 𝑏 𝑐
If lines are parallel then 𝑎1 = 𝑏1 = 𝑐1
2 2 2

⃗⃗⃗⃗⃗
𝑏 .𝑏 ⃗⃗⃗⃗⃗
• 𝑎1 + 𝜆 ⃗⃗⃗⃗
Angle between two lines 𝑟⃗ = ⃗⃗⃗⃗⃗ 𝑎2 + 𝜇 ⃗⃗⃗⃗⃗
𝑏1 and 𝑟⃗ = ⃗⃗⃗⃗⃗ 𝑏2 is 𝑐𝑜𝑠𝜃 = |𝑏⃗⃗⃗⃗⃗1 ||𝑏⃗⃗⃗⃗⃗2|
1 2

If lines are perpendicular then ⃗⃗⃗⃗


𝑏1 . ⃗⃗⃗⃗
𝑏1 = 0
If lines are parallel then ⃗⃗⃗⃗ ⃗⃗⃗⃗1 , 𝑡 ∈ 𝑅
𝑏1 = 𝑡𝑏

• Skew lines : Lines in space, which are neither parallel, nor intersecting are called skew
lines,such pair of lines are non-coplanar.
• 𝑎1 + 𝜆 ⃗⃗⃗⃗
Shortest distance between skew lines : Shortest distance between skew lines 𝑟⃗ = ⃗⃗⃗⃗⃗ 𝑏1
and
|(𝑎
⃗⃗⃗⃗⃗⃗−𝑎 1 ⃗⃗⃗⃗⃗
2 ⃗⃗⃗⃗⃗⃗).(𝑏 ⃗⃗⃗⃗⃗
1 ×𝑏 2 )|
• 𝑎2 + 𝜇 ⃗⃗⃗⃗⃗
𝑟⃗ = ⃗⃗⃗⃗⃗ 𝑏2 is 𝑑 = ⃗⃗⃗⃗⃗
|𝑏 ⃗⃗⃗⃗⃗
.
1 ×𝑏 2|

• If shortest distance between skew lines is zero then lines are said to be intersecting or
coplanar lines.
• 𝑎1 + 𝜆 𝑏⃗⃗ and
Distance between two parallel lines : Distance between two parallel lines 𝑟⃗ = ⃗⃗⃗⃗⃗
|(𝑎
⃗⃗⃗⃗⃗⃗−𝑎 ⃗⃗|
⃗⃗⃗⃗⃗⃗)×𝑏
𝑎2 + 𝜇 ⃗⃗⃗⃗⃗
𝑟⃗ = ⃗⃗⃗⃗⃗ 𝑏2 is d then 𝑑 = 2 1 ⃗⃗|
|𝑏
MULTIPLE CHOICE QUESTIONS
1.The coordinates of foot of perpendicular drawn from the point (2, −3,4) on the y-axis is.
(a) (2,3,4) (b) (−2, −3, −4) (c) (0, −3,0) (d) (2,0,4)
2.If a line makes angles 𝛼, 𝛽 𝑎𝑛𝑑 𝛾 with the axes the cos 2𝛼 + cos 2𝛽 + cos 2 𝛾 =
(a) −2 (b) −1 (c) 1 (d) 2
82

3.Distance of point (a,b,c) from y-axis is


(a) √𝑎2 + 𝑏 2 (b) √𝑏 2 + 𝑐 2 (c) √𝑎2 + 𝑐 2 (d) √𝑎2 + 𝑏 2 + 𝑐 2
4.If direction cosines of a line are 𝑘, 𝑘, 𝑘 then
1 1
(a) 𝑘 > 0 (b) 0 < 𝑘 < 1 (c) 𝑘 = 1 (d) 𝑘 = 𝑜𝑟 −
√3 √3
𝜋 𝜋
5.If a line makes angles 3 and with x-axis and y-axis respectively, then the angle made by the line
4
with z-axis is
𝜋 𝜋 𝜋 15𝜋
(a) 2 (b) 3 (c) 4 (d) 12
6. P is a point on the line segment joining the points (3,2, −1) and (6,2, −2) if x-coordinate of P
is 5,then its y coordinate is
(a) −2 (b) −1 (c) 1 (d) 2
7.The distance of point (1,6,3) to the line 𝑟⃗ = (𝑗̂ + 2𝑘̂) + 𝜆(𝑖̂ + 2𝑗̂ + 3𝑘̂) is
(a) √13 (b)13 (c)2√13 (d) none of these
𝑥−3 𝑦−2 𝑧−1
8. The straight line = = is
3 1 0
(a) parallel to x-axis (b) parallel to y-axis (c) parallel to z-axis (d) perpendicular to z-axis
𝑦 𝑧 𝑦 𝑧
9. If lines 𝐿1 : 𝑥 = 5, = −2; and 𝐿2 : 𝑥 = 2, = 2−𝛼 are perpendicular then value of 𝛼 𝑖𝑠
3−𝛼 −1
2
(a) 3 (b) 3 (c) 4 (d) 7/3
𝑥 𝑦 𝑧 𝑥−1 𝑦−2 𝑧−3
10. The lines 1 = = 3 and = = are
2 −2 −4 −6
(a) coincident (b) skew (c)intersecting (d) parallel
𝑥+1 𝑦−2 𝑧+3 𝑥−1 𝑦+2 𝑧−3
11. The angle between the straight lines = = and = = is
2 5 4 1 2 −3
(a)450 (b) 300 (c) 60 0
(d) 90 0

cos 2  + cos 2  + cos 2 


12. If  is an angle given by cos  = where  ,  ,  are the angles made by a
sin2  + sin2  + sin2 

line with the positive directions of the axes of reference then the measure of  is:-

   
(a) (b) (c) (d)
4 6 2 3

13. The direction ratios of two parallel lines are 4, − 3, − 1 and  + , 1 + , 2 . The value of the pair
( ,  ) is:-
(a) (1, 7) (b) (- 1, - 7) (c) (7, 1) (d) no fixed value
14. If A = (1, 2, 3), B = (− 2, 4,  ) and AOB =  / 2 where O is the origin then  is:-
(a) 6 (b) – 6 (c) 0 (d) – 2
15. ABC is a triangle where A = (2, 3, 5), B = (− 1, 3, 2) and C = ( , 5,  ) . If the median through A is
equally inclined with the axes then:-
7
(a)  = 14,  = 20 (b)  = 7,  = 10 (c)  = ,  = 5 (d)  = 10,  = 7
2
83

11.2-(15). Find the shortest distance between the lines

𝒙+𝟏 𝒚+𝟏 𝒛+𝟏 𝒙−𝟑 𝒚−𝟓 𝒛−𝟕


= = and = =
𝟕 −𝟔 𝟏 𝟏 −𝟐 𝟏
𝒙+𝟏 𝒚+𝟏 𝒛+𝟏 𝒙−𝟑 𝒚−𝟓 𝒛−𝟕
SOL: Given lines: = = and = =
𝟕 −𝟔 𝟏 𝟏 −𝟐 𝟏

|(⃗⃗⃗⃗⃗ 𝒂𝟏 ) . (𝒃
𝒂𝟐 – ⃗⃗⃗⃗⃗ ⃗⃗⃗⃗⃗ ⃗⃗⃗⃗⃗
𝟏 𝑿 𝒃𝟐 )|
Shortest Distance between the lines =
⃗⃗⃗⃗⃗
|𝒃 ⃗⃗⃗⃗⃗
𝟏 𝑿 𝒃𝟐 |

⃗⃗⃗⃗⃗𝟏 = −𝒊̂ − 𝑱̂ − 𝒌̂
𝒂 ⃗⃗⃗⃗⃗𝟐 = 𝟑𝒊̂ + 𝟓𝑱̂ + 𝟕𝒌̂
𝒂 ⃗⃗⃗⃗⃗𝟏 = 𝟒𝒊̂ + 𝟔𝑱̂ + 𝟖𝒌̂
⃗⃗⃗⃗⃗𝟐 − 𝒂
𝒂

⃗⃗⃗⃗⃗
𝒃𝟏 = 𝟕𝒊̂ − 𝟔𝑱̂ + 𝒌̂ ⃗⃗⃗⃗⃗
𝒃𝟐 = 𝒊̂ − 𝟐𝑱̂ + 𝒌̂

𝒊̂ 𝑱̂ ̂
𝒌
⃗⃗⃗⃗⃗⃗⃗
𝒃𝟏 𝑿 ⃗⃗⃗⃗⃗⃗⃗
𝒃𝟐 = |𝟕 −𝟔 𝟏| = 𝒊̂(−𝟔 + 𝟐) − 𝑱̂(𝟕 − 𝟏) + 𝒌̂ (−𝟏𝟒 + 𝟔) = −𝟒𝒊̂ − 𝟔𝑱̂ − 𝟖𝒌̂
𝟏 −𝟐 𝟏

|⃗⃗⃗⃗⃗⃗⃗
𝒃𝟏 𝑿 ⃗⃗⃗⃗⃗⃗⃗
𝒃𝟐| = √𝟏𝟔 + 𝟑𝟔 + 𝟔𝟒 = √𝟏𝟏𝟔

|(⃗⃗⃗⃗⃗ 𝒂𝟏 ) . (𝒃
𝒂𝟐 – ⃗⃗⃗⃗⃗ ⃗⃗⃗⃗⃗ ⃗⃗⃗⃗⃗
𝟏 𝑿 𝒃𝟐 )| (𝟒𝒊̂ + 𝟔𝑱̂ + 𝟖𝒌̂ ).(−𝟒𝒊̂ − 𝟔𝑱̂ − 𝟖𝒌̂ )
SD =
⃗⃗⃗⃗⃗ ⃗⃗⃗⃗⃗
=| |
|𝒃 𝟏 𝑿 𝒃𝟐 | √𝟏𝟏𝟔

(𝟒)(−𝟒)+(𝟔)(−𝟔)+(𝟖)(−𝟖) −𝟏𝟔−𝟑𝟔−𝟔𝟒 𝟏𝟏𝟔


SD = | |=| |= = √𝟏𝟏𝟔 = 𝟐√𝟐𝟗
√𝟏𝟏𝟔 √𝟏𝟏𝟔 √𝟏𝟏𝟔

11.2-(17). Find the shortest distance between the lines whose vector
equations are

⃗⃗⃗
𝒓 = (𝟏 − 𝒕)𝒊̂ + (𝟐 − 𝒕)𝑱̂ + (𝟑 − 𝟐𝒕)𝒌̂ & ⃗⃗⃗
𝒓 = (𝒔 + 𝟏)𝒊̂ + (𝟐𝒔 − 𝟏)𝑱̂ − (𝟐𝒔 + 𝟏)𝒌̂
SOL: The equations of the given lines are

⃗⃗⃗ = (𝟏 − 𝒕)𝒊̂ + (𝟐 − 𝒕)𝑱̂ + (𝟑 − 𝟐𝒕)𝒌̂


𝒓 & ⃗⃗⃗ = (𝒔 + 𝟏)𝒊̂ + (𝟐𝒔 − 𝟏)𝑱̂ − (𝟐𝒔 + 𝟏)𝒌̂
𝒓

⃗⃗⃗ = 𝒊̂ − 𝟐𝑱̂ + 𝟑𝒌̂


𝒓 ̂)
+ 𝒕(− 𝒊̂ + 𝑱̂ − 𝟐𝒌 & ⃗⃗⃗ = 𝒊̂ − 𝑱̂ − 𝒌̂
𝒓 ̂)
+ 𝒔( 𝒊̂ + 𝟐𝑱̂ − 𝟐𝒌

⃗⃗⃗⃗⃗⃗
|(𝒂 𝟐 – ⃗⃗⃗⃗⃗⃗)
𝒂𝟏 . (𝒃 ⃗⃗⃗⃗⃗ ⃗⃗⃗⃗⃗
𝟏 𝑿 𝒃𝟐 )|
Shortest Distance between the lines =
|𝒃⃗⃗⃗⃗⃗ ⃗⃗⃗⃗⃗
𝟏 𝑿 𝒃𝟐 |

⃗⃗⃗⃗⃗𝟏 = 𝒊̂ − 𝟐𝑱̂ + 𝟑𝒌̂


𝒂 ⃗⃗⃗⃗⃗𝟐 = 𝒊̂ − 𝑱̂ − 𝒌̂
𝒂 ⃗⃗⃗⃗⃗𝟏 = 𝟎𝒊̂ + 𝑱̂ − 𝟒𝒌̂
⃗⃗⃗⃗⃗𝟐 − 𝒂
𝒂
84

⃗⃗⃗⃗⃗
𝒃𝟏 = −𝒊̂ + 𝑱̂ − 𝟐𝒌̂ ⃗⃗⃗⃗⃗
𝒃𝟐 = 𝒊̂ + 𝟐𝑱̂ − 𝟐𝒌̂

𝒊̂ 𝑱̂ ̂
𝒌
⃗⃗⃗⃗⃗⃗⃗ ⃗⃗⃗⃗⃗⃗⃗
𝒃𝟏 𝑿 𝒃𝟐 = |−𝟏 𝟏 −𝟐| = 𝒊̂(−𝟐 + 𝟒) − 𝑱̂(𝟐 + 𝟐) + 𝒌̂ (−𝟐 − 𝟏) = 𝟐𝒊̂ − 𝟒𝑱̂ − 𝟑𝒌̂
𝟏 𝟐 −𝟐

|⃗⃗⃗⃗⃗⃗⃗
𝒃𝟏 𝑿 ⃗⃗⃗⃗⃗⃗⃗
𝒃𝟐| = √𝟒 + 𝟏𝟔 + 𝟗 = √𝟐𝟗

|(⃗⃗⃗⃗⃗ 𝒂𝟏 ) . (𝒃
𝒂𝟐 – ⃗⃗⃗⃗⃗ ⃗⃗⃗⃗⃗ ⃗⃗⃗⃗⃗
𝟏 𝑿 𝒃𝟐 )|
̂) .(𝟐𝒊̂−𝟒𝑱̂− 𝟑𝒌
(𝟎𝒊̂+𝑱̂−𝟒𝒌 ̂)
SD =
⃗⃗⃗⃗⃗ ⃗⃗⃗⃗⃗
=| |
|𝒃 𝟏 𝑿 𝒃𝟐 | √𝟐𝟗

(𝟎)(𝟐)+(𝟏)(−𝟒)+(−𝟒)(−𝟑) −𝟒 + 𝟏𝟐 𝟖 𝟖√𝟐𝟗
SD = | |=| |= =
√𝟐𝟗 √𝟐𝟗 √𝟐𝟗 𝟐𝟗

EXTRA QUESTIONS
EX:1(a). Find the Foot of the perpendicular of (1, 2, 3) in the line

𝒙−𝟔 𝒚 −𝟕 𝒛 −𝟕
= =
𝟑 𝟐 −𝟐

EX: 1(b). Find the length of perpendicular form the point (1, 2, 3) to the
𝒙−𝟔 𝒚 −𝟕 𝒛 −𝟕
line = =
𝟑 𝟐 −𝟐

EX: 1(c). Find the Equation of the perpendicular form the point (1, 2, 3)
𝒙−𝟔 𝒚 −𝟕 𝒛 −𝟕
to the Line = =
𝟑 𝟐 −𝟐

EX: 1(d). Find the image of the point (2, 4, –1) in the

𝒙+𝟓 𝒚+𝟑 𝒛−𝟔


line = =
𝟏 𝟒 −𝟗

A (2,4,–1) )
SOL: Given point (2, 4, –1)

𝒙+𝟓 𝒚+𝟑 𝒛−𝟔


Given line = =
𝟏 𝟒 −𝟗

𝒙+𝟓 𝒚+𝟑 𝒛−𝟔


Let = = =𝒌 D
𝟏 𝟒 −𝟗

Any Point on given line D (k–5, 4k–3, –9k+6)


𝑩(𝜶, 𝜷, 𝜸)
85

DRs of AD: x2 – x1 , y2 – y1 , z2 – z1

k–5 –2, 4k–3 –4, –9k+6 +1 k–7, 4k–7, –9k+7

𝒙+𝟓 𝒚+𝟑 𝒛−𝟔


DRs of the given line = = are 1, 4, –9
𝟏 𝟒 −𝟗

AD is perpendicular to given line: a1 a2 + b1 b2 + c1 c2 = 0

1(k – 7) + 4(4k – 7) +(– 9) (–9k+7)

k – 7 + 16k – 28 + 81k – 63 = 0 98k – 98 = 0 k = 1 sub in D

Foot of the perpendicular = D (– 4, 1, – 3)

Let 𝑩(𝜶, 𝜷, 𝜸) be the image of A

Then mid-point of AB = D A (2,4,–1) )

𝜶+𝟐 𝜷+𝟒 𝜸−𝟏


( , , ) = (−𝟒, 𝟏, −𝟑)
𝟐 𝟐 𝟐

𝜶+𝟐 D
= −𝟒 𝜶 + 𝟐 = −𝟖 𝜶 = −𝟏𝟎
𝟐
𝑩(𝜶, 𝜷, 𝜸)
𝜷+𝟒
=𝟏 𝜷+𝟒=𝟐 𝜷 = −𝟐
𝟐
𝜸−𝟏
= −𝟑 𝜸 − 𝟏 = −𝟔 𝜸 = −𝟓 Image = B (−𝟏𝟎, −𝟐, −𝟓)
𝟐

EX: Show that the lines intersect and find the point of intersection.

𝒙−𝟒 𝒚+𝟑 𝒛+𝟏 𝒙−𝟑 𝒚−𝟓 𝒛−𝟕


= = and = =
𝟏 −𝟒 𝟕 𝟏 −𝟐 𝟏

𝒙−𝟒 𝒚+𝟑 𝒛+𝟏 P


SOL: Let = = = 𝒓𝟏
𝟏 −𝟒 𝟕

Any point on First line = 𝑷(𝒓𝟏 + 𝟒, −𝟒𝒓𝟏 − 𝟑, 𝟕𝒓𝟏 − 𝟏)


𝒙−𝟏 𝒚+𝟏 𝒛 + 𝟏𝟎
Let = = = 𝒓𝟐
𝟐 −𝟑 𝟖
Any point on Second line = 𝑸(𝟐𝒓𝟐 + 𝟏, −𝟑𝒓𝟐 − 𝟏, 𝟖𝒓𝟐 − 𝟏𝟎)
86

If the lines intersect P = Q

𝒓𝟏 + 𝟒 = 𝟐𝒓𝟐 + 𝟏 ; 𝒓𝟏 − 𝟐𝒓𝟐 = −𝟑 … . . (𝟏)


−𝟒𝒓𝟏 − 𝟑 = −𝟑𝒓𝟐 − 𝟏 ; −𝟒𝒓𝟏 + 𝟑𝒓𝟐 = 𝟐 … … (𝟐)
𝟕𝒓𝟏 − 𝟏 = 𝟖𝒓𝟐 − 𝟏𝟎 ; 𝟕𝒓𝟏 − 𝟖𝒓𝟐 = − 𝟗 … . . (𝟑)

𝑺𝒐𝒍𝒗𝒊𝒏𝒈 (𝟏)& (𝟐) 𝒘𝒆 𝒈𝒆𝒕, 𝒓𝟏 = 𝟏, 𝒓𝟐 = 𝟐

𝒔𝒖𝒃 𝒊𝒏 (𝟑), 𝑳𝑯𝑺 = −𝟒 + 𝟑(𝟐) = 𝟐 = 𝑹𝑯𝑺 𝑳𝑯𝑺 = 𝑹𝑯𝑺

The lines intersect

Sub 𝒓𝟏 = 𝟏 𝒊𝒏 𝑷 𝒘𝒆 𝒈𝒆𝒕 𝑷𝒐𝒊𝒏𝒕 𝒐𝒇 𝒊𝒏𝒕𝒆𝒓𝒔𝒆𝒄𝒕𝒊𝒐𝒏 𝑷(𝟓, −𝟕, 𝟔)


SHORT ANSWER TYPE QUESTIONS (2MARKS/3MARKS)

𝑥−1 3−𝑦 𝑧+1


1. Find the vector equation of the line parallel to the line 5 = −2 = 4 and passing through
( 3 , 0, -4 ) ANS; 𝑟⃗ = 3𝑖̂ – 4𝑘̂ + 𝜆( 5𝑖̂ – 2𝑗̂ + 4𝑘̂ )
𝑥−2 𝑦+1 𝑧+3
2. Find the image of the point ( 1 , -2 , 1 ) in the line 3 = −1 = 2 .

Ans ( 39 / 7 , -6 / 7 , -37 / 7 )

3. If a line makes angles α , β , γ with the positive direction of coordinate axes then find the value
of sin2 α + sin2β + sin2γ . Ans . 2
𝑥−1 𝑦−2 𝑧−3 𝑥−4 𝑦−1
4. Show that the lines = = and = = z intersect .Find their point of intersection.
2 3 4 5 2
Ans . ( -1 , -1 , -1 ).
2x −1 4 − y z + 1
5. Cartesian equation of line AB is = = . Write the direction ratios of a line
2 7 2
parallel to AB.
𝑥+4 𝑦−5 𝑧+4 𝑥−3 𝑦+7 𝑧−9
6. Find the angle between the pair of lines = = ; = =
2 7 3 4 2 4

x + 2 y +1 z − 3
7. Find the point on the line = = at a distance 3 2 from the point (1, 2, 3).
3 2 2
1 − x 7 y − 14 z − 3 7 − 7x y − 5 6 − z
= = = =
8. Find the value of p so that the lines 3 2 p 2 𝑎𝑛𝑑 3 p 1 5
are at right angles.
x −1 y − 3 z − 4
9. Find image of the point (1, –2, 3) in the line = =
3 1 −5

10. Find the vector and Cartesian equation of line which is perpendicular to the lines
87

𝑥+2 𝑦−3 𝑧+1 𝑥−1 𝑦−2 𝑧−3


= = and = = and passes through the point (1,1,1).
1 2 4 2 3 4

11. Find the value of k so that the lines 𝑥 = −𝑦 = 𝑘𝑧 𝑎𝑛𝑑 𝑥 − 2 = 2𝑦 + 1 = −𝑧 + 1 are

perpendicular to each other.


𝑥+2 2𝑦−7 5−𝑧
12. Find direction cosines of the line = = . Also find the vector equation of line
2 6 6

through the point(−1,2,3) and parallel to given line.

13. If coordinates of the points A, B, C and D are (1,2,3); (4,5,7); (−4,3, −6) and (2,9,2)

respectively then find the angle between lines AB and CD.

14. Find equation of line passing through the point (1, −1,1) and perpendicular to the lines joining

the points (4,3,2); (1, −1,0) and (1,2, −1); (2,1,1).

15. Find equation of line passing through the point (2, −1,3) and parallel to line

𝑟⃗ = 𝑖̂ – 2𝑗̂ + 𝑘̂ + 𝜆( 2𝑖̂ + 3𝑗̂ − 5𝑘̂ )

16. Find equation of line passing through the point 𝑖̂ + 𝑗̂ − 3𝑘̂ and perpendicular to the lines

𝑟⃗ = 𝑖̂ + 𝜆( 2𝑖̂ + 𝑗̂ − 3𝑘̂ ) and 𝑟⃗ = 2𝑖̂ + 𝑗̂ − 𝑘̂ + 𝜇( 𝑖̂ + 𝑗̂ + 𝑘̂ ).

−𝑥+2 𝑦−1 𝑧+3 𝑥+2 2𝑦−8 𝑧−5


17. Find angle between the lines = = and = = .
−2 7 −3 −1 4 4

𝑥−1 𝑦−2 𝑧−3 𝑥−4 𝑦−1


18. Show that the lines = = and = = 𝑧 intersect. Find their point of
2 3 4 5 2

intersection.

19. Find the equation of the perpendicular drawn from the point P (2, 4, -1) to the
𝑥+5 𝑦+3 𝑧−6
line = = .
1 4 −9

20. Find the foot of the perpendicular drawn from the point 𝑖̂ + 6𝑗̂ + 3𝑘̂ to the line

𝑟⃗ = 𝑗̂ + 2𝑘̂ + 𝜇( 𝑖̂ + 2𝑗̂ + 3𝑘̂ ).Also find the length of perpendicular.

LONG ANSWER TYPE QUESTIONS (5 MARKS)

1. Find the shortest distance between the lines

𝑟⃗ = 4𝑖̂ − 𝑗̂ + 𝜆( 𝑖̂ + 2𝑗̂ − 3𝑘̂ ) and 𝑟⃗ = 𝑖̂ − 𝑗̂ + 2𝑘̂ + 𝜇( 2𝑖̂ + 4𝑗̂ − 5𝑘̂ )

𝑥−1 𝑦−2 𝑧−3 𝑥−2 𝑦−4 𝑧−5


2. Find the shortest distance between the lines = = and = =
2 3 4 3 4 5
88

𝑥−1 𝑦+1 𝑥+1 𝑦−2


3. Find the shortest distance between the lines = = 𝑧 and = ;𝑧=2
2 3 5 1

4. Find the shortest distance between the lines

𝑟⃗ = 𝑖̂ + 2𝑗̂ + 3𝑘̂ + 𝜆( 2𝑖̂ + 3𝑗̂ + 4𝑘̂ ) and 𝑟⃗ = 2𝑖̂ + 4 𝑗̂ + 5𝑘̂ + 𝜇( 4𝑖̂ + 6𝑗̂ + 8𝑘̂ )

5. Find the shortest distance between the lines

𝑟⃗ = (1 − 𝑡)𝑖̂ + (𝑡 − 2)𝑗̂ + (3 − 𝑡)𝑘̂ and 𝑟⃗ = (𝑠 + 1)𝑖̂ + (2𝑠 − 1)𝑗̂ − (2𝑠 + 1)𝑘̂

6. Find the vector equation of the line passing through the point (2,3,2) and parallel to the line 𝑟⃗ =

−2𝑖̂ + 3𝑗̂ + 𝜆( 2𝑖̂ − 3𝑗̂ + 6𝑘̂ ) .Also find the distance between these lines.
𝑥+2 𝑦−1 𝑧
7. Vertices B and C of a ∆𝐴𝐵𝐶 lie along the line = = 4 .Find the area of the triangle given
2 1

that A has coordinates (1, −1,2) and line segment BC has length 5.
𝑥 𝑦−1 𝑧−2
8. Find the image of the point (1,6,3) in the line = = .Also write the equation of the line
1 2 3

joining the given point and its image and find the length of the segment joining the given point and

its image.

9. Find the angle between the lines whose direction cosines are given by the equations

3𝑙 + 𝑚 + 5𝑛 = 0 and 6𝑚𝑛 − 2𝑛𝑙 + 5𝑙𝑚 = 0

10. Show that the straight lines whose direction cosines are given by the equations

𝑎𝑙 + 𝑏𝑚 + 𝑐𝑛 = 0 and 𝑢𝑙 2 + 𝑣𝑚2 + 𝑤𝑛2 = 0 are

(i) perpendicular if 𝑎2 (𝑣 + 𝑤) + 𝑏 2 (𝑢 + 𝑤) + 𝑐 2 (𝑢 + 𝑣) = 0

𝑎2 𝑏2 𝑐2
(ii) Parallel if = =
𝑢 𝑣 𝑤

1
11. Show that the angle between diagonals of a cube is cos −1 (3).

12. An insect is crawling along the line 𝑟⃗ = 6𝑖̂ + 2𝑗̂ + 2𝑘̂ + 𝜆( 𝑖̂ − 2𝑗̂ + 2𝑘̂ ) and another insect is

crawling along the line 𝑟⃗ = −4𝑖̂ − 𝑘̂ + 𝜇(3 𝑖̂ − 2𝑗̂ − 2𝑘̂ ) At what points on the lines should they

reach so that the distance between them is the shortest? lines should they reach so that the distance

between them is the shortest? Find the shortest distance between them.

13. The equations of motion of a rocket are: 𝑥 = 2𝑡, 𝑦 = −4𝑡, 𝑧 = 4𝑡, where the time t is given in
89

seconds, and the coordinates of a moving point in km. What is the path of the rocket? At what

distances will the rocket be from the starting point O(0, 0, 0) and from the following line in 10

seconds? Equation of line is 𝑟⃗ = 20𝑖̂ − 10𝑗̂ + 40𝑘̂ + 𝜆(10 𝑖̂ − 20𝑗̂ + 10𝑘̂ ).

14. The vector form of equations of two lines, 𝑙1 𝑎𝑛𝑑 𝑙2 𝑎𝑟𝑒:

𝑙1 : 𝑟⃗ = 2𝑖̂ − 𝑘̂ + 𝜆( −2𝑗̂ + 𝑘̂ ) and 𝑙2 : 𝑟⃗ = 𝑖̂ + 3𝑗̂ + 2𝑘̂ + 𝜇( 𝑖̂ − 2𝑘̂ ) show that

𝑙1 𝑎𝑛𝑑 𝑙2 are skew lines.

CASE STUDY QUESTIONS

CASE STUDY: 1 The Indian coast guard, while patrolling, saw a suspicious boat
with people. They were nowhere looking like fishermen. The coast guard were
closely observing the movement ofthe boat for an opportunity to seize the boat.
They observed that the boat is moving alonga planar surface. At an instant of time,
the coordinates of the position of the coast guard helicopter and the boat is (1, 3,
5) and (2, 5, 3) respectively.

Based on the above answer the following:

1. If the coast guard decide to shoot the boat at that given instant of time, then what isthe
distance (in meters) that the bullet has to travel?

(𝑎)5 𝑚 (b) 3 𝑚 (c) 6 𝑚 (d) 4 𝑚

2. If the coast guard decides to shoot the boat at that given instant of time, when the speed
of bullet is 36m/sec, then what is the time taken for the bullet to travel and hitthe
boat?
90

1 1 1 1
(𝑎) 𝑠𝑒𝑐 (b) 𝑠𝑒𝑐 (c) 𝑠𝑒𝑐 (d) sec
8 14 10 12

3. At that given instant of time, the equation of line passing through the positions of the
helicopter and boat is
𝑥−1 𝑦−3 𝑧−5 𝑥−1 𝑦+3 𝑧−5
(a) = = (b) = =
1 2 −2 2 1 −2

𝑥+1 𝑦−3 𝑧−5 𝑥−1 𝑦+3 𝑧+5


(c) = = (d) = =
−2 −1 −2 2 −1 2

4. At a different instant of time, the boat moves to a different position along the planar
surface. What should be the coordinates of the location of the boat if the coast guard
shoots the bullet along the line whose equation is

𝑥−1 𝑦−3 𝑧−5


= = for the bullet to hit the boat?
1 2 −2
(a) (8/3, 19/3, -14/3) (b) (8/3 ,-19/3,-14/3)

(c) (8/3 ,-19/3,14/3) (d) none of the above

1 𝑥−1 𝑦−3 𝑧−5


Answers: 1. (b) 3 𝑚 2. (d)
12
sec 3. (a) 1
=
2
=
−2
4. (d) none

CASE STUDTY: 2

The equation of motion of a missile are x = 3t, y = -4t, z = t, where the time ‘t’ is
given inseconds, and the distance is measured in kilometres.

Based on the above answer the following:


1. What is the path of the missile?

1). Straight line 2). Parabola 3). Circle 4). Ellipse


2. Which of the following points lie on the path of the missile?
91

a. (6, 8, 2) b. (6, -8, -2) c. (6, -8, 2) d. (-6, -8, 2)


3. At what distance will the rocket be from the starting point (0, 0, 0) in 5 seconds? (a)√550 𝑘𝑚𝑠
(b)√650 𝑘𝑚𝑠 (c)√450 𝑘𝑚𝑠 (d)√750 𝑘𝑚𝑠

4. If the position of rocket at a certain instant of time is (5, -8, 10), then what will be theheight of the
rocket from the ground? (The ground is considered as the xy – plane).

a). 12 km B). 11 km c). 20 km d). 10 km

ANSWERS: 1. (a) Straight line 2. (c) (6, -8, 2) 3. (𝒃)√650 𝑘𝑚𝑠 4. (d) 10 km

Chapter 12. LINEAR PROGRAMMING PROBLEMS


Formulae/Important Points

• Linear Programming Problem: A linear programming problem is one in which we have


to find optimal value (maximum or minimum) of a linear function of several variables
(called objective function) subject to certain conditions that the variables are non-
negative and satisfying by a set of linear inequalities with variables, are sometimes
called division variables.
• Terms related to Linear Programming
Objective Function: A linear function z = px + qy (p and q are constants) which has to
be maximised or minimised, is called an objective function.
• Feasible Region: The common region determined by all the constraints including non-
negative constraints 𝑥, 𝑦 ≥ 0 of a linear programming problem is called the feasible
region for the problem. The region other than the feasible region is called an infeasible
region. The feasible region is always a convex polygon.
• Optimal Feasible Solution: Any point in the feasible region that gives the optimal value
of the objective function is called the optimal feasible solution.
• Bounded and Unbounded Region: A feasible region of a system of linear inequalities
is said to be bounded, if it can be enclosed within a circle. Otherwise, it is called
unbounded.
• Fundamental Theorems for Solving Linear Programming
Theorem 1: Let R be the feasible region for a linear programming problem and let z = ax
+ by be the objective function. When z has an optimal value (maximum or minimum),
where the variables x and y are subject to constraints described by linear inequalities.
This optimal value must occur at a corner point (vertex) of the feasible region.
Note: A corner point of a feasible region is a point in the region which is the intersection
of two boundary lines.
• Theorem 2: Let R be the feasible region for a linear programming problem and let z = ax
+ by be the objective function. If R is bounded, then z has both a maximum and a
minimum value on R and each of these recurs at a corner point of JR.
Note: Maximum or a minimum may not exist,- if the feasible region is unbounded.
92

• Steps for Applying Corner Point Method


Find the feasible region of the linear programming problem and determine its corner
points either by inspection or by solving the two equations of the lines intersecting at that
point.

• Evaluate the objective function z = ax + by at each corner point. Let M and m be,
respectively denote the largest and the smallest values of these points.

• If the feasible region is bounded, then M and m respectively are the maximum and
minimum values of the objective function at corner points.

• If the feasible region is unbounded , then


(a) M is the maximum value of objective function z, if the open half plane determined by
ax + by > M has no point in common with the feasible region. Otherwise, z has no
maximum value.
(b) m is the minimum value of z, if the open half plane determined by ax + by < m has no
point in common with the feasible region. Otherwise, z has no minimum value.

• If two corner points of the feasible region are both optimal solutions of the same type, i.e
both produce the same maximum or minimum, then any point on the line segment joining
these two points is also an optimal solution of the same type.

MULTIPLE CHOICE QUESTIONS

1 The corner points of the feasible region determined by the system of linear
constraints are (0, 0), (0,40), (20,40),(60,20),(60,0).The objective function is
Compare the quantity in Column A and Column B

Column A Column B
Maximum of Z 325
(a) The quantity in column A is greater (b)The quantity in column B is greater

(c) The two quantities are equal. (d) The relationship cannot be
determined on the basis of the
information supplied.
93

2 The feasible solution for a LPP is shown in given figure. Let Z=3x-4y be the objective
function. Minimum of Z occurs at
(4, 10)

(0, 8) (6, 8)

(6, 5)

a) (0,0) (b) (0,8)

(c) (5,0) (d) (4,10)

3 Corner points of the feasible region determined by the system of linear constraints
are (0,3),(1,1) and (3,0). Let Z= px+qy, where p, q>0. Condition on p and q so that
the minimum of Z occurs at (3,0) and (1,1) is

(a) p=2q (b) p=q/2

(c) p=3q (d) p=q

4 The set of all feasible solutions of a LPP is a ____ set.

(a) Concave (b) Convex

(c) Feasible (d) None of these

5 Corner points of the feasible region for an LPP are (0,2), (3,0), (6,0), (6,8) and (0,5).
Let F=4x+6y be the objective function. Maximum of F – Minimum of F =

(a) 60 (b) 48

(c) 42 (d) 18

6 In a LPP, if the objective function Z = ax+by has the same maximum value on two
corner points of the feasible region, then every point on the line segment joining
these two points give the same……….value.

(a) minimum (b) maximum

(c) zero (d) none of these

7 In the feasible region for a LPP is ………, then the optimal value of the objective
function Z = ax+by may or may not exist.

(a) bounded (b) unbounded


94

(c) in circled form (d) in squared form

8 A linear programming problem is one that is concerned with finding the …A … of a


linear function called …B… function of several values (say x and y), subject to the
conditions that the variables are …C… and satisfy set of linear inequalities called
linear constraints.

(a) Objective, optimal value, negative (b) Optimal value, objective, negative

(c) Optimal value, objective, non- (d) Objective, optimal value, non-
negative negative

9 Maximum value of the objective function Z = ax+by in a LPP always occurs at only
one corner point of the feasible region.

(a) true (b) false

(c) can’t say (d) partially true

10 Region represented by x≥0,y≥0 is:

(a) First quadrant (b) Second quadrant

(c) Third quadrant (d) Fourth quadrant

11 Z =3x + 4y, Subject to the constraints x+y 1, x,y ≥0.


the shaded region shown in the figure as OAB is bounded and the coordinates of
corner points O, A and B are (0,0),(1,0) and (0,1), respectively.

The maximum value of Z is 2.

(a) true (b) false

(c) can’t say (d) partially true

12 The feasible region for an LPP is shown shaded in the figure. Let Z = 3x-4y be
objective function. Maximum value of Z is:
95

(a) 0 (b) 8

(c) 12 (d) -18

13 The maximum value of Z = 4x+3y, if the feasible region for an LPP is as shown
below, is

(a) 112 (b) 100

(c) 72 (d) 110

14 The feasible region for an LPP is shown shaded in the figure. Let Z = 4x-3y be
objective function. Maximum value of Z is:

(a) 0 (b) 8

(c) 30 (d) -18

15 In the given figure, the feasible region for a LPP is shown. Find the maximum and
minimum value of Z = x+2y.
96

(a) 8, 3.2 (b) 9, 3.14

(c) 9, 4 (d) none of these

16 The linear programming problem minimize Z= 3x+2y,subject to constraints

x+y8, 3x+5y 15, x,y ≥0, has


(a) One solution (b) No feasible solution

(c) Two solutions (d) Infinitely many solutions

17 The graph of the inequality 2x+3y > 6 is:

(a) half plane that contains the origin (b) half plane that neither contains
the origin nor the points of the line
2x+3y =6

(c) whole XOY-plane excluding the (d) entire XOY-plane


points on the line 2x+3y =6

18 Of all the points of the feasible region for maximum or minimum of objective function
the points

(a) Inside the feasible region (b) At the boundary line of the
feasible region

(c) Vertex point of the boundary of (d) None of these


the feasible region

19 The maximum value of the object function Z = 5x + 10 y subject to the constraints x +


2y ≤ 120, x + y ≥ 60, x – 2y ≥ 0, x ≥ 0, y ≥ 0 is

(a) 300 (b) 600

(c) 400 (d) 800

20 Z = 6x + 21 y, subject to x + 2y ≥ 3, x + 4y ≥ 4, 3x + y ≥ 3, x ≥ 0, y ≥ 0. The minimum


value of Z occurs at

(a) (4, 0) (b) (28, 8)


97

(c) (2,2/7 ) (d) (0, 3)

21 Shape of the feasible region formed by the following constraints x + y ≤ 2, x+


y ≥ 5, x ≥ 0, y ≥ 0

(a) No feasible region (b) Triangular region

(c) Unbounded solution (d) Trapezium

22 Maximize Z = 4x + 6y, subject to 3x + 2y ≤ 12, x + y ≥ 4, x, y ≥ 0.

(a) 16 at (4, 0) (b) 24 at (0, 4)

(c) 24 at (6, 0) (d) 36 at (0, 6)

23 Feasible region for an LPP shown shaded in the following


figure. Minimum of Z = 4x+3y occurs at the point:

(a) (0,8) (b) (2,5)

(c) (4,3) (d) (9,0)

24 The region represented by the inequalities


x ≥ 6, y ≥ 2, 2x + y ≤ 0, x ≥ 0, y ≥ 0 is

(a) unbounded (b) a polygon

(c) exterior of a triangle (d) None of these

25 Minimize Z = 13x – 15y subject to the constraints : x + y ≤ 7, 2x – 3y + 6 ≥ 0 , x ≥ 0, y


≥ 0.

(a) -23 (b) -32

(c) -30 (d) -34

26 The solution set of the inequality 3x + 5y < 4 is

(a) an open half-plane not containing the (b) an open half-plane containing the
origin. origin.
98

(c) the whole XY-plane not containing the (d) a closed half plane containing the
line 3x + 5y = 4. origin.

27 The corner points of the shaded unbounded feasible region of an LPP are (0, 4),
(0.6, 1.6) and (3, 0) as shown in the figure. The minimum value of the objective
function Z = 4x + 6y occurs at

(a) (0.6, 1.6) 𝑜𝑛𝑙𝑦 (b) (3, 0) only

(c) (0.6, 1.6) and (3, 0) only (d) at every point of the line-segment joining
the points (0.6, 1.6) and (3, 0)

28 The value of objective function is maximum under linear constraints-------


(a) at the centre of feasible region (b) at (0,0)

(c) at a vertex of feasible region (d) the vertex which is of maximum distance
from (0,0)

29 Which of the following is correct


(a) Every LPP has an optimal solution (b) A LPP has unique optimal solution

(c) If LPP has two optimal solutions (d) the vertex which is of maximum distance
then it has infinite number of optimal from (0,0)
solutions.

30 Objective function of LPP is


(a) a constraint (b) A function to be maximized or
minimised

(c) a relation between the decision (d) equation of a straight line


variables
99

Chapter 13 – PROBABILITY
• Random Experiment - Sample Space – Event - Simple Event - Compound Event
• Sure Event - Impossible Event - Mutually Exclusive Events -Independent Events
• Equally Likely Events - Exhaustive events - Probability of happening of an event E = P(E)
• Probability of not happening of an event E = P(E’)
• P(E) + P(E’) = 1 0≤P(E)≥1
1 ) Addition theorem on probability
2) Conditional Probability 3) Multiplication theorem on Probability
# ADDITION THEOREM ON PROBABILITY: If 𝑬𝟏 𝒂𝒏𝒅𝑬𝟐 two events of a sample space of a random
experiment then P (𝑬𝟏 𝑶𝑹 𝑬𝟐 ) = P (𝑬𝟏 𝑬𝟐 ) = P(𝑬𝟏 ) + 𝑷(𝑬𝟐 ) − 𝑷 (𝑬𝟏  𝑬𝟐 )

* If 𝑬𝟏 𝒂𝒏𝒅𝑬𝟐 are two mutually exclusive events then 𝑷 (𝑬𝟏 𝑬𝟐 ) = 0

P (𝑬𝟏 𝑬𝟐) = P(𝑬𝟏 ) + 𝑷(𝑬𝟐 )

Conditional Probability: If 𝑬𝟏 𝒂𝒏𝒅𝑬𝟐 two events of a sample space of a random experiment such that
𝑷 (𝑬𝟏 ∩ 𝑬𝟐 )
𝑷(𝑬𝟐 )  then P(𝑬𝟏 / 𝑬𝟐 ) = 𝑷(𝑬𝟐 )

Multiplication theorem on Probability: If 𝑬𝟏 𝒂𝒏𝒅𝑬𝟐 two events of a sample space of a random experiment
then P(𝑬𝟏 𝑨𝑵𝑫 𝑬𝟐 ) = P (𝐄𝟏 𝐄𝟐) = P(𝐄𝟏 ) 𝐱 P(𝐄𝟐 / 𝐄𝟏 )

If 𝑬𝟏 𝒂𝒏𝒅𝑬𝟐 are two Independent events then P(𝑬𝟐 /𝑬𝟏 ) = P(𝑬𝟐 ) P (𝑬𝟏 𝑬𝟐 ) = P(𝑬𝟏 ) 𝒙 𝑷(𝑬𝟐 )

𝑩𝑨𝒀𝑬𝑺′ 𝑻𝑯𝑬𝑶𝑹𝑬𝑴: 𝑳𝒆𝒕 𝑬𝟏 , 𝑬𝟐 , 𝑬𝟑 … … 𝑬𝒏 𝒃𝒆 𝒏𝒐𝒏 𝒆𝒎𝒑𝒕𝒚 𝒆𝒗𝒆𝒏𝒕𝒔 𝒘𝒉𝒊𝒄𝒉 𝒄𝒐𝒏𝒔𝒕𝒊𝒕𝒖𝒕𝒆 𝒂 𝒑𝒂𝒓𝒕𝒊𝒕𝒊𝒐𝒏 𝒐𝒇

𝒕𝒉𝒆 𝒔𝒂𝒎𝒑𝒍𝒆 𝒔𝒑𝒂𝒄𝒆 𝑺, 𝒊. 𝒆 𝑬𝟏 , 𝑬𝟐 , 𝑬𝟑 … … 𝑬𝒏 𝒂𝒓𝒆 𝒑𝒂𝒊𝒓𝒘𝒊𝒔𝒆 𝒅𝒊𝒔𝒋𝒐𝒊𝒏𝒕 𝒂𝒏𝒅 𝑬𝟏 𝑬𝟐  … … 𝑬𝒏 = 𝑺


𝑨
𝑬 𝑷(𝑬𝒊 ) 𝑷( )
𝑬𝒊
𝒂𝒏𝒅 𝑨 𝒊𝒔 𝒂𝒏𝒚 𝒆𝒗𝒆𝒏𝒕 𝒕𝒉𝒆𝒏𝑷 ( 𝑨𝒊 ) = 𝑨
, 𝒇𝒐𝒓 𝒂𝒏𝒚 𝒊 = 𝟏, 𝟐 … … . 𝒏
∑ 𝑷(𝑬𝒊 ) 𝑷( )
𝑬𝒊

Random Variable: A random variable is real valued function whose domain is the sample of the random
experiment. Ex. When a coin is tossed two times then

S = { HH, TT, TH, HT } X = No of heads obtain = 0, 1, 2

Probability Distribution: Probability Distribution is a table showing a random variable and its
corresponding frequencies.

ONE Mark & TWO Marks Questions


1). Find Sample Space when a coin is tossed two times. S = { TT, HH, HT, TH }

2). Find Sample Space when Three coins are tossed S= { TTT, TTH, THT, THH, HTT, HTH,HHT, HHH }

3). Find Sample Space when: A Die is thrown S = { 1, 2, 3, 4, 5, 6 }.

4). Find Sample Space when: Two dice are thrown


100

S = { (1,1), (1,2),(1,3),(1,4),(1,5),(1,6) (2,1), (2,2),(2,3),(2,4),(2,5),(2,6)

(3,1), (3,2),(3,3),(3,4),(3,5),(3,6) (4,1), (4,2),(4,3),(4,4),(4,5),(4,6)

(5,1), (5,2),(5,3),(5,4),(5,5),(5,6) (6,1), (6,2),(6,3),(6,4),(6,5),(6,6) }.

5). Find Sample Space when: A coin is tossed and then a die is rolled only in case a head is shown on the
coin. S = { T,(H,1), (H,2) (H,3),(H,4),(H,5),(H,6) }.

6). An experiment consists of recording boy-girl composition of families with 2 children.


What is the sample space if we are interested in knowing whether it is a boy or girl in the order of their
births? S = { BB, BG, GB, GG}.

7). Find the probability of drawing 2 kings from a pack of cards.


𝟒 𝟑 𝟒 𝟑
P(𝑲𝟏 ) = 𝟓𝟐 P(𝑲𝟐 / 𝑲𝟏) =𝟓𝟏 P (Two Kings) = 𝟓𝟐 x 𝟓𝟏 = 1/221

8). 4 cards are drawn from a well-shuffled deck of 52 cards. What is the probability of obtaining 3
diamonds and one spade?

Diamond cards = 13, Spade cards = 13, Total cards = 52, Cards drawn = 4

𝟏𝟑𝑪𝟑 𝒙 𝟏𝟑𝑪𝟏
Required Probability =
𝟓𝟐𝑪𝟒

THREE Marks & FIVE Marks Questions


1). When a pair of dice are thrown find the probability of getting following events:

(1). P(sum 8) = (2,6)(3,5)(4,4)(5,3)(6,2) = 5/36 (2). P(sum 11)= (5,6)(6,5) = 2 / 36

(3). P(Doublet)= (1,1)(2,2)(3,3)(4,4)(5,5)(6,6) = 6 / 36 (4). P(sum 8 or 11)= = 7 / 36

2). Three coins are tossed once. Find the probability of getting the following:

(i) P ( 3 heads) = 1/8 ii) P ( 2 heads ) = 3/8 iii) P ( at least 2 heads) =4/8
iv) P ( at most 2 heads ) =7/8 v) P ( no head ) =1/8 vi) P ( 3 tails )=1/8
vii) P ( exactly two tails ) =3/8
3). 𝑮𝒊𝒗𝒆𝒏 𝒕𝒉𝒂𝒕 𝑬 𝒂𝒏𝒅 𝑭 𝒂𝒓𝒆 𝒆𝒗𝒆𝒏𝒕𝒔 𝒔𝒖𝒄𝒉 𝒕𝒉𝒂𝒕 𝑷(𝑬) = 𝟎. 𝟔, 𝑷(𝑭) = 𝟎. 𝟑 𝒂𝒏𝒅

̅ )(𝒊𝒊). 𝑷( 𝑭̅ ) (𝒊𝒊𝒊). 𝑷 (𝑬 ∪ 𝑭) (𝒊𝒗). 𝑷 (𝑬|𝑭)𝒂𝒏𝒅 (𝒗). 𝑷(𝑭|𝑬)


𝑷(𝑬 ∩ 𝑭) = 𝟎. 𝟐, 𝑭𝒊𝒏𝒅 (𝒊). 𝑷( 𝑬

̅ ) = 𝟏 − 𝑷( 𝑬 ) = 𝟏 − 𝟎. 𝟔 = 𝟎. 𝟒
(i). 𝑷( 𝑬 (𝒊𝒊). 𝑷( 𝑭̅ ) = 𝟏 − 𝑷( 𝑭 ) = 𝟏 − 𝟎. 𝟑 = 𝟎. 𝟕

(𝒊𝒊𝒊). 𝑷 (𝑬 ∪ 𝑭) = 𝑷(𝑬) + 𝑷(𝑭) − 𝑷(𝑬 ∩ 𝑭) = 𝟎. 𝟔 + 𝟎. 𝟑 − 𝟎. 𝟐 = 𝟎. 𝟕


𝑷(𝑬 ∩ 𝑭) 𝟎.𝟐 𝟐 𝑷(𝑬 ∩ 𝑭) 𝟎.𝟐 𝟏
(𝒊𝒗). 𝑷 (𝑬|𝑭) = = = (𝒗). 𝑷 (𝑭|𝑬) = = =
𝑷( 𝑭) 𝟎.𝟑 𝟑 𝑷( 𝑬) 𝟎.𝟔 𝟑

4). 𝑨𝒔𝒔𝒖𝒎𝒆 𝒕𝒉𝒂𝒕 𝒆𝒂𝒄𝒉 𝒃𝒐𝒓𝒏 𝒄𝒉𝒊𝒍𝒅 𝒊𝒔 𝒆𝒒𝒖𝒂𝒍𝒍𝒚 𝒍𝒊𝒌𝒆𝒍𝒚 𝒕𝒐 𝒃𝒆 𝒂 𝒃𝒐𝒚 𝒐𝒓 𝒂 𝒈𝒊𝒓𝒍. 𝑰𝒇 𝒂 𝒇𝒂𝒎𝒊𝒍𝒚 𝒉𝒂𝒔 𝒕𝒘𝒐

𝒄𝒉𝒊𝒍𝒅𝒓𝒆𝒏, 𝒘𝒉𝒂𝒕 𝒊𝒔 𝒕𝒉𝒆 𝒄𝒐𝒏𝒅𝒊𝒕𝒊𝒐𝒏𝒂𝒍 𝒑𝒓𝒐𝒃𝒂𝒃𝒊𝒍𝒊𝒕𝒚 𝒕𝒉𝒂𝒕 𝒃𝒐𝒕𝒉 𝒂𝒓𝒆 𝒈𝒊𝒓𝒍𝒔 𝒈𝒊𝒗𝒆𝒏 𝒕𝒉𝒂𝒕
101

(𝒊) 𝒕𝒉𝒆 𝒚𝒐𝒖𝒏𝒈𝒆𝒔𝒕 𝒊𝒔 𝒂 𝒈𝒊𝒓𝒍, (𝒊𝒊)𝒂𝒕 𝒍𝒆𝒂𝒔𝒕 𝒐𝒏𝒆 𝒊𝒔 𝒂 𝒈𝒊𝒓𝒍 ?

𝑺 = {(𝑩𝑩), (𝑩𝑮), (𝑮𝑩), (𝑮𝑮)}

𝑳𝒆𝒕 𝑨 𝒃𝒆 𝒕𝒉𝒆 𝒆𝒗𝒆𝒏𝒕 𝒕𝒉𝒂𝒕 𝒃𝒐𝒕𝒉 𝒄𝒉𝒊𝒍𝒅𝒓𝒆𝒏 𝒂𝒓𝒆 𝒈𝒊𝒓𝒍𝒔, 𝑨 = { 𝑮𝑮 }

𝑳𝒆𝒕 𝑩 𝒃𝒆 𝒕𝒉𝒆 𝒆𝒗𝒆𝒏𝒕 𝒕𝒉𝒂𝒕 𝒕𝒉𝒆 𝒚𝒐𝒖𝒏𝒈𝒆𝒔𝒕 𝒄𝒉𝒊𝒍𝒅 𝒊𝒔 𝒂 𝒈𝒊𝒓𝒍 𝑩 = { 𝑩𝑮, 𝑮𝑮 }


𝟏
𝟐 𝟏 𝟏 𝑷(𝑨 ∩𝑩) 𝟏
𝑨  𝑩 = { 𝑮𝑮 } 𝑷(𝑩) = = 𝟒 𝟐
𝑷(𝐀 ∩ 𝑩) = 𝟒
𝑷 (𝑨|𝑩) = 𝑷( 𝑩)
= 𝟒
𝟏 = 𝟐
𝟐

𝟏 𝟐 𝟏
5). 𝑻𝒉𝒆 𝒑𝒓𝒐𝒃𝒂𝒃𝒊𝒍𝒊𝒕𝒊𝒆𝒔 𝒐𝒇 𝒔𝒐𝒍𝒗𝒊𝒏𝒈𝒂 𝒂 𝒎𝒂𝒕𝒉𝒔 𝒑𝒓𝒐𝒃𝒍𝒆𝒎 𝒃𝒚 𝒕𝒉𝒓𝒆𝒆 𝒔𝒕𝒖𝒅𝒆𝒏𝒕𝒔 𝑨, 𝑩, 𝑪 𝒂𝒓𝒆 𝟐 , 𝟑 , 𝒂𝒏𝒅 𝟑

respectively. Find the probability that (1). The problem is solved. (2). Exactly one solves the problem.
𝟏 𝟏 𝟐 𝟏 𝟏 𝟐
𝑷(𝑨) = 𝟐 , 𝑷(𝑨′ ) = 𝟐 ; 𝑷(𝑩) = 𝟑 , 𝑷(𝑩′ ) = 𝟑 ; 𝑷(𝑪) = 𝟑 𝑷(𝑪′ ) = 𝟑 ,

𝟏 𝟏 𝟐 𝟏 𝟖
𝑷(𝑻𝒉𝒆 𝒑𝒓𝒐𝒃𝒍𝒆𝒎 𝒊𝒔 𝒔𝒐𝒍𝒗𝒆𝒅) = 𝟏 − 𝑷(𝑨′ ). 𝑷(𝑩′ ). 𝑷(𝑪′ ) = 𝟏 − ( 𝟐
) . (𝟑) . (𝟑) =𝟏 − 𝟗
=𝟗

𝑷(𝑬𝒙𝒂𝒂𝒄𝒕𝒍𝒚 𝒐𝒏𝒆 𝒔𝒐𝒍𝒗𝒆𝒔 𝒕𝒉𝒆 𝒑𝒓𝒐𝒃𝒍𝒆𝒎 ) = 𝑷(𝑨). 𝑷(𝑩′ ). 𝑷(𝑪′ )) + 𝑷(𝑨′ ). 𝑷(𝑩). 𝑷(𝑪′ )

+ 𝑷(𝑨′ ). 𝑷(𝑩′ ). 𝑷(𝑪)

𝟏 𝟏 𝟐 𝟏 𝟐 𝟐 𝟏 𝟏 𝟏 𝟏 𝟐 𝟏 𝟕
=( 𝟐
) . (𝟑) . (𝟑) + ( 𝟐
) . (𝟑) . (𝟑) + ( 𝟐
) . (𝟑) . (𝟑) =( 𝟗
) + (𝟗) + (𝟏𝟖) =( 𝟏𝟖
)

6). 𝐀 𝐛𝐚𝐠 𝐜𝐨𝐧𝐭𝐚𝐢𝐧𝐬 𝟒 𝐫𝐞𝐝 𝐚𝐧𝐝 𝟒 𝐛𝐥𝐚𝐜𝐤 𝐛𝐚𝐥𝐥𝐬, 𝐚𝐧𝐨𝐭𝐡𝐞𝐫 𝐛𝐚𝐠 𝐜𝐨𝐧𝐭𝐚𝐢𝐧𝐬 𝟐 𝐫𝐞𝐝 𝐚𝐧𝐝 𝟔 𝐛𝐥𝐚𝐜𝐤 𝐛𝐚𝐥𝐥𝐬.

𝐎𝐧𝐞 𝐨𝐟 𝐭𝐡𝐞 𝐭𝐰𝐨 𝐛𝐚𝐠𝐬 𝐢𝐬 𝐬𝐞𝐥𝐞𝐜𝐭𝐞𝐝 𝐚𝐭 𝐫𝐚𝐧𝐝𝐨𝐦 𝐚𝐧𝐝 𝐚 𝐛𝐚𝐥𝐥 𝐢𝐬 𝐝𝐫𝐚𝐰𝐧 𝐟𝐫𝐨𝐦 𝐭𝐡𝐞 𝐛𝐚𝐠 𝐰𝐡𝐢𝐜𝐡 𝐢𝐬 𝐟𝐨𝐮𝐧𝐝

𝐭𝐨 𝐛𝐞 𝐫𝐞𝐝. 𝐅𝐢𝐧𝐝 𝐭𝐡𝐞 𝐩𝐫𝐨𝐛𝐚𝐛𝐢𝐥𝐢𝐭𝐲 𝐭𝐡𝐚𝐭 𝐭𝐡𝐞 𝐛𝐚𝐥𝐥 𝐢𝐬 𝐝𝐫𝐚𝐰𝐧 𝐟𝐫𝐨𝐦 𝐭𝐡𝐞 𝐟𝐢𝐫𝐬𝐭 𝐛𝐚𝐠.

Bag – A: Red balls = 4 Black balls = 4 Total = 8

Bag – B: Red balls = 2 Black balls = 6 Total = 8

𝟏
𝑳𝒆𝒕 𝑬𝟏 = 𝒄𝒉𝒐𝒐𝒔𝒊𝒏𝒈 𝒃𝒂𝒈 𝑨 𝑷(𝑬𝟏 ) = 𝟐

𝟏
𝑳𝒆𝒕 𝑬𝟐 = 𝒄𝒉𝒐𝒐𝒔𝒊𝒏𝒈 𝒃𝒂𝒈 𝑩 𝑷(𝑬𝟏 ) = 𝑳𝒆𝒕 𝑨 = 𝑫𝒓𝒂𝒘𝒊𝒏𝒈 𝒓𝒆𝒅 𝒃𝒂𝒍𝒍
𝟐

𝑨 𝟒 𝟏 𝑨 𝟐 𝟏
𝑷( ) = = 𝑷( ) = =
𝑬𝟏 𝟖 𝟐 𝑬𝟐 𝟖 𝟒

𝑨 𝑨
𝑷(𝑬𝒊 ) 𝑷( ) 𝑷(𝑬𝟏 ) 𝑷( )
𝑬𝒊 𝑬𝟏
𝑹𝒆𝒒𝒖𝒊𝒓𝒆𝒅 𝑷𝒓𝒐𝒃 = 𝑨
= 𝑨 𝑨
∑ 𝑷(𝑬𝒊 ) 𝑷( ) 𝑷(𝑬𝟏 ) 𝑷( )+𝑷(𝑬𝟐 ) 𝑷( )
𝑬𝒊 𝑬𝟏 𝑬𝟐

𝟏 𝟒
𝑹𝒆𝒒𝒖𝒊𝒓𝒆𝒅 𝑷𝒓𝒐𝒃 = 𝟐 (𝟖) =
𝟒 𝟐
=
𝟏 𝟒 𝟏 𝟐 𝟔 𝟑
(
𝟐 𝟖) + (
𝟐 𝟖 )

𝑶𝒇 𝒕𝒉𝒆 𝒔𝒕𝒖𝒅𝒆𝒏𝒕𝒔 𝒊𝒏 𝒂 𝒄𝒐𝒍𝒍𝒆𝒈𝒆, 𝒊𝒕 𝒊𝒔 𝒌𝒏𝒐𝒘𝒏 𝒕𝒉𝒂𝒕 𝟔𝟎% 𝒓𝒆𝒔𝒊𝒅𝒆 𝒊𝒏 𝒉𝒐𝒔𝒕𝒆𝒍 𝒂𝒏𝒅 𝟒𝟎% 𝒂𝒓𝒆 𝒅𝒂𝒚 𝒔𝒄𝒉𝒐𝒍𝒂𝒓𝒔

(𝒏𝒐𝒕 𝒓𝒆𝒔𝒊𝒅𝒊𝒏𝒈 𝒊𝒏 𝒉𝒐𝒔𝒕𝒆𝒍). 𝑷𝒓𝒆𝒗𝒊𝒐𝒖𝒔 𝒚𝒆𝒂𝒓 𝒓𝒆𝒔𝒖𝒍𝒕𝒔 𝒓𝒆𝒑𝒐𝒓𝒕 𝒕𝒉𝐚𝒕 𝟑𝟎% 𝒐𝒇 𝒂𝒍𝒍 𝒔𝒕𝒖𝒅𝒆𝒏𝒕𝒔 𝒘𝒉𝒐 𝒓𝒆𝒔𝒊𝒅𝒆
102

𝒊𝒏 𝒉𝒐𝒔𝒕𝒆𝒍 𝒂𝒕𝒕𝒂𝒊𝒏 𝑨 𝒈𝒓𝒂𝒅𝒆 𝒂𝒏𝒅 𝟐𝟎% 𝒐𝒇 𝒅𝒂𝒚 𝒔𝒄𝒉𝒐𝒍𝒂𝒓𝒔 𝒂𝒕𝒕𝒂𝒊𝐧 𝑨 𝒈𝒓𝒂𝒅𝒆 𝒊𝒏 𝒕𝒉𝒆𝒊𝒓 𝒂𝒏𝒏𝒖𝒂𝒍 𝒆𝒙𝒂𝒎𝒊𝒏𝒂𝒕𝒊𝒐𝒏.

𝑨𝒕 𝒕𝒉𝒆 𝒆𝒏𝒅 𝒐𝒇 𝒕𝒉𝒆 𝒚𝒆𝒂𝒓, 𝒐𝒏𝒆 𝒔𝒕𝒖𝒅𝒆𝒏𝒕 𝒊𝒔 𝒄𝒉𝒐𝒔𝒆𝐧 𝒂𝒕 𝒓𝒂𝒏𝒅𝒐𝒎 𝒇𝒓𝒐𝒎 𝒕𝒉𝒆 𝒄𝒐𝒍𝒍𝒆𝒈𝒆 𝒂𝒏𝒅 𝒉𝒆 𝒉𝒂𝒔 𝒂𝒏 𝑨 𝒈𝒓𝒂𝒅𝒆,

𝒘𝒉𝒂𝒕 𝒊𝒔 𝒕𝒉𝐞 𝒑𝒓𝒐𝒃𝒂𝒃𝒊𝒍𝒊𝒕𝒚 𝒕𝒉𝒂𝒕 𝒕𝒉𝒆 𝒔𝒕𝒖𝒅𝒆𝒏𝒕 𝒊𝒔 𝒉𝒐𝒔𝒕𝒍𝒆𝐫.

𝑳𝒆𝒕 𝑬𝟏 𝒃𝒆 𝒕𝒉𝒆 𝒆𝒗𝒆𝒏𝒕 𝒕𝒉𝒂𝒕 𝒕𝒉𝒆 𝒔𝒕𝒖𝒅𝒆𝒏𝒕 𝒊𝒔 𝒂 𝒉𝒐𝒔𝒕𝒍𝐢𝒆𝒓

𝑳𝒆𝒕 𝑬𝟐 𝒃𝒆 𝒕𝒉𝒆 𝒆𝒗𝒆𝒏𝒕 𝒕𝒉𝒂𝒕 𝒕𝒉𝒆 𝒔𝒕𝒖𝒅𝒆𝒏𝒕 𝒊𝒔 𝒂 𝒅𝒂𝒚 𝒔𝒄𝒉𝒐𝒍𝒂𝒓

𝑳𝒆𝒕 𝑨 𝒃𝒆 𝒕𝒉𝒆 𝒆𝒗𝒆𝒏𝒕 𝒕𝒉𝒂𝒕 𝒕𝒉𝒆 𝒔𝒕𝒖𝒅𝒆𝒏𝒕 𝒈𝒆𝒕𝒔 𝑨 𝒈𝒓𝒂𝒅𝒆

𝟔𝟎 𝟑 𝟒𝟎 𝟐
𝑷(𝑬𝟏 ) = 𝟔𝟎% = = 𝑷(𝑬𝟐 ) = 𝟒𝟎% = =
𝟏𝟎𝟎 𝟓 𝟏𝟎𝟎 𝟓
𝑨 𝟑𝟎 𝟑 𝑨 𝟐𝟎 𝟐
𝑷 (𝑬 ) = 𝟑𝟎% = 𝟏𝟎𝟎 = 𝟏𝟎
𝑷 (𝑬 ) = 𝟐𝟎% = 𝟏𝟎𝟎 = 𝟏𝟎
𝟏 𝟐

𝑨 𝑨
𝑷(𝑬𝒊 ) 𝑷( ) 𝑷(𝑬𝟏 ) 𝑷( )
𝑬𝒊 𝑬𝟏
𝑹𝒆𝒒𝒖𝒊𝒓𝒆𝒅 𝑷𝒓𝒐𝒃 = 𝑨
= 𝑨 𝑨
∑ 𝑷(𝑬𝒊 ) 𝑷( ) 𝑷(𝑬𝟏 ) 𝑷( )+𝑷(𝑬𝟐 ) 𝑷( )
𝑬𝒊 𝑬𝟏 𝑬𝟐

𝟔𝟎 𝟑𝟎
( ) 𝟏𝟖𝟎𝟎 𝟗
𝑹𝒆𝒒𝒖𝒊𝒓𝒆𝒅 𝑷𝒓𝒐𝒃 = 𝟏𝟎𝟎 𝟏𝟎𝟎 = =
𝟔𝟎 𝟑𝟎 𝟒𝟎 𝟐𝟎 𝟐𝟔𝟎𝟎 𝟏𝟑
( )+ ( )
𝟏𝟎𝟎 𝟏𝟎𝟎 𝟏𝟎𝟎 𝟏𝟎𝟎

8). In answering a question on a multiple choice test, a student either knows the answer or guesses. Let
3/4 be the probability that he knows the answer and 1/4 be the probability that he guesses.
Assuming that a student who guesses at the answer will be correct with probability ¼. What is the
probability that the student knows the answer given that he answered it correctly?
𝑳𝒆𝒕 𝑬𝟏 = 𝑺𝒕𝒖𝒅𝒆𝒏𝒕 𝒌𝒏𝒐𝒘𝒔 𝒕𝒉𝒆 𝒂𝒏𝒔𝒘𝒆𝒓
𝑳𝒆𝒕 𝑬𝟐 = 𝑺𝒕𝒖𝒅𝒆𝒏𝒕 𝒈𝒖𝒆𝒔𝒔𝒆𝒔 𝒕𝒉𝒆 𝒂𝒏𝒔𝒘𝒆𝒓
𝑳𝒆𝒕 𝑨 = 𝑾𝒓𝒊𝒕𝒊𝒏𝒈 𝒄𝒐𝒓𝒓𝒆𝒄𝒕 𝒂𝒏𝒔𝒘𝒆𝒓
𝟑 𝟏 𝑨 𝑨 𝟏
𝑷(𝑬𝟏 ) = 𝑷(𝑬𝟐 ) = 𝑷 ( ) = 𝟏, 𝑷( ) =
𝟒 𝟒 𝑬𝟏 𝑬𝟐 𝟒
𝑨 𝟑
𝑷(𝑬𝟏 ) 𝑷( ) (𝟏) (𝟑) 𝟏𝟐
𝑬𝟏 𝟒
𝑹𝒆𝒒𝒖𝒊𝒓𝒆𝒅 𝑷𝒓𝒐𝒃 = 𝑨 𝑨
= 𝟑 𝟏 𝟏 = 𝟏 = 𝟏𝟑
𝑷(𝑬𝟏 ) 𝑷( )+𝑷(𝑬𝟐 ) 𝑷( ) (𝟏)+ ( ) (𝟑)+( )
𝑬𝟏 𝑬𝟐 𝟒 𝟒 𝟒 𝟒

9). An Insurance company insured 2000 scooter drivers, 4000 car drivers and 6000 truck drivers. The
probability of accidents are 0.01, 0.03 and 0.15 respectively. One of the insured persons meets with an
accident. What is the probability that he is a scooter driver?
𝑳𝒆𝒕 𝑬𝟏 = 𝑷𝒆𝒓𝒔𝒐𝒏 𝒅𝒓𝒊𝒗𝒆𝒔 𝒂 𝒔𝒄𝒐𝒐𝒕𝒆𝒓 𝑳𝒆𝒕 𝑬𝟐 = 𝑷𝒆𝒓𝒔𝒐𝒏 𝒅𝒓𝒊𝒗𝒆𝒔 𝒄𝒂𝒓
𝑳𝒆𝒕 𝑬𝟑 = 𝑷𝒆𝒓𝒔𝒐𝒏𝒔 𝒅𝒓𝒊𝒗𝒆𝒔 𝒕𝒓𝒖𝒄𝒌 𝑳𝒆𝒕 𝑨 = 𝑻𝒉𝒆 𝐩𝐞𝐫𝐬𝐨𝐧 𝐦𝐞𝐞𝐭𝐬 𝐚𝐜𝐜𝐢𝐝𝐞𝐧𝐭
𝟐𝟎𝟎𝟎 𝟏 𝟒𝟎𝟎𝟎 𝟏 𝟔𝟎𝟎𝟎 𝟏
𝑷(𝑬𝟏 ) = = , 𝑷(𝑬𝟐 ) = = , 𝑷(𝑬𝟑 ) = =
𝟏𝟐𝟎𝟎𝟎 𝟔 𝟏𝟐𝟎𝟎𝟎 𝟑 𝟏𝟐𝟎𝟎𝟎 𝟐
𝑨 𝟏 𝑨 𝟑 𝑨 𝟏𝟓
𝑷 ( ) = 𝟎. 𝟎𝟏 = , 𝑷 ( ) = 𝟎. 𝟎𝟑 = 𝑷 ( ) = 𝟎. 𝟎𝟑 =
𝑬𝟏 𝟏𝟎𝟎 𝑬𝟐 𝟏𝟎𝟎 𝑬𝟐 𝟏𝟎𝟎
103

𝑨 𝑨
𝑷(𝑬𝒊 ) 𝑷( ) 𝑷(𝑬𝟏 ) 𝑷( )
𝑬𝒊 𝑬𝟏
From Bayes' theorem, 𝑹𝒆𝒒𝒖𝒊𝒓𝒆𝒅 𝑷𝒓𝒐𝒃 = 𝑨 = = 𝑨 𝑨 𝑨
∑ 𝑷(𝑬𝒊 ) 𝑷( ) 𝑷(𝑬𝟏 ) 𝑷( )+𝑷(𝑬𝟐 ) 𝑷( )+𝑷(𝑬𝟑 ) 𝑷( )
𝑬𝒊 𝑬𝟏 𝑬𝟐 𝑬𝟑

𝟏 𝟏 𝟏
( )
𝟔 𝟏𝟎𝟎
(𝟔) 𝟏
= 𝟏 𝟏 𝟏 𝟑 𝟏 𝟏𝟓 = 𝟏 𝟏𝟓 =
( )+ ( )+ ( ) (𝟔)+𝟏+( 𝟐 ) 𝟓𝟐
𝟔 𝟏𝟎𝟎 𝟑 𝟏𝟎𝟎 𝟐 𝟏𝟎𝟎

10). A card from a pack of 52 cards is lost. From the remaining cards of the pack, two cards are
drawn and are found to be both diamonds. Find the probability of the lost card being a diamond?
𝟏𝟑 𝟏
𝑳𝒆𝒕 𝑬𝟏 = 𝑪𝒉𝒐𝒐𝒔𝒊𝒏𝒈 𝒂 𝒅𝒊𝒂𝒎𝒐𝒏𝒅 𝒄𝒂𝒓𝒅 𝑷(𝑬𝟏 ) = 𝟓𝟐 = 𝟒
𝟑
𝑳𝒆𝒕 𝑬𝟐 = 𝑪𝒉𝒐𝒐𝒔𝒊𝒏𝒈 𝒂 𝒄𝒂𝒓𝒅 𝒘𝒉𝒊𝒄𝒉 𝒊𝒔 𝒏𝒐𝒕 𝒂 𝒅𝒊𝒂𝒎𝒐𝒏𝒅 𝑷(𝑬𝟐 ) = 𝟒

𝑳𝒆𝒕 𝑨 = 𝑫𝒓𝒂𝒘𝒊𝒏𝒈 𝒕𝒘𝒐 𝒅𝒊𝒂𝒎𝒐𝒏𝒅 𝒄𝒂𝒓𝒅𝒔


𝑨 𝟏𝟐𝑪𝟐 𝟏𝟐 (𝟏𝟏) 𝑨 𝟏𝟑𝑪𝟐 𝟏𝟑 (𝟏𝟐)
𝑷 (𝑬 ) = 𝟓𝟏 = 𝑷 (𝑬 ) = 𝟓𝟏 =
𝟏 𝑪𝟐 𝟓𝟏(𝟓𝟎) 𝟐 𝑪𝟐 𝟓𝟏(𝟓𝟎)

𝑨
𝑷(𝑬𝟏 ) 𝑷 (𝑬 )
𝟏
𝑹𝒆𝒒𝒖𝒊𝒓𝒆𝒅 𝑷𝒓𝒐𝒃 =
𝑨 𝑨
𝑷(𝑬𝟏 ) 𝑷 (𝑬 ) + 𝑷(𝑬𝟐 ) 𝑷 (𝑬 )
𝟏 𝟐

𝟏 𝟏𝟐 (𝟏𝟏)
(𝟒 ) 𝟏𝟐(𝟏𝟏) 𝟏𝟏
𝟓𝟏(𝟓𝟎)
= = =
𝟏 𝟏𝟐 (𝟏𝟏) 𝟑 𝟏𝟑 (𝟏𝟐) 𝟏𝟐(𝟏𝟏) + 𝟑𝟗(𝟏𝟐) 𝟓𝟎
(𝟒) + (𝟒)
𝟓𝟏(𝟓𝟎) 𝟓𝟏(𝟓𝟎)
11). Find the probability distribution of heads when three coins are tossed.
ANS: When 3 coins are tossed then S= {TTT, TTH, THT, THH, HTT, HTH, HHT, HHH }
X = No of heads obtained = 0, 1, 2, 3

X 0 1 2 3

P (X) 1/8 3/8 3/8 1/8

12). Find the probability distribution of Queens when two cards are drawn from pack of cards.
Total cards = 52 Queens = 4 Cards drawn =2 X = No of Queens = 0, 1, 2

X 0 1 2

P (X) 188/221 32/221 1/221

𝟒𝟖 𝟒𝟕
𝑷(𝑿 = 𝟎) = ̅̅̅̅
𝑸𝟏 ̅̅̅̅
𝑸𝟐 = (𝟓𝟐) (𝟓𝟏)
𝟒𝟖 𝟒 𝟒 𝟒𝟖 𝟒 𝟒𝟖
𝑷(𝑿 = 𝟏) = ̅̅̅̅
𝑸𝟏 𝑸𝟐 +𝑸𝟏 ̅̅̅̅
𝑸𝟐 = (𝟓𝟐) (𝟓𝟏) + (𝟓𝟐) (𝟓𝟏) = 𝟐 (𝟓𝟐) (𝟓𝟏)
𝟒 𝟑
𝑷(𝑿 = 𝟐) = 𝑸𝟏 𝑸𝟐 = (𝟓𝟐) (𝟓𝟏) ********
104

KENDRIYA VIDYALAYA SANGATHAN MUMBAI REGION


SUBJECT: MATHEMATICS (041) SAMPLE PAPER-01
MAX. MARKS : 80 CLASS : XII TIME: 3 Hrs.
General Instructions :
1. This Question paper contains - five sections A, B, C, D and E. Each section is compulsory. However,
there are
internal choices in some questions.
2. Section A has 18 MCQ’s and 02 Assertion-Reason based questions of 1 mark each.
3. Section B has 5 Very Short Answer (VSA)-type questions of 2 marks each.
4. Section C has 6 Short Answer (SA)-type questions of 3 marks each.
5. Section D has 4 Long Answer (LA)-type questions of 5 marks each.
6. Section E has 4 source based/case based/passage based/integrated units of assessment (4 marks
each) with sub parts.
Ma
S.No Part- A
rks
The domain of (1 − 𝑥) is
Q1 1
a) [-2, 0] b) [0, 2] c) (-2, 0) d) (0, 2)

|𝑥|
The function 𝑓(𝑥) = {1 , 𝑥 ≤ 0 𝑥 , 𝑥 > 0 is
a) Continuous at x = 0, and the value is 1
Q2 b) Continuous at x = 0, and the value is 0 1
c) Continuous at x = 0, and the value is -1
d) Discontinuous at x = 0.

If A = [𝑎𝑖𝑗 ], where 𝑎𝑖𝑗 = {−2 , 𝑖 ≠ 𝑗 0 , 𝑖 = 𝑗 , where A is a square matrix of


Q3 order 2 then A4 is 1
a) 4 A b) 4 I c) 16 A d) 16 I

Find the value of k < 0, so that the area of triangle with vertices (1, 0), (5, 0) and (1,
Q4 k) is 6 sq.units. 1
a) 3 b) -3 c) ± 3 d) -10

The function 𝑓(𝑥) = 𝑥 is ________________


Q5 a) always increasing on R b) always decreasing on R 1
c) Increasing on interval (0, ∞) d) Decreasing on interval (0, ∞)

For the following problem, the minimum value of Z is


Q6 Minimize Z = 3x + 12y, subject to x + 2y ≥ 6, 2x + y ≤ 6, x ≥ 0, y ≥ 0 1
a) 36 b) 9 c) 30 d) 72
𝑑𝑦
If 𝑦 = √𝑐𝑜𝑠 𝑐𝑜𝑠 𝑥 − 𝑦, then =
Q7 𝑑𝑥 1
𝑐𝑜𝑠𝑐𝑜𝑠 𝑥 𝑐𝑜𝑠𝑐𝑜𝑠 𝑥 𝑠𝑖𝑛𝑠𝑖𝑛 𝑥 𝑠𝑖𝑛𝑠𝑖𝑛 𝑥
a) − 2𝑦 + 1 b) − 2𝑦 − 1 c) − 2𝑦 + 1 d) − 2𝑦 − 1

If A = [0 𝑐𝑜𝑠 𝑐𝑜𝑠 𝑥 −𝑠𝑖𝑛 𝑠𝑖𝑛 𝑥 0 ], then for what value angle x, 2A2 = I
Q8 a) 𝑥 = − 𝜋/2 b) 𝑥 = − 𝜋/4 1
c) 𝑥 = 𝜋/2 d) 𝑥 = 𝜋/4.
105

Given that A is a square a matrix of order 3, such that IAI = λ > 0 and Iadj(A)I = 16,
Q9 then I5AI =_____ 1
a) 4 b) 20 c) 80 d) 500

If A = [2 12 1 6 ], then the matrix A(adj A) is


Q10 1
a) [6 − 12 − 1 2 ] b) [−6 12 1 − 2 ] c) [1 0 0 1 ] d) [0 0 0 0 ]

If |𝑎 ⃗⃗ = 60,
⃗⃗| = 5 and 𝑎⃗. 𝑏
⃗⃗| = 13, |𝑏 then find |⃗⃗⃗𝑎⃗ × 𝑏⃗⃗ |.
Q11 1
a) 5 b) 625 c) 25 d) 125.

3
𝑑2𝑦 𝑑𝑦 4
The degree of the differential equation ( ) = (𝑑𝑥 ) 1
Q12 𝑑𝑥 2
a) 2 b) 3 c) 4 d) 1
The direction cosines of line making equal angles with positive direction of co-
Q13 ordinate axes. 1
1 1 1 −1 −1 −1
a) , , b) , , c) 1, 1, 1 d) All of these
√2 √2 √2 √3 √3 √3

𝑥+2 𝑦−10 𝑧−8


The coordinates of the point where the line = = cuts x-axis.
Q14 2 5 4 1
a) (2, 0, 0) b) (6, 0, 0) c) (- 2, 0, 0) d) (-6, 0, 0)

A vector in the direction of vector 𝑎 = 2𝑖̂ − 𝑗̂ + 2𝑘̂ which has a magnitude of 8


Q15 units. 1
8 3
a) 8 𝑎 b) 24 𝑎 c) 𝑎 d). 𝑎
3 8
In the given probability distribution valid, the value of k is
X 0 1 2 3 4 5

Q16 P(X) k 2k 2k k 3k k1

a) 1 b) 0.1 c) 0.2 d) 0

If for the given graphical solution the


objective function Z = a x + b y has
value 60 at point (0, 3) and 24 at
point (2, 0), then (a, b) =
a) (20, 12)
Q17 1
b) (20, 24)

c) (12, 20)

d) (24, 20)

𝜋
𝜋 𝜋
Q 18 ∫02 𝑐𝑜𝑠 2 𝑥𝑑𝑥 = a) b) c)  d) 0 1
2 4
106

ASSERTION-REASON BASED QUESTIONS Question 19 and 20


In the following questions, a statement of assertion (A) is followed by a statement of
Reason (R). Choose the correct answer out of the following choices.
(a) Both A and R are true and R is the correct explanation of A.
(b) Both A and R are true but R is not the correct explanation of A.
(c) A is true but R is false.
(d) A is false but R is true.
Assertion (A) : The area of the region bounded by the curve y = x2 and the line x = 4 is
3/32 sq.u.

Reason (R) :
Q 19 1

𝑥 𝑦 𝑧 𝑥 𝑦 𝑧
Assertion (A) : The angle between the lines = = and = = is 90o.
2 5 4 1 −2 2
Q 20
Reason (R) : Skew lines are lines in different planes which are parallel and
intersecting.
SECTION B

11𝜋
Find the value of (𝑠𝑖𝑛 𝑠𝑖𝑛 ( 6 ) )
Q 21 OR 2
If 𝑓: 𝑅 → 𝑅is given by f(x)=3x-5, then show that f is bijective function.
𝜋
Find the value of k so that the function 𝑓 is continuous at 𝑥 = 2
Q 22 𝑘 𝑐𝑜𝑠 𝑐𝑜𝑠 𝑥 𝜋 𝜋 2
𝑓(𝑥) = { , 𝑖𝑓 𝑥 ≠ 5, 𝑖𝑓 𝑥 =
𝜋 − 2𝑥 2 2

Find a vector of magnitude 2 units which is perpendicular to the vectors 𝑖̂ − 𝑗̂ + 𝑘̂


and 𝑖̂ + 𝑗̂ + 𝑘̂
1. OR
Q 232. 2
𝜋 𝜋
If a unit vector 𝑎⃗ makes angles 3 with 𝑖̂, 4 with 𝑗̂ and an angle 𝜃 with 𝑘̂, then find the
value of 𝜃.
Sand is pouring from a pipe at the rate of 12𝑐𝑚3 /𝑠𝑒𝑐. The falling sand form
a cone on the ground such a way that the height of the cone is always one-sixth
Q 24 2
of the radius of the base. How fast is the height of the sand cone increasing
when the height is 4cm.

Q 25 Find the projection of the vector 7i + j +-4k on 2i + 6j + 3k. 2


107

SECTION C
𝑑𝑦 𝑥+ 𝑦
Find the general Solution of the differential =
𝑑𝑥 𝑥− 𝑦
Q26 OR 3
𝑑𝑦
Find a particular solution of the differential equation + 2 y tanx = sinx.
𝑑𝑥
1
Evaluate :∫ 𝑑𝑥 .
Q27 √5−4𝑥+𝑥2 3

Solve the Linear Programming Problem Garphically


Q28 Minimize Z = 3x + 12y, subject to x + 2y ≥ 6, 2x + y ≤ 6, x ≥ 0, y ≥ 0 3

1
Evaluate: ∫0 𝑥(1 − 𝑥)𝑛
OR
Q29 𝜋 3
By using the properties of definite integrals evaluate ∫04 𝑙𝑜𝑔 (1 +
𝑡𝑎𝑛 𝑡𝑎𝑛 𝑥)𝑑𝑥.
A box contains 4 white balls and 7 red balls. Two balls are drawn at random
without replacement. Find the probability that both balls drawn are of same
colour.
Q30 OR 3
Given that E and F are events such that P(E) = 0.7, P(F) = 0.6 and P(E∩F) = 0.4, find
𝐸
P(𝐹)

Q 31 Solve the integral ∫ 𝑒𝑥 (21 +


+
𝑠𝑖𝑛𝑠𝑖𝑛 (2𝑥)
𝑐𝑜𝑠𝑐𝑜𝑠 (2𝑥)
) 𝑑𝑥 3

SECTION D
Let R:NN, is a relation defined by R = {(x, y) : x – y is divisible by 2}. Check
whether R is an equivalence relation on set of natural number N. If so, find the
equivalence class of element 1.

Q 32 OR 5
Let A be a set of all non negative real numbers and 𝑓: 𝐴 → [15, ∞) be a function
defined as
𝑓(𝑥) = 4𝑥 2 + 12𝑥 + 15.Show that f is a bijective function.
Find the shortest distance between the lines whose vector equations are
𝑟 → = (𝑖̂ + 2𝑗̂ + 3𝑘̂ ) + 𝜆 (𝑖̂ − 3𝑗̂ − 2𝑘̂ )and
Q33 5
𝑟⃗⃗ = (4𝑖̂ + 5𝑗̂ + 6𝑘̂ ) + 𝜇 (2𝑖̂ + 3𝑗̂ + 𝑘̂ )
108

Find A-1, for A = [2 1 1 1 2 1 1 1 2 ]. Hence using matrix method, solve the following
equations: 2x + y + z = 2, x + 2y + z = 1 and x + y + 2z = 1.
OR
Q34 5
If A= [1 − 1 0 2 3 4 0 1 2 ] and B = [2 2 − 4 − 4 2 − 4 2 − 1 5 ], Evaluate the
product matrix BA. Hence solve the system of linear equations: x – y = 3, 2x + 3y +
4z = 17, y + 2z = 7.

Q35 Find the area enclosed by the parabola 3𝑥 2 = 4𝑦 and the line 2𝑦 = 3𝑥 + 12. 5

SECTION E
A builder makes a rectangular window surmounted
with semicircle (as shown in figure) so that more
light enters in the room. The perimeter of complete
window is 10 m. Let x represents diameter of
1x
Q36 semicircle and y represents length of rectangle. 4
Based on above information answer the following
questions.

The relation between variables is (a) 2 x + 2 y +  x = 10 (b) 2 x + 2 y +  y = 10


(i)
(c) 2 x + y + (/2) y = 10 (d) x + 2 y + (/2) x = 10
The area of the rectangular region A expressed as a function of x is
40𝑥 − 4𝑥 2 − 𝜋𝑥 2 40𝑥 − 4𝑥 2 − 𝜋𝑥 2
(a) (b)
8 4
(ii)
40𝑥 − 4𝑥 2 − 𝜋𝑥 2
(c) (d) None of these
2

The maximum of window is ______m2.


(iii) 50 30 10
(a) (b) (c) (d) None of these
𝜋+ 4 𝜋+ 4 𝜋+ 4
For maximum area, the length of the rectangle is __________ m.
(iv) 50 30 10
(a) (b) (c) (d) None of these
𝜋+ 4 𝜋+ 4 𝜋+ 4

An insurance company insured 5000 truck drivers, 3000 car drivers and 2000
scooter drivers. The chance of an accident of truck driver is 0.6, that of car driver is 1x
Q37
4
0.4 and that of scooter is 0.2. Let events A, B and C represents insured truck
109

drivers, car drivers and scooter drivers and event E represents that an insured
person meets an accident. Based on above information answer the following
questions

(i) P(E) = (a) 0.56 (b) 0.46 (c) 0.2 (d) None of these
P(AE) = (a) 15/23 (b) 6/23 (c) 2/23 (d) None of
(ii)
these
P(BE) = (a) 15/23 (b) 6/23 (c) 2/23 (d) None of
(iii)
these
P(CE) = (a) 15/23 (b) 6/23 (c) 2/23 (d) None of
(iv)
these

An open box is to be made out of a piece


of a cardboard measuring 24 cm x 24 cm,

Q 38 by cutting of equal squares of


x cm. from the corners and turning up the sides.

The volume function of box in terms of x is

(i) a) 4𝑥 3 − 96𝑥 2 + 576𝑥 b) 4𝑥 3 + 96𝑥 2 − 576𝑥


c) 2𝑥 3 − 48𝑥 2 + 288𝑥 d) 2𝑥 3 + 48𝑥 2 + 288𝑥
The rate of change of volume with respect to x is

(ii) a) 12(𝑥 2 + 16𝑥 − 48) b) 12(𝑥 2 − 16𝑥 + 48)


c) 6(𝑥 2 + 16𝑥 + 48) d) 6(𝑥 2 − 16𝑥 + 48)
The second derivative of volume with respect to x is
(iii)
a) 12(𝑥 − 8) b) 12(𝑥 + 8) c) 24(𝑥 − 8) d) 24(𝑥 + 8)
For maximum volume of box, the length of box is _________ cm.
(iv)
a) 4 b) 16 c) 12 d) 8

***********
110

Mathematics SAMPLE PAPER – 02 Class-XII

Time-3 hrs M.M.-80

General Instructions –
1. This Question paper contains - five sections A, B, C, D and E. Each section is
compulsory. However, there are internal choices in some questions.
2. Section A has 18 MCQ’s and 02 Assertion-Reason based questions of 1 mark each.
3. Section B has 5 Very Short Answer (VSA)-type questions of 2 marks each.
4. Section C has 6 Short Answer (SA)-type questions of 3 marks each.
5. Section D has 4 Long Answer (LA)-type questions of 5 marks each.
6. Section E has 3 source based/case based/passage based/integrated units of
assessment (4 marks each) with sub parts.
-------------------------------------------------------------------------------------------------------------
Section-A

Choose the correct answer

1. Let R be the relation in the set N given by R={(a,b,:a=b-2,b> 6}. choose correct answer
(A) (2,4)∈ 𝑅 (B) (3,8) ∈ 𝑅 (C)(6;8)∈ R (D) (8,7)∈R
2. Which of the following functions from Z into Z are bijection
(A) f(x) = x2 (B) f(x) = x + 2 (C) f(x) = 2x + 1 (D) f(x) = x2 + 1

3. If f: R->R be given by f(x)=(3-x3)1/3,then fof(X) is

(A) X1/3 (B) X3 (C) X (D) (3-X3)


𝜋
4.𝑥 = 10 for some x∈ 𝑅 then value of 𝑥 is

(A) 𝜋/5 (B) 2𝜋/5 (c) 3𝜋/5 (D) 4𝜋/5

5. Which of the following is the principal value of 𝑥


𝜋
(A) (-2 , 𝜋/2) (B) (0, 𝜋)-{𝜋/2} [C] [-𝜋/2, 𝜋/2] [D] [−𝜋/2, 𝜋/2] − {0}

If a = 2, b = 3 and a  b = 3, find the projecion of b on a.


6.

a) 18 b) 1 c) 2 /3 d) 3/2

If a and b are two vectors such that a  b = a  b , then what is the

7. angle between a and b ?

a) 𝜋/2 b) 𝜋/3 c) 𝜋/4 d) 𝜋/6

8.
Find  if ( 2i + 6 j + 14 k )  ( i −  j + 7k )=0
a) -2 b) -3 c) -1 d) N.O.T
111

9. The angle between the planes x+y+z=1 and x-2y+z=7 is

a) 𝜋/2 b) 𝜋/3 c) 𝜋/4 d) 𝜋/6

10. ∫ 𝑒 𝑥 (𝑐𝑜𝑠 𝑐𝑜𝑠 𝑥 −𝑠𝑖𝑛 𝑠𝑖𝑛 𝑥 )dx=

(A) ex 𝑐𝑜𝑠 𝑐𝑜𝑠 𝑥 + 𝑐 (B) ex𝑠𝑖𝑛 𝑠𝑖𝑛 𝑥 + 𝑐 (C) -ex𝑐𝑜𝑠 𝑐𝑜𝑠 𝑥 + 𝑐 (D)
none of these

11 . The slope of tangent to the curve x=t2+3t-8, y=2t2-2t-5 at the point (2-1)

(A) 22/7 (B) 6/7 (C) -6/7 (D) -6

12. The direction ratios of the line segment joining the points (1, 0, 0) and (0, 1, 1)

a) -1, -1, 1 b) -1, 1, 1 c) 1, -1, 1 d) None

13. Let A square matrix of order 3X3 , then IKAI is equal to

(A) K IAI (B) K2 IAI (C) K3 IAI (D) None of these

14. If |𝑥 2 18 𝑥 | =|6 2 18 6 | then x is equal to

(A) 6 (B) ±6 (C) -6 (D) 0

15. The area of triangle with vertices(-3,0),(3,0) and (0,k) is 9 square unit . The value of k will be:-

(A) 9 (B) -9 c) 6 D) None

16. The function f(x)=[x] where [x] denote the greater integer function ,is continuous at

(A) 4 (B) -2 (C) 1 (D) 1.5

17. If x = t2, y = t3, then d2y/dx2 is

(A) 3/2 (B) 3/4t (C) 3/2t (d) None

18. The value of a for which y= x2 +ax+25 touches the axis of x are

(A) 10 (B) -10 (C) ±10 (D) None of these

ASSERTION-REASON BASED QUESTIONS (19-20)

In the following questions, a statement of assertion (A) is followed by a statement of Reason (R).
Choose the correct answer out of the following choices.
(a) Both A and R are true and R is the correct explanation of A.
(b) Both A and R are true but R is not the correct explanation of A.
(c) A is true but R is false. (d) A is false but R is true.
112

−1 −1
19. Assertion A: cos x  sin x, for all x  [−1,1]

Reason (R): cos−1 x is decreasing function in [-1,1]

20. Assertion (A): For the curve y = tanx , the tangent and normal exists at a point (0, 0).
Reason (R): Tangent and Normal lines are x – y = 0 and x + y = 0.
Section-B
3𝜋
21. Find the principal value of 𝑠𝑖𝑛 𝑠𝑖𝑛 5
).

k 1 
 
22. Find the value of k if the matrix  2 −4  is singular

23. Mean value of theorem for the function f(x)=x2 in the interval [2,4] be verified. Find the value of
C.

→    →   

24. Find the projection of a = 2 i − j + k on b = i − 2 j + k


𝑥 −𝑥
25. Evaluate ∫ 𝑠𝑖𝑛𝑠𝑖𝑛 𝑥 𝑐𝑜𝑠𝑐𝑜𝑠 𝑥
dx

Section-C

26. Prove that the relation R in the set A = {1,2,3,4,5} given by R = {(a, b) : a − b is even} , is an
equivalence
Relation.
(OR)

Consider f : R+ → [−5, ) given by f ( x) = 9 x + 6 x − 5 show that f one-one and onto.


2

27. Find the shortest distance between the following skew lines

r = ( − 1)iˆ + ( + 1) ˆj − (1 +  ) kˆ
r = (1 −  )iˆ + (2 − 1) ˆj + (  + 2)kˆ

28. Three friends go for coffee. They decide who will pay the bill, by each tossing a coin and then
letting the “odd person” pay. There is no odd person if all three tosses produce the same result. If
there is no odd person in the first round, they make a second round of tosses and they continue to do
so until there is an odd person. What is the probability that exactly three rounds of tosses are made?
OR
Find the mean number of defective items in a sample of two items drawn one-by-one without
replacement from an urn containing 6 items, which include 2 defective items. Assume that the items
are identical in shape and size.
113

2𝑥+1
29. Differentiate with respect to x , 1+4𝑥
)
Or

𝑑2 𝑦
If x=a(𝑐𝑜𝑠 𝑐𝑜𝑠 𝑡 + 𝑡 𝑠𝑖𝑛 𝑠𝑖𝑛 𝑡 ), y=a(𝑠𝑖𝑛 𝑠𝑖𝑛 𝑡 − 𝑡 𝑐𝑜𝑠 𝑐𝑜𝑠 𝑡 ) Find 𝑑𝑥 2

30. Solve xdy − ydx = x + y dx


2 2

1 𝑥2
31. Evaluate ∫ dx 0r ∫ 2 dx
𝑥−𝑎)𝑐𝑜𝑠𝑐𝑜𝑠 (𝑥−𝑏) (𝑋 +1)(𝑋2 +4)

Section –D

32. Given three identical boxes I, II and III each containing two coins. In box I, both coins are gold
coins, in box II, both are silver coins and in box III, there is one gold and one silver coin. A person
chooses a box at random and takes out a coin. If the coin is of gold, what the probability that the
other coin in the box is also of gold?

33. An open box with a square base is to be made from a given quantity of cardboard of area c 2.

c3
Show that the maximum volume of the box is 6 3 .

(OR)

Show that the height of a closed right circular cylinder of a given volume and least surface is equal
to its diameter.

34. Find the area of the region enclosed by circles x + y = 4and ( x − 2) + y = 4 .


2 2 2 2

(OR)

Find the area of the region (x, y ): x + y  1  x + y }.


2 2

35. A farmer mixes two brands P and Q of cattle feed. Brand P, costing Rs. 250 per bag contains 3
units of nutritional element A, 2.5 units of element B and 2 units of element C. Brand Q costing Rs 200
per bag contains 1.5 units of nutritional elements A, 11.25 units of element B, and 3 units of element
C. The minimum requirements of nutrients A, B and C are 18 units, 45 units and 24 units respectively.
Determine the number of bags of each brand which should be mixed in order to produce a mixture
having a minimum cost per bag? What is the minimum cost of the mixture per bag?
114

Section – E

36. Suppose a dealer in rural area wishes to purchase a number of sewing


machines. He has only Rs. 5760 to invest and has space for atmost 20
items for storage. An electronic sewing machine costs him Rs. 360 and a
manually operated sewing machine Rs. 240. He can sell an electronic
sewing machine at a profit of Rs. 22 and a manually operated sewing
machine at a profit of Rs.18.
Based on the above information, answer the following questions.

(a) x+y≥ 0 (b) x+y< 0

(c) x+y> 0 (c) x+y≤ 0


A) Let x and y denote the number of electronic sewing machines and manually operated sewing
machines purchased by the dealer. If it is assumed that the dealer purchased at-least one of
the given machines then:
B) Let the constraints in the given problem is represented by the following inequalities:x+y≤20;
360x+240y≤5760 and x,y≥0. Then which of the following point lie in its feasible region.

(a) (0,24) (b) (8,12)


(c) (20,2) (d) None of these

C) If the objective function of the given problem is maximize Z = 22x+18y, then its optimal value
occur at:

(a) (0,0) (b) (16,0)


(c) (8,12) (d) (0,2)
D) Suppose the following shaded region APDO, represent the feasible region corresponding to
mathematical formulation of the given problem.
Then which of the following represent the coordinates of one of its corner points.

(a) (0,24) (b) (12,8)


(c) (8,12) (d) (6,14)

37. There are two antiaircraft guns, named as A and B. The probabilities that the shell fired from
them hits an airplane are 0.3 and 0.2 respectively. Both of them fired one shell at an airplane at the
same time.

(i) What is the probability that the shell fired from exactly one of them hit the plane?
115

(ii) If it is known that the shell fired from exactly one of them hit the plane, then what is the
probability that it was fired from B?

38. An architect designs a building for a multi-national company. The floor consists of a rectangular
region with semicircular ends having a perimeter of 200m as shown below:

Design of Floor Building

Based on the above information answer the following:


A) If x and y represent the length and breadth of the rectangular region, then the relation
between the variables is
a. x +  y = 100 b. 2x +  y = 200
c.  x + y = 50 D.x + y = 100
B) The area of the rectangular region A expressed as a function of x is
2 1
a. (100𝑥 − 𝑥 2 ) b. (100𝑥 − 𝑥 2 )
𝜋 𝜋
𝑥 2
c. 𝜋
(100 − 𝑥) d. 𝜋𝑦2 + 𝜋 (100𝑥 − 𝑥2 )

C) The maximum value of area A is


𝜋 3200
a. 3200 𝑚2 b. 𝑚2
𝜋
5000 1000
c. 𝜋 𝑚2 d. 𝜋 𝑚2
D) The CEO of the multi-national company is interested in maximizing the area of the whole floor
including the semi-circular ends. For this to happen the valve of x should be

a. 0 m b. 30 m
c. 50 m d. 80 m
********************
116

CLASS XII MATHEMATICS SAMPLE PAPER-03

Time Allowed : 3 HOURS Maximum Marks :80

General Instructions :

1. This Question Paper Contains –five sections A,B,C,D and E. Each Section is
compulsory.However, there are internal choices in some questions.

2. Section A has 18 MCQ’s and 02 Assertion-Reason based questions of 1 mark each.

3.Section B has 5 very short Answer (VSA)- type question of 2 marks each.

4.Section C has 6 short answer (SA)- type questions of 3 marks each.

5.Section D has 4 Long Answer (LA)- type questions of 5 marks each.

6. Section E has 3 Source Based/ case based/Passage based/integrated units of


assessment (4 marks each) with sub parts.

SECTION A Multiple Choice Questions Each question carries 1 mark.

1. If [𝑥 + 𝑦 𝑥 − 𝑦 ] = [2 1 4 3 ] [1 − 2 ], then (x, - y) is

A) (1,1) B) (1,-1) C) (-1,1) D) (-1,-1)

2. If A is a 3X3 invertible matrix, then what will be the value of k if det(A -1) = (detA)k

A) 1 B) -1 C) 2 D) -2

3. if 𝑎⃗ = 2 𝑖̂ + 4 𝑗̂ - 𝑘̂ and 𝑏⃗⃗ = 3𝑖̂ - 2 𝑗̂ +𝛼 𝑘̂ are perpendicular to each other then the value
of 𝛼 is

A) 1 B) -1 C) 2 D) -2

4. Find the value of k if so that the given function is continuous at x = 0?

𝑠𝑖𝑛5𝑥
F(x) = {
3𝑥
, 𝑖𝑓 𝑥 ≠ 0 𝑘 ,𝑥 = 0
2 4 5 8
A) B) C) D)
3 3 3 3
𝜋
√𝑠𝑖𝑛𝑥
5. ∫02 dx =
√𝑐𝑜𝑠𝑥 + √𝑠𝑖𝑛𝑥

𝜋 𝜋 𝜋 𝜋
A) B) C) D)
3 4 2 6

𝑑 𝑑𝑦 3
6. The sum of the order and degree of the differential equation [( ) ] = 0 is
𝑑𝑥 𝑑𝑥

A) 2 B)1 C)3 D)4


117

7. The Feasible region for an LPP is shown in the figure.

Let F = 3x - 4y be the objective function, then Maximum


value of F is

A) 0 B) 8 C) 12 D)-18

8. The Projection of 𝑎⃗ = 𝑖̂ + 3 𝑗̂ + 𝑘̂ along 𝑏⃗⃗ = 2 𝑖̂ - 3 𝑗̂


+6 𝑘̂ is

1 4 1 2
A) B) C) D)
3 7 7 3
1
9.∫ 𝑒𝑥 ( 𝑙𝑜𝑔𝑥 + 𝑥
) 𝑑𝑥 =

1 1
A) 𝑒𝑥 B) 𝑒 𝑥 logx C) D) logx,
𝑥 𝑥

10. Value of |𝑠𝑖𝑛200 − 𝑐𝑜𝑠200 𝑠𝑖𝑛700 𝑐𝑜𝑠700 | is

A) 1 B) -1 C)2 D)-2

11. The feasible region for an LPP is always a …………………. Polygon

A) Concave B) Convex C) Regular D) None

12. If A is a square matrix and |𝐴| = 2, then the value of |𝐴𝐴′|, where A’ is the transpose of
the matrix A

A) 2 B)3 C)4 D)5

13.If A is a square matrix of order 3 and |𝐴|= 4 then |𝑎𝑑𝑗𝐴| =

1
A) 4 B) 16 C)24 D)
4

14. Given two independent events A and B such that P(A) = 0.3 and P(B) = 0.6 then the
value of P(A and Not B) is

A) 0.14 B) 0.12 C) 0.16 D) 0.18

𝑑𝑦 1 2
15. The integrating factor of the differential equation : + y= is
𝑑𝑥 𝑥𝑙𝑜𝑔𝑥 𝑥

1
A) 1 B) x C) logx D)
𝑥
118

16. If f(1) = 4 and f’(1) = 2 then the value of log f(ex) with respect to x at the point x = 0

1 1 1 1
A) B) C) D) 5
2 3 4

17. For what value of a the vectors 2 𝑖̂ -3 𝑗̂ + 4 𝑘̂ and a 𝑖̂ + 6 𝑗̂ - 8 𝑘̂ are collinear?

A) -2 B) -3 C)-4 D)-5

𝑥−1 𝑦−2 𝑧−3 𝑥−1 𝑦−1 𝑧−6


18. If the lines = = and = = are perpendicular, then the
−3 2𝑘 2 3𝑘 1 −5
value of k is

7 −7 10 −10
A) B) C) D)
10 10 7 7

ASSERTION REASON BASED QUESTIONS

In the following questions, a statement of assertion (A) is followed by a statement of Reason


( R). Choose the correct answer out of the following choices.

A) Both A and R are true and R is the correct explanation of A.

B) Both A and R are true but R is not the correct explanation of A.

C) A is true but R is false D) A is false but R is true

𝜋 −1 1
19. Assertion (A) : The value of Cos ( + 𝑠𝑖𝑛−1 ( )) =
2 2 2

Reason ( R) : 𝑠𝑖𝑛−1 (−𝑥 ) = −𝑠𝑖𝑛−1 𝑥


𝑥+1 𝑦−2 𝑧+3
20. Assertion (A) : The angle between the lines = = and
2 5 4
𝑥−1 𝑦+2 𝑧−3
= = is 900
1 2 −3

Reason (R) : Skew lines are lines in different planes which are parallel and intersecting.

SECTION B

This section comprises of very short answer type question (VSA) of 2 marks each
3𝜋
21. Find the value of 𝑠𝑖𝑛−1 (𝑠𝑖𝑛 )
5

(OR)

Let A = {1, 2, 3} write all one-one functions on A


119

22. A stone is dropped in to a quiet lake and waves move in circles at a speed of 4cm per
second. At the instant, when radius of the circular wave is 10cm, how fast is the enclosed
area increasing?

23. Find the direction Cosines of the line passing through the points (-2, 4, -5) and

(1, 2, 3).

OR

Show that the points A (2, 3, -4) B (1, -2, 3) and C(3, 8, -11) are collinear.

24. Find the derivative of y = cosx3 sin2x w.r.t x


25. Find 𝛼 and 𝛽 if (2𝑖̂ + 6 𝑗̂ + 27𝑘̂ )𝑋 ( 𝑖̂ + 𝛼 𝑗̂ + 𝛽𝑘̂ ) = ⃗0⃗

SECTION C

This section comprises of short answer type questions (SA) of 3 marks each)

2𝑥2 +3
26. Find: ∫ 𝑑𝑥
𝑥2 +5𝑥+6

27. The random variable X has a probability distribution p(X) of the following form, where k
is some number:

P(X) = {𝑘 𝑖𝑓 𝑥 = 0 2𝑘, 𝑖𝑓 𝑥 = 1 3𝑘, 𝑥 = 2 0, 𝑜𝑓 𝑜𝑡ℎ𝑒𝑟𝑤𝑖𝑠𝑒

a) Determine the value of k. b) Find P(X≥2) c) P( X<2)

OR

Two numbers are selected at random (without replacement) from the first six positive
integers. Let X denote the larger of the two numbers obtained. Find mean of the distribution.

3
𝜋 𝑥𝑠𝑖𝑛𝑥
28. Evaluate ∫ 2 |𝑥𝑐𝑜𝑠𝜋𝑥 |dx OR Evaluate ∫ 𝑑𝑥
0 0 1+𝑐𝑜𝑠2 𝑥
𝑑𝑦 𝑦
29. Solve the differential equation : x − 𝑦 + 𝑥 𝑠𝑖𝑛 ( )=0
𝑑𝑥 𝑥

OR Solve the differential equation: (1+ y2 ) ( 1+logx) dx + x dy = 0

30. Maximise: Z= 3x + 2y subject to the constraints:

x + 2y≤ 10, 3x + y ≤ 15 , 𝑥, 𝑦 ≥ 0

31. Find ∫
𝑥 dx
2
(𝑥−1) (𝑥+2)
120

SECTION D

This section Comprises of long answer type questions (LA) of 5 marks each

32. Find the area of the region included between the parabola 4y = 3x 2 and the line

3x - 2y +12 = 0

33. Define the relation R in the set NXN as follows:

For (a, b), (c, d) ∈ NXN , (a, b)R (c, d) iff a + d = b + c.

Prove that R is an equivalence relation in NXN.

34. Use product [1 − 1 2 0 2 − 3 3 − 2 4 ] [−2 0 1 9 2 − 3 6 1 − 2 ] to solve the system of


equations

x – y + 2z = 1 , 2y – 3z = 1, 3x – 2y + 4z = 2

35. An insect is crawling along the line 𝑟⃗ = 6 𝑖̂ + 2 𝑗̂ +2 𝑘̂ + λ( 𝑖̂ - 2 𝑗̂ +2 𝑘̂) and another


insect is crawling along the line 𝑟⃗ = -4 𝑖̂ - 𝑘̂ + 𝜇 ( 3 𝑖̂ -2 𝑗̂ - 2𝑘̂ ). At what points on the lines
should they reach so that the distance between them is the shortest? Find the possible
shortest distance between them.

OR

At a certain instant of time, if the missile is above the sea level, where the equation of the
imaginary line on the surface of the water is 𝑟⃗ = - 𝑖̂ + 3 𝑗̂ + 𝑘̂ + λ( 2𝑖̂ +3 𝑗̂ - 𝑘̂) and the
position of missile at the instant of time is (5, 4, 2) , then find the image of the position of the
rocket in the sea?

SECTION E

This SECTION comprises of 3 case study passage based questions of 4 marks each with
two sub parts. First two case study questions have three sub parts i),ii),iii) of marks 1,1,2
respectively.

The third case study question has two sub parts of 2 marks each.
121

36. Case study 1: An Architect designs a garden in a society. The garden is in the shape of
of rectangle inscribed in a circle of radius of 10m.

i) IF 2x and 2y represents the length and breadth of the


rectangular garden, then find the relation between the
variables.

ii) Find the expression for area of the green grass shaped as
rectangle in single variable

iii) Find the maximum area of the garden?

37. Case study 2: Read the following passage and


answer the questions given below

The temperature of a person during an illness is given


by F(X) = 10 +mx – 2x2 ,
−2
i) If ( 3 ) is the critical point of the function, then find the
value of constant m.

ii) find f’’(x)

iii) Find the intervals in which the function is strictly increasing and strictly decreasing

38). CASE STUDY 3: A biased die is tossed and respective probabilities for various faces
to turn up are the following

Face 1 2 3 4 5 6

Probability 0.1 0.24 0.19 0.18 0.15 k

a) What is the value of k?

b) If a face showing an even number has turned


up, then what is the probability that it is the face
with 2 or 4?

***********************
122

CLASS XII MATHEMATICS SAMPLE PAPER - 04

Time Allowed: 3 Hours Maximum Marks: 80

General Instructions:

1. This Question paper contains - five sections A, B, C, D and E. Each section is compulsory.

2. Section A has 18 MCQ’s and 02 Assertion-Reason based questions of 1 mark each.

3. Section B has 5 Very Short Answer (VSA)-type questions of 2 marks each.

4. Section C has 6 Short Answer (SA)-type questions of 3 marks each.

5. Section D has 4 Long Answer (LA)-type questions of 5 marks each.

6. Section E has 3 source based/case based/passage based/integrated units of assessment


(4 marks each) with sub parts.

SECTION A (Multiple Choice Questions) Each question carries 1 mark

1. If 𝐴[4 𝑥 + 2 2𝑥 − 3 𝑥 + 1 ] is symmetric, then the value of x is:


(A) 2 (B) 3 (C) – 1 (D) 5
2. If A is a square matrix of order 3, such that |𝑎𝑑𝑗𝐴| = 225, then |𝐴′| is equal to:
(A) 15 (B) – 15 (C) ±15 (D) 0
⃗⃗⃗⃗ = 2𝑖̂ − 𝑗̂ + 𝑘̂ on 𝑏
3. The projection of 𝑎 ⃗⃗⃗⃗ = 𝑖̂ − 2𝑗̂ + 𝑘̂ is:
5 5 6
(A) (B) − (C) (D) 𝑖̂ + 𝑗̂ + 𝑘̂
√6 √6 √5
1
4. Let 𝑓(𝑥) = {(𝑐𝑜𝑠𝑥)𝑥 , 𝑥 ≠ 0𝑘 , 𝑥=0 is continuous at 𝑥 = 0, then the
value of 𝑘 is:
(A) 0 (B) 1 (C) – 1 (D) 𝑒
𝑒 𝑑𝑥
5. The value of 𝐼 = ∫1 is:
𝑒 𝑥(𝑙𝑜𝑔𝑥)1/3

1
(A) e (B) 𝑒 (C) 0 (D) 1

𝑑2 𝑦 3 𝑑𝑦
6. The sum of order and the degree of the differential equation 𝑑𝑥 2 + √𝑑𝑥 + (1 + 𝑥) = 0 is:

(A) 3 (B) 5 (C) 2 (D) cannot determined


7. The graph of the inequality 6𝑥 + 8𝑦 ≤ 24 is:
(A) Half plane that contains the origin
(B) Half plane that neither contains the origin nor the points of the line 6𝑥 + 8𝑦 = 24.
(C) Whole XOY – plane excluding the points on line 6𝑥 + 8𝑦 = 24.
(D) None of the above.
123

𝑎 = 5, |⃗⃗⃗⃗
8. If |⃗⃗⃗⃗| 𝑎 × ⃗⃗⃗⃗
𝑏 | = 13 and |⃗⃗⃗⃗ ⃗⃗⃗⃗ ∙ ⃗⃗⃗⃗
𝑏 | = 25, then 𝑎 𝑏 is equal to:

(A) 12 (B) 5 (C) 13 (D) 60


3𝜋/4 √𝑡𝑎𝑛𝑥
9. The value of 𝐼 = ∫−𝜋/4 𝑑𝑥 is:
1+√𝑡𝑎𝑛𝑥
𝜋 𝜋
(A) 2 (B) 4 (C) 𝜋 (D) 0

10. If A is a square matrix of order 3 such that |𝐴| = 2 then |𝑎𝑑𝑗(𝑎𝑑𝑗𝐴)| is:
(A) 4 (B) – 4 (C) 16 (D) – 16
11. The corner points of the feasible region determined by the following system of linear
inequalities:
2𝑥 + 𝑦 ≤ 10, 𝑥 + 3𝑦 ≤ 15, 𝑥, 𝑦 ≥ 0 are (0, 0), (3, 4), (5, 0). Let 𝑍 = 𝑝𝑥 + 𝑞𝑦, where 𝑝, 𝑞 >
0. Condition on p and q so that the maximum of Z occurs at both (3, 4) and (0, 5) is :
(A) 𝑝 = 𝑞 (B) 𝑝 = 2𝑞 (C) 𝑝 = 3𝑞 (D) 𝑞 = 3𝑝
12. Given 2𝑥 − 𝑦 + 2𝑧 = 2, 𝑥 − 2𝑦 + 𝑧 = −4 and𝑥 + 𝑦 + 𝛼𝑧 = 4, then the value of 𝛼 such that
the given system of equation has no solution is:
(A) 3 (B) 1 (C) 0 (D) – 3
13. The maximum value of |1 1 1 1 1 + 𝑠𝑖𝑛𝜃 1 1 1 1 + 𝑐𝑜𝑠𝜃 | is:
1
(A) 2 (B) 1 (C) – 1 (D) 0

14. If events A and B are independent, P(A) = 0.35, P(A ∪ B) = 0.60, then P(B) is:
(A) 0.25 (B) 0 (C)0.95 (D) None of these
𝑑𝑥
15. The integrating factor + 𝑥 𝑐𝑜𝑡𝑦 = 𝑐𝑜𝑠𝑦 is:
𝑑𝑦

(A) 𝑠𝑖𝑛𝑥 (B) 𝑠𝑖𝑛𝑦 (C) 𝑙𝑜𝑔𝑠𝑖𝑛𝑦 (D) 𝑒 𝑠𝑖𝑛𝑦


𝜋
16. If |𝑐𝑜𝑠𝑥 − 𝑠𝑖𝑛𝑥|, then 𝑓′ ( 3 ) is equal to:
2 √3+1
(A) √3 + 1 (B) (C) (D) None of these
√3+1 2

17. The value of | 𝑥 ⃗⃗⃗⃗, ( ⃗⃗⃗⃗


⃗⃗⃗⃗| if for a unit vector 𝑎 𝑥 −𝑎 ⃗⃗⃗⃗) ∙ ( 𝑥
⃗⃗⃗⃗ + 𝑎
⃗⃗⃗⃗) = 35 is:
(A) 18 (B) 16 (C) 7 (D) 6
18. The ratio in which the line joining the points (4, 4, −10) and (−2, 2, 4) divided by the yz –
plane is:
(A) 2 : 1 internally (B) 2 : 1 externally (C) 1 : 3 internally (D) 3 : 1 internally
(I) Assertion and Reason Based Questions
Directions: Each of these questions contains two statements: Assertion (A) and Reason (R).
Each of these questions has four alternative choices, any one of which is the correct answer. You
have to select one of the codes (a), (b), (c) and (d) given below.
(a) A is true, R is true; R is correct explanation for A.
(b) A is true, R is true; R is not correct explanation for A.
(c) A is true; R is false. A is false; R is true.
124

√3 𝜋
19. Assertion (A) : The value of 𝑠𝑖𝑛−1 {𝑐𝑜𝑠 (𝑠𝑖𝑛−1 ( 2 ))} is 6

Reason (R) : The principal value branch of 𝑐𝑜𝑠 −1 𝑥 is [0, 𝜋].


𝑥−5 2−𝑦 1−𝑧 𝑥 2𝑦+1 1−𝑧
20. Assertion (A) : The value of k for which lines 5𝑘+2 = = ; = = is 1.
5 −1 1 4𝑘 −3

𝑎1 𝑎2 +𝑏1 𝑏2 +𝑐1 𝑐2
Reason (R) : The angle between two lines is given by 𝜃 = 𝑐𝑜𝑠 −1 ( )
√𝑎12 +𝑏12 +𝑐12 √𝑎22 +𝑏22 +𝑐22

SECTION B
This section comprises of very short answer type-questions (VSA) of 2 marks each
21. Find the domain of 𝑠𝑒𝑐 −1 (2𝑥 + 1).
OR
Show that 𝑓: 𝑅 ⟶ 𝑅, given by 𝑓(𝑥) = 𝑥 − [𝑥] is neither one – one nor onto.
̂ ; 𝑖̂ + 𝜆𝑗̂ − 𝑘
22. Find the value of 𝜆 for which the vectors 𝜆𝑖̂ + 𝑗̂ + 2𝑘 ̂ 𝑎𝑛𝑑 2𝑖̂ − 𝑗̂ + 𝜆𝑘
̂ are

coplanar.
⃗⃗⃗⃗ = 3𝑖̂ − 2𝑗̂ + 2𝑘̂ and ⃗⃗⃗⃗
OR The vectors 𝑎 𝑏 = −𝑖̂ − 2𝑘̂ are the adjacent sides of a
parallelogram. Find the angle between its diagonals.
23. If x and y are the sides of two squares such that 𝑦 = 𝑥 − 𝑥 2 . Find the change of the area of
Second Square with respect to the area of the first square.
𝑑𝑦 1−𝑦 2
24. If √1 − 𝑥 2 + √1 − 𝑦 2 = 𝑎(𝑥 − 𝑦), then prove that 𝑑𝑥 = √1−𝑥 2 .

⃗⃗⃗⃗ are such that |𝑎 2


⃗⃗⃗⃗| = and 𝑎 ⃗⃗⃗⃗ is a unit vector. Find the angle
25. Vectors 𝑎
⃗⃗⃗⃗ and 𝑏 ⃗⃗⃗⃗| = √3, |𝑏
3
⃗⃗⃗⃗ × 𝑏

⃗⃗⃗⃗ and ⃗⃗⃗⃗


between 𝑎 𝑏.
SECTION C (short answer type questions (SA) of 3 marks each)
26. Evaluate: ∫ √5 − 4𝑥 − 𝑥2 𝑑𝑥

27. An insurance company charges from a man of age 50 an annual premium of ` 15 on a policy
of ` 1000. If the death rate is ` 6 per thousand per year for this age group. What is the expected
gain for the insurance company?
OR
1 1 1
A problem in Statistics is given to three students, whose chances of solving it are 2 , 3 and 4

respectively. What is the probability that only one of the three students solves it correctly?
1 1
28. Evaluate: ∫0 𝑙𝑜𝑔 |𝑥 − 1| 𝑑𝑥. OR
3.5
Evaluate: ∫0.2 [𝑥] 𝑑𝑥, where [𝑥] is greatest integer function.
29. Solve the differential equation:
𝑑𝑦
(1 + 𝑦 2 )𝑑𝑥 = (𝑡𝑎𝑛−1 𝑦 − 𝑥)𝑑𝑦. OR Solve : 𝑥 = 𝑦(𝑙𝑜𝑔𝑦 − 𝑙𝑜𝑔𝑥 + 1).
𝑑𝑥
125

30. Solve the LPP graphically: Maximise, 𝑍 = 100𝑥 + 150𝑦 subject to constraints 𝑥 + 3𝑦 ≤ 50,
2𝑥 + 2𝑦 ≤ 85, 𝑥 ≥ 0, 𝑦 ≥ 0.
1
31. Evaluate: ∫ 𝑥(𝑥𝑛 +1)
𝑑𝑥.

SECTION D (long answer-type questions (LA) of 5 marks each)


32. Let 𝑁 denote the set of all natural numbers and R be the relation on 𝑁 × 𝑁 defined by
(𝑎, 𝑏)𝑅(𝑐, 𝑑) if 𝑎𝑑(𝑏 + 𝑐) = 𝑏𝑐(𝑎 + 𝑑). Show that R is an equivalence relation.
OR
Consider 𝑓: 𝑅+ → [−5, ∞) given by 𝑓(𝑥) = 9𝑥 2 + 6𝑥 − 5. Show that f is a bijective
function. Also find pre – image of 10.
33. Draw a rough sketch and find the area of the region bounded by the curve 4𝑦 = 3𝑥 2 and the
line 2𝑦 = 3𝑥 + 12 by using method of integration.
34. During war between two countries one of the country launch a missile towards another along
𝑥−4 𝑦+3 𝑧+1
the path = = . On detecting by Radar another country launch a Ballistic missile to
1 −4 7
𝑥−1 𝑦+1 𝑧+10
destroy the missile launched by the first one along the path = = . Show that the
2 −3 8

Ballistic missile will destroy the missile. Also find the point in air where they collide with
each other.
OR
𝑥 𝑦−1 𝑧−2
Find the image of the point (1, 6, 3) in the line 1 = =
2 3

35. If [ 2 2 −4 −4 2 −4 2 −1 5 ] and 𝐵 = [1 − 1 0 2 340 1 2 ], then find BA


and use this to solve the system of equations 𝑦 + 2𝑧 = 7, 𝑥 − 𝑦 = 3, 2𝑥 + 3𝑦 + 4𝑧 = 17.
SECTION E
(This section comprises of 3 case-study/passage-based questions of 4 marks each with two sub-
parts. First two case study questions have three sub – parts (i), (ii), (iii) of marks 1, 1, 2
respectively. The third case study question has two sub – parts of 2 marks each.)
126

36. A man is running a racecourse observes that the sum of distances from two fixed flag posts
from him is 10 metres and the distance between the flag posts is 8 metres.
A line 3𝑥 + 5𝑦 = 15 is drawn to intersect the race course at two distinct points.

Based upon above situation answer the following questions:

(i) Write the equation of the race course.


(ii) Find the area enclosed by the race course.
(iii) Find the area of the larger portion between the line and the race course.
OR
(iii) Find the area of the smaller portion between the line and the race course.

37. Case Study 2

To promote vaccination in a city the municipal corporation decide to erect advertising boards in some
places like bus stand, railway station, malls, and other crowded places in the city. For this, the top and
bottom margins of the poster should be 9 cm and the side margin be 6 cm. Also the area for printing
the message should be 864 square centimetres. 𝑥 cm be the width and 𝑦 cm be the height of the
poster. Based upon the above situation answer the following questions:

(i) Write the relation between 𝑥 and 𝑦.


(ii) Write Area of poster in terms 𝑦.
(iii) Find the height of the poster, so that the area of the poster is minimum is:
OR (iii)The width of the poster, so that the area of the poster is minimum is:
127

38. Case Study Question:


During lock down in 2019 some family members of a society decided to entertain members of their
family to make the stress of pandemic light. For this they decide to play the game of playing cards.
They realise that a card from the pack of cards is missing. From the remaining cards, two cards are
drawn at random and found to be both spades. The leader of the group gets curios and starts thinking
of possibilities as under.

Based on this information answer the following questions:

(i) Find the probability of getting 2 spades, given that a card of spade is missing.
(ii) Find the probability of the missing card being a spade, given that 2 spades are drawn
from the remaining card.
**************
128

CLASS XII MATHEMATICS SAMPLE PAPER-05

Time Allowed: 3 Hours Maximum Marks: 80

1. This Question paper contains - five sections A, B, C, D and E. Each section is


compulsory. However, there are internal choices in some questions.
2. Section A has 18 MCQ’s and 02 Assertion-Reason based questions of 1 mark each.
3. Section B has 5 Very Short Answer (VSA)-type questions of 2 marks each.
4. Section C has 6 Short Answer (SA)-type questions of 3 marks each.
5. Section D has 4 Long Answer (LA)-type questions of 5 marks each.
6. Section E has 3 source based/case based/passage based/integrated units
of assessment (4 marks each) with sub parts.

SECTION A (Multiple Choice Questions) Each question carries 1 mark

1. If 𝐴 = [2 − 1 3 1 ] then 𝐴𝐴𝑇 =
(a) [5 5 10 5 ] (b) 5[1 1 1 2 ] (c)5𝐼2 (d)None
2. If A is a square matrix such that |𝐴| = 8 then the value of |(𝐴𝑇 )−1 | is
(a) 1/8 (b)8 (c)-1/8 (d)-8
3. If 𝑎⃗ , 𝑏⃗⃗ are diagonals of a rhombus then
(a)𝑎⃗ × 𝑏⃗⃗ = 0 (b) 𝑎⃗. 𝑏⃗⃗ = 0 (c) 𝑎⃗. 𝑏⃗⃗ = 1 (d) 𝑎⃗ × 𝑏⃗⃗ = 𝑎⃗
1
4. The value of k which make 𝑓𝑥) = {𝑥 𝑠𝑖𝑛 ( ) , 𝑥 ≠ 0 𝑘, 𝑥 = 0 continuous at x=0, is
𝑥

(a) -1 (b)1 (c) 0 (d ) none of these


5. If 𝑓 ′ (𝑥) =𝑡𝑎𝑛 𝑡𝑎𝑛 𝑥 then 𝑓(𝑥) =
(a) 𝑙𝑜𝑔 𝑙𝑜𝑔 𝑠𝑖𝑛 𝑠𝑖𝑛 𝑥 + 𝑐 (b) 𝑥 + 𝑐 (c) − 𝑙𝑜𝑔 𝑙𝑜𝑔 𝑐𝑜𝑠 𝑐𝑜𝑠 𝑥 + 𝑐 (d ) None
2
𝑑2 𝑦 𝑑𝑦
6. If m and n are respectively order and degree of differential equation ( 2 ) − ( ) = 𝑦 3 then
𝑑𝑥 𝑑𝑥

m+n=
(a) 1/2 (b)2 (c) 3 (d ) 4
7. The solution set of the inequation 2𝑥 + 𝑦 > 5 is
(a) half plane that contains the origin
(b) open half plane not containing the origin
(c) whole xy-plane except the points lying on the line 2𝑥 + 𝑦 = 5
(d) None of these
8. If 𝑎⃗ and 𝑏⃗⃗ are unit vectors then which of the following values of 𝑎⃗. 𝑏⃗⃗ is not possible
√3 1 1
(a) √3 (b) (c) (d ) − 2
2 √2
129

1
9. The value of ∫0 √𝑥(1 − 𝑥) 𝑑𝑥 equals
𝜋 𝜋 𝜋 𝜋
(a) 2
(b) 4
(c) 6 (d) 8

10. If 𝐴 = [𝑎𝑖𝑗 ] is a matrix of order 2 × 2 , such that |𝐴| = −15 and 𝐶𝑖𝑗 represent cofactor of 𝑎𝑖𝑗
the 𝑎21 𝐶21 + 𝑎22 𝐶22 equals
(a) 0 (b)-15 (c) 15 (d ) 225
11. The corner points of the feasible region determined by the system of linear constraints are (0,
10), (5, 5), (15, 15), (0, 20). Let 𝑍 = 𝑝𝑥 + 𝑞𝑦, where 𝑝, 𝑞 > 0. Condition on 𝑝 and 𝑞 so that
the maximum of 𝑍 occurs at both the points (15, 15) and (0, 20) is
(𝑎) 𝑝 = 𝑞 (𝑏) 𝑝 = 2𝑞 (𝑐) 𝑞 = 2𝑝 (𝑑) 𝑞 = 3𝑝

12. If |𝑥 + 1 𝑥 − 1 𝑥 − 3 𝑥 + 2 | = |4 − 1 1 3 | then the value of x is


(a) 2 (b) 3 (c) 0 (d ) 4

13. If 𝐴 = [𝑎 0 0 0 𝑎 0 0 0 𝑎 ] , then the value of |𝑎𝑑𝑗 𝐴| is


(a) 𝑎27 (b) 𝑎6 (c) 𝑎9 (d) 𝑎2
𝐴
14. If A and B are two independent events such that 𝑃(𝐴) = 0.3 ,𝑃(𝐴 ∪ 𝐵) = 0.5 then 𝑃 (𝐵) −
𝐵
𝑃 (𝐴 ) =

(a) 2/7 (b) 3/35 (c) 1/70 (d) 1/7


𝑦 𝑑𝑥−𝑥 𝑑𝑦
15. The general solution of differential equation = 0, is
𝑦

(a) 𝑥𝑦 = 𝐶 (b) 𝑥 = 𝐶𝑦 2 (c) 𝑦 = 𝐶𝑥 (d) 𝑦 = 𝐶𝑥 2


𝑑2 𝑥
16. If 𝑥 = 𝐴𝑐𝑜𝑠 4𝑡 + 𝐵 𝑠𝑖𝑛 𝑠𝑖𝑛 4𝑡, then =
𝑑𝑡 2

(a) −𝑥 (b) 𝑥 (c) −16𝑥 (d ) 16x


2 2
17. If |𝑎⃗ × 𝑏⃗⃗| + |𝑎⃗. 𝑏⃗⃗| = 400 and |𝑎⃗| = 5, then the value of |𝑏⃗⃗|=
(a) 2 (b) 3 (c) ±4 (d ) 4
18. P is a point on the line segment joining the points (3, 2, –1) and (6, 2, –2). If x co-ordinate of
P is 5, then its y co-ordinate is
(a) 2 (b) 1 (c) –1 (d) –2

ASSERTION-REASON BASED QUESTIONS


In the following questions, a statement of assertion (A) is followed by a statement of
Reason (R). Choose the correct answer out of the following choices.

(a) Both A and R are true and R is the correct explanation of A.


(b) Both A and R are true but R is not the correct explanation of A.
(c) A is true but R is false. (d). A is false but R is true.
130

1 1
19. Assertion (A):2𝑥 is defined only if − 2 ≤ 𝑥 ≤ 2

Reason (R): Domain of 𝑥 is −1 ≤ 𝑥 ≤ 1


𝑥+2 2𝑦−7 5−𝑧
20. Assertion (A):Direction ratios of the line 2
= 6
= 6
are 2,6, −6
𝑥−𝑥1 𝑦−𝑦1 𝑧−𝑧1
Reason (R): Direction ratios of the line = = are a,b,c.
𝑎 𝑏 𝑐

SECTION B
Very short answer type-questions (VSA) of 2 marks each
2𝜋
21. Find the value of 𝑐𝑜𝑠 𝑐𝑜𝑠 3
4𝑥
OR Let 𝑓: 𝑅 − {−4/3} → 𝑅 be a function defined as 𝑓(𝑥) = .
3𝑥+4

Show that 𝑓: 𝑅 − {−4/3} → 𝑅𝑎𝑛𝑔𝑒𝑓 is one one and onto.


22. A stone is dropped into a quite lake and waves moves in circles at the speed of 5cm /s. At
the instant when the radius of the circular wave is 8 cm, how fast is enclosed area
increasing?
23. Find the value of 𝑎⃗. 𝑏⃗⃗ if |𝑎⃗| = 10, |𝑏⃗⃗| = 2 and |𝑎⃗ × 𝑏⃗⃗| = 16
OR
Find Cartesian equation of the line which passes through the point (−2,4, −5) and is parallel
𝑥+3 4−𝑦 𝑧+8
to line = = .
3 5 6
𝜋 𝑥 𝑑𝑦
24. If 𝑦 =𝑙𝑜𝑔 𝑙𝑜𝑔 𝑡𝑎𝑛 𝑡𝑎𝑛 ( + ) show that =𝑠𝑒𝑐 𝑠𝑒𝑐 𝑥.
4 2 𝑑𝑥

25. Find |𝑎⃗| 𝑎𝑛𝑑 |𝑏⃗⃗| if |𝑎⃗| = 2 |𝑏⃗⃗| and (|𝑎⃗| + |𝑏⃗⃗|). (|𝑎⃗| − |𝑏⃗⃗|) = 12

SECTION C (short answer type questions (SA) of 3 marks each)


𝑥−1
26. Find ∫ (𝑥−2)(𝑥−3)
𝑑𝑥

27. A random variable X has the following probability distribution


X 0 1 2 3 4 5 6

P(X) K 2k 2k 3k K2 2k2 7 k2 + k

Find the value of the k and mean of the distribution.


Or
A bag X contains 4 white balls and 2 black balls, while another bag Y contains 3 white balls
and 3 black balls. Two balls are drawn (without replacement) at random from one of the bags
and were found to be one white and one black. Find the probability that the balls were drawn
from bag Y.
131
𝜋
𝜋 𝑥𝑠𝑖𝑛𝑠𝑖𝑛 𝑥 1−𝑠𝑖𝑛𝑠𝑖𝑛 2𝑥
28. Evaluate ∫0 1+3𝑥
𝑑𝑥 OR Evaluate 2
∫−𝜋 𝑒2𝑥 (1−𝑐𝑜𝑠𝑐𝑜𝑠 2𝑥
) 𝑑𝑥
2
𝑑𝑦
29. Solve the differential equation (1 + 𝑥 2 ) + 𝑦 = 𝑒𝑥 .
𝑑𝑥
𝑑𝑥
OR Solve the differential equation (𝑦 + 3𝑥 2 ) 𝑑𝑦 = 𝑥.

30. Solve the following linear programming problem graphically.


Maximize Z=80x+120y subject to the constraints 3𝑥 + 4𝑦 ≤ 60 , 𝑥 + 3𝑦 ≤ 30 , 𝑥, 𝑦 ≥ 0.
𝑑𝑥
31. Evaluate ∫ 𝑠𝑖𝑛𝑠𝑖𝑛 (𝑥−𝑎)𝑐𝑜𝑠 (𝑥−𝑏)

SECTION D
(This section comprises of long answer type questions (LA) of 5 marks each)

32. Make a rough sketch of the region {(𝑥, 𝑦); 𝑥 2 + 𝑦 2 ≤ 4, 𝑥 + 𝑦 ≥ 2} and find area of the region
using integration.
33. Define the relation R in the set 𝑁 × 𝑁 as follows:
For (a, b), (c, d) ∈ 𝑁 × 𝑁, (a, b) R (c, d) iff ad (b + c) = bc (a + d). Prove that R is an
equivalence relation in 𝑁 × 𝑁.
34. The equation of motion of a missile are 𝑥 = 3𝑡, 𝑦 = −4𝑡, 𝑧 = 𝑡, where the time t is given in
seconds and distance is measured in km. What is the path of the missile. At what distance
will the rocket be from the starting point (0,0,0) in 5 second?If the position of missile at a
certain instant of time is (5,-8,10) then what will be the height of the missile from the ground
(xy-plane).
35. Find A-1 where 𝐴 = [1 1 1 1 2 − 3 2 − 1 3 ] Hence solve the system of equations

x + y + 2 z = 0, x + 2 y − z = 9, and x − 3 y + 3 z = −14.

SECTION E
(This section comprises of 3 case-study/passage-based questions of 4 marks each with two
sub-parts. First two case study questions have three sub-parts (i), (ii), (iii) of marks 1, 1, 2
respectively. The third case study question has two sub-parts of 2 marks each.)

36. Case Study 1:


132

The Relation between the height of the plant (y in cm) with respect to exposure to sunlight is
1
governed by the equation 𝑦 = 4𝑥 − 2 𝑥 2 where x is the number of days exposed to sunlight.

(i) Find the rate of growth of the plant with respect to sunlight .

(ii) What is the number of days it will take for the plant to grow to the maximum
eight?
(iii) If the height of the plant is 7/2 cm then find the number of days it has been exposed
to the sunlight .

37. Case study 2:

Mr.Shashi who is an architect, designs a building for a small company. The design of window on the
ground floor is proposed to be different than other floors. The window is in the shape of a rectangle
which is surmounted by a semicircular opening. This window is having a perimeter of 10m as shown
below :Based on the above information answer the following questions.
(i) If 2x and 2y are the length and breadth of the rectangular portion of the window, then find the
relation between variables
(ii) Find the combined area A of rectangular region and semi-circular region of the window
in terms of x.
(iii)Find the maximum value of area A, of the whole window.
38. Case study 3:
133

The reliability of a COVID PCR test is specified as follows:


Of people having COVID, 90% of the test detects the disease but 10% goes undetected. Of people
free of COVID, 99% of the test is judged COVID negative but 1% are diagnosed as showing COVID
positive. From a large population of which only 0.1% have COVID, one person is selected at random,
given the COVID PCR test, and the pathologist reports him/her as COVID positive.
Based on the above information, answer the following

(i) What is the probability of the ‘person to be tested as COVID positive’ given that ‘he is
actually having COVID?

(ii) What is the probability that the ‘person is actually not having COVID?
**************************
134

CLASS XII SAMPLE PAPER-06 MATHEMATICS


1. This Question paper contains - five sections A, B, C, D and E. Each
section is compulsory. However, there are internal choices in some
questions.
2. Section A has 18 MCQ’s and 02 Assertion-Reason based questions of 1 mark
each.
3. Section B has 5 Very Short Answer (VSA)-type questions of 2 marks each.
4. Section C has 6 Short Answer (SA)-type questions of 3 marks each.
5. Section D has 4 Long Answer (LA)-type questions of 5 marks each.
6. Section E has 3 source based/case based/passage based/integrated
units of assessment (4 marks each) with sub parts.

SECTION A
1 If A = [3 𝑥 + 1 2𝑥 + 3 𝑥 + 2 ] is a symmetric matrix , then x is

(a) 4 (b)2 ( c ) -4 ( d ) -Z
2 ′
If A = [ a ij ] m X n , then 𝐴 is equal to
(a). 0[ a ji ] n X m (b) [ a ij ] m X n (c) [ a ji ] m X n (d) [ a ij ] n X m
3 ̂ + 𝑗̂and 𝑘̂+ 𝑖̂ is
The area of the parallelogram whose diagonals are 𝑘
√3 3
(a) (b) 2 (c) 3 (d)√3
2

4 𝑠𝑖𝑛𝑠𝑖𝑛 3𝑥 𝑘
The function f(x) ={ 𝑥
, 𝑥 ≠ 0 2 , 𝑥 = 0 , then the value of k is

(a) 3 (b) 6 (c) 9 (d)12


5
∫ 𝑒𝑥 𝑠𝑒𝑐𝑥(1 + 𝑡𝑎𝑛𝑥)𝑑𝑥 = ⋯

(𝑎)𝑒 𝑥 𝑐𝑜𝑠𝑥 + 𝑐 (𝑏)𝑒 𝑥 𝑠𝑒𝑐𝑥 + 𝑐 (c) 𝑒 𝑥 𝑠𝑖𝑛𝑥 + 𝑐 (d) 𝑒 𝑥 𝑡𝑎𝑛𝑥 + 𝑐

6 𝑑𝑦
Integrating factors of the differential equation cos x + ysin x=1, is
𝑑𝑥

(a) sin x (b) sec x (c) tan x (d) cos x


7 The graph of the inequality 2x+3y > 6 is:

(a) half plane that contains the origin


(b) half plane that neither contains the origin nor the points of
the line 2x+3y =6
(c) whole XOY-plane excluding the points on the line 2x+3y =6
(d) entire XOY-plane
135

8 The area of a parallelogram whose adjacent sides represented by the


̂ and 4𝑖̂ + 2𝑗̂ is
vectors 2𝑖̂ − 3𝑘

(a) 10 (b)14 (c) √11 (d)


4√14
𝜋
9 √𝑠𝑖𝑛𝑥
∫02 𝑑𝑥 =…
√𝑠𝑖𝑛𝑥+√𝑐𝑜𝑠𝑥
𝜋 𝜋 𝜋
(a) (b) (c) (d) 𝜋
2 3 4

10 The number of all possible matrices of order 3 X 3 will each entry 0 or 1 is


(a) 27 (b) 18 (c) 81 (d) 512.
11
Z =3x + 4y, Subject to the constraints x+y 1, x,y ≥0.the shaded region shown in
the figure as OAB is bounded and the coordinates of corner points O, A and B are
(0,0),(1,0) and (0,1), respectively.

The maximum value of Z is 2.

(a) True (b) false (c) can’t say (d) partially true
12 If the area of a triangle with vertices (-3, 0), (3, 0) and (0, k) is 9 sq
units. Then the value of k will be
(a)9 (b)3 (c)-9 (d)6

13 𝑥−5 𝑦+4
The vector equation of the symmetrical form of equation of straight line = =
3 7
𝑧−6
is (a) 𝑟⃗⃗⃗ = (3𝑖 + 7𝑗 + 2𝑘) + 𝜇(5𝑖 + 4𝑗 − 6𝑘)
2
(b) ⃗⃗⃗
𝑟 = (5𝑖 + 4𝑗 − 6𝑘) + 𝜇(3𝑖 + 7𝑗 + 2𝑘)
(c) ⃗⃗⃗
𝑟 = (5𝑖 − 4𝑗 − 6𝑘) + 𝜇(3𝑖 − 7𝑗 − 2𝑘)
(d) ⃗⃗⃗
𝑟 = (5𝑖 − 4𝑗 + 6𝑘)+ 𝜇(3𝑖 + 7𝑗 +
2𝑘)

14 𝐴
𝐼𝑓 𝑃(𝐴 ∩ 𝐵) = 70%, 𝑃(𝐵) = 85%, 𝑡ℎ𝑒𝑛 𝑃 ( ) 𝑖𝑠 𝑒𝑞𝑢𝑎𝑙 𝑡𝑜
𝐵
14 17 7 1
(a) (b) (c ) (d)
17 20 8 8
136

15 1
The principal value of 𝑠𝑖𝑛−1 (− ) 𝑖𝑠
2

𝜋 𝜋 𝜋 5𝜋
(a) (b)- (c ) − (d)
3 3 6 6
16 𝑑𝑦 𝑦
The general solution of the differential equation = is
𝑑𝑥 𝑥

(a) log y=k x (b) y= k x (c) x y=k (d) y=k log x


17 ̂ and -𝑖̂ +
The unit vector perpendicular to both the vectors 𝑖̂ + 2𝑗̂ − 2𝑘
1 1
2 𝑗̂ + 2𝑘̂ (a) (2𝑖̂ − 𝑘̂ ) (b) (−2𝑖̂ − 𝑘̂ )
√5 √5
1 1
(c) (2𝑖̂ + 𝑗̂ + 𝑘̂) (d) (2𝑖̂ + 𝑘̂ )
√5 √5

18 A line makes angles of 45° and 60° with the positive axes of X and Y
respectively. The angle made by the same line with the positive axis of
Z, is.
(a )30° or 60° (b) 60° or 90° (c) 90° or 120° (d) 60° or 120°
19 Assertion (A): If x = at2 and y = 2at where ‘t’ is the parameter and ‘a’ is a
𝑑2𝑦 −1
constant, then = 𝑡
𝑑𝑥 2 𝑡2

𝑑2𝑦 𝑑2𝑦 𝑑2𝑥


Reason (R): = ÷
𝑑𝑥 2 𝑑𝑡 2 𝑑𝑡 2

a) A and R are true and R is the correct explanation of A


b) Both A and R are true but R is NOT the correct explanation of A.
c) A is Both true but R is false
d) A is false but R is true (e)Both A and R are false
20 Assertion A : If A =[0 0 1 0 1 2 0 0 4 ], 𝑡ℎ𝑒𝑛 |3𝐴| = 9|𝐴|

Reason R : If A is a square matrix of order n then,|𝑘𝐴| =𝑘 𝑛 |𝐴|

a) A and R are true and R is the correct explanation of A


b) Both A and R are true but R is NOT the correct explanation of A.
c) A is Both true but R is false
d) A is false but R is true (e). Both A and R are
false
SECTION B
21 −17𝜋
Find the value of (𝑆𝑖𝑛 ( )) OR
8

If f: R→ 𝑅 be a function defined by f(x) = x4, check whether f(x) is one-one and onto.
137

22 Sand is pouring from a pipe at the rate of 12𝑐𝑚3 /sec.The falling sand

forms a cone on the ground in such a way that the height of the cone

is always one-sixth of the radius of the base. How fast is the height of

the sand cone increasing when the height is 4cm.


23 Find a vector of magnitude 7 in the direction of vector 𝐴𝐵 where co-
ordinates of points A and B are (2,-1,3) and (3,2,-1) respectively.

OR Find 𝜆 when projection of 𝑎⃗ = 𝜆𝑖 ∧ + 𝑗 ∧ + 4𝑘 ∧ on

𝑏⃗⃗ = 2𝑖 ∧ + 6𝑗 ∧ + 3𝑘 ∧ is 4 units.
24 dy
Find dx if y = x .
x y

25 →
Find the value of p, if (2 î + 6 ĵ +27 k̂ )  ( î +3 ĵ + p k̂ ) = 0

SECTION C
26 1+𝑠𝑖𝑛𝑥
Integrate: ex ( )
1+𝑐𝑜𝑠𝑥

27 In a test an examinee either guesses or copies or knows the answer to a


multiple choice questions with four choices. The probability that he makes a
1 1
guess is and the probability that he copied the answer is . The probability
3 6
1
that his answer is correct, given that he copied it is . Find the probability
8
that he knew the answer to the question, given that he correctly answered it.

OR From a lot of 15 bulbs which include 5 defectives, a sample of 4


bulbs is drawn one by one with replacement. Find the probability
distribution of number of defective bulbs. Hence find the mean of the
distribution.
28 2 1 𝑙𝑜𝑔(1+𝑥)
Evaluate: ∫−1|𝑥 3 − 3𝑥 2 + 2𝑥| 𝑑𝑥 OR ∫0 𝑑𝑥
1+𝑥 2

29 Solve: (𝑥 2 − 𝑦 2 )𝑑𝑥 + 2𝑥𝑦𝑑𝑦 = 0 𝑔𝑖𝑣𝑒𝑛 𝑡ℎ𝑎𝑡 𝑦 = 1, 𝑤ℎ𝑒𝑛𝑥 = 1.

OR Solve the differential equation (𝑡𝑎𝑛−1 𝑦– 𝑥) 𝑑𝑦 = (1 + 𝑦 2 ) 𝑑𝑥.


138

30 Solve the following LPP graphically: Maximise Z = 3x + 9y, subject to


the constraints: x+3y≤ 60

x+y≥ 10 , x≤ 𝑦 , x≥ 0 , 𝑦 ≥ 0
31 x2 + x +1
 (x + 2) x 2 + 1 dx
( )
Evaluate

SECTION D
32 Using integration, find the area of the region in the first quadrant enclosed
by the x-axis and 𝑥 = √3𝑦 by the circle 𝑥 2 + 𝑦 2 = 4.
33 Show that the relation R in the set {1, 2, 3} given by R = {(1, 2), (2, 1)} is
symmetric, but is neither reflexive nor transitive.

OR Show that the relation R in the set A = {1, 2, 3, 4, 5} is given by

S = {(a, b) : |a – b| is divisible by 2 } is an equivalence relation.


34 Find the shortest distance between the lines

𝑟⃗ = (4𝑖 ∧ − 𝑗 ∧ ) + 𝜆(𝑖 ∧ + 2𝑗 ∧ − 3𝑘 ∧ ) 𝑎𝑛𝑑

𝑟⃗ = (𝑖 ∧ − 𝑗 ∧ + 2𝑘 ∧ ) + 𝜇(2𝑖 ∧ + 4𝑗 ∧ − 5𝑘 ∧ )

OR A line passes through (2,-1,3) and is perpendicular to the lines

𝑟⃗ = (𝑖 ∧ + 𝑗 ∧ − 𝑘 ∧ ) + 𝜆(2𝑖 ∧ − 2𝑗 +∧ 𝑘 ∧ ) 𝑎𝑛𝑑

𝑟⃗ = (2𝑖 ∧ − 𝑗 ∧ − 3𝑘 ∧ ) + 𝜇(𝑖 ∧ + 2𝑗 ∧ + 2𝑘 ∧ )

Obtain its equation in the vector and the cartesian form


35 Find the product

1 −1 2   −2 0 1 
0 2 −3 .  9 2 −3
   
 3 −2 4   6 1 −2 
.Use the product to solve the system of

equations x − y + 2 z = 1; 2 y − 3z = 1; and ,3x − 2 y + 4 z = 2

SECTION E
139

35 CASE STUDY 1

The owner of a small restaurant can prepare a particular meal at a cost of


rupees 100. He estimates that if the menu price of the meal is x rupees,
then the number of customers who will order that meal at that price in an
evening is given by the function D(x) = 200-x.

1. His day revenue as function of x can be expressed as


2. Express the total cost of the meal as function of x
0R Express the profit function in terms of x

3. What is the domain of the function D(x) = 200-x

37 CASE STUDY 2 : A society constructs a rectangular tank to fulfil water supply for its
residents on a piece of land 8 X 3 m2 in such a way that the length and breadth of tank
are less than length and breadth of land respectively by twice of its height x.

1. Express the Volume of the tank V as function of x


2. For having maximum volume of the tank, what should be the value of x
3. What are the dimensions of tank constructed
OR What is the maximum volume of the tank ?
140

MATHEMATICS CLASS XII SAMPLE PAPER-07


General Instructions :
1. This Question paper contains - five sections A, B, C, D and E. Each section is compulsory. However,
there are internal choices in some questions.
2. Section A has 18 MCQ’s and 02 Assertion-Reason based questions of 1 mark each.
3. Section B has 5 Very Short Answer (VSA)-type questions of 2 marks each.
4. Section C has 6 Short Answer (SA)-type questions of 3 marks each.
5. Section D has 4 Long Answer (LA)-type questions of 5 marks each.
6. Section E has 3 source based/case based/passage based/integrated units of assessment (4 marks
each) with sub parts.
SECTION A: ( each question carries 1 mark)

1 If A and B are symmetric matrices of order nxn, then AB – BA is

A) null matrix B) identity matrix C) symmetric matrix D) skew symmetric

matrix

2 x +1 x −1 4 −1
=
If x − 3 x+2 1 3 , then find the value of x.

A) 1 B) 2 C) 3 D) 4

3 2  −3
A =  0 2 5 
1 1 3  , then find the value of λ for which A-1 exists.
If

A) λ≠8/5 B) λ≠ -8/5 C) λ≠5/8 D) λ≠ -5/8

4 ˆ ˆ
The projection of a = 2iˆ + ˆj − k on b = iˆ − 2 ˆj − 2k is

A) 2/3 B) 1/3 C)4/3 D) 5/3

5 If a is a unit vector and ( x − a )  ( x + a ) = 8 , then the value of x is

A) 3 B) 4 C) 9 D) 1
6 ˆ ˆ ˆ
The value of ˆj  ˆj  k + iˆ  iˆ  k + k  iˆ  ˆj is ___
141

A) 0 B) 1 C) -1 D) 2

7 Find the value λ for which the vectors 𝑎⃗ = 𝑖̂ − 2 𝑗̂ + 3 𝑘̂ & 𝑏⃗⃗ = 3𝑖̂ + 𝜆 𝑗̂ + 9 𝑘̂ are
parallel to each other

A) 6 B) 2/3 C) -2/3 D) -6
If 𝐴 = [1 0 − 1 7 ], then find k if A2 = 8A +kI
8
A) -7 B) 7 C) 5 D) -5

9 If P(𝐴) = 0.4, P(B) = 0.2 and P(A/B) =0.5, then P(AUB) =

A) 0.7 B) 0.6 C) 0.5 D) 0.8

10  4 −1
A= 
If  4 −3 then find the value of |adj A|.

A) 8 B) -8 C) 16 D) -16

11 𝑑 𝑑𝑦
Sum of order and degree of differential equation 𝑑𝑥 (𝑑𝑥 )=0

A) 2 B) 3 C) 1 D) 4

12 𝑑𝑦
If 𝑑𝑥 =x2y + yn is homogeneous differential equation, then n =

A)2 B) 1 C) 3 D) 0

3x − 8, if x  5
13 f ( x) = 
If the function 2k , if x  5 is continuous, then find value of k.

A) 5/2 B) -5/2 C) 7/2 D) 9/2

14  sin x cos xdx =


A) -cos2x / 4 B) -cos2x / 2 C) cos2x / 4 D) cos2x / 2

15 4
dx
x 2
−9
Find the value of 2

A) log(5/7) B) log(7/5) C) log(7/5) / 6 D) log(5/7) / 6


16 dy
=
If y = xx , then dx
A) xx-1 B) xx + logx C) xx(1+logx) D) xx logx
142

17 The corner points of the shaded bounded feasible


region of an LPP are (0, 10), (4,4) and (6, 0) as shown
in the figure. The maximum value of the objective
function Z = 5x + 3y is

A) 30 B) 32 C) 28 D) 34
18 The solution set of the inequality 2y < x – 2 is
(a) an open half-plane not containing the origin.
(b) an open half-plane containing the origin.
(c) the whole XY-plane not containing the line 2y = x – 2
(d) a closed half plane containing the origin.

ASSERTION-REASON BASED QUESTIONS


In the following questions, a statement of assertion (A) is followed by a statement of
Reason (R). Choose the correct answer out of the following choices.
(a) Both A and R are true and R is the correct explanation of A.
(b) Both A and R are true but R is not the correct explanation of A.
(c) A is true but R is false.
(d) A is false but R is true.

19 Assertion: Domain of cot-1(x) is all real numbers


Reason: cot-1(-1) = -π/4
20 ˆ ˆ ˆ ˆ
Assertion: if a = 2i + j − k and b = iˆ − 2 ˆj − 2k then the vectors are intersecting.

Reason: if two vectors are parallel, then 𝑎⃗ = 𝑡𝑏⃗⃗

SECTION B: ( each question carries 2 marks)

 1− x + 1+ x 
21 tan −1  
Simplify to simplest form:  1 − x − 1 + x 

OR
If a function f:R+ 🡪A be defined as f(x) = 5x2 + 6x – 9. Find set A if the function is
ONTO.
A ladder of length 5m is sliding along the walls. The base is moving away at the rate of
22
5cm/sec. At what rate the height of the ladder is decreasing when the height is 4m.
143

dy y ( x − 1)
=
23
If xy = e( x − y ) , then show that dx x( y + 1)

24
Let 𝑎⃗ and 𝑏⃗⃗ be two vectors such that |𝑎 ⃗⃗⃗⃗ = √2/3 and a  b is a unit vector.
⃗⃗| = 3 and |𝑏|

Find the angle between the two vectors.

25 ˆ ˆ ˆ
Find the area of the parallelogram whose adjacent sides are a = 2i + j − k and

b = iˆ − 2 ˆj − 2kˆ .

ˆ ˆ ˆ ˆ
OR Find a unit vector perpendicular to both the vectors a = 2i + j − k & b = iˆ − 2 ˆj − 2k

SECTION C: ( each question carries 3 marks)


26 Solve the following Linear Programming Problem graphically:
Maximize Z = x – 7y + 9
subject to 𝑥 + 𝑦 ≤ 8, 𝑥 ≤ 5, y ≤ 5, x, 𝑦 ≥ 0

27 Integrate ∫ 𝑒𝑥 (1+𝑐𝑜𝑠𝑥
1−𝑠𝑖𝑛𝑥 ) 𝑑𝑥
28 
x sin x
3  1 + cos 2
x
dx
Evaluate ∫
−1
|𝑥 − 1|𝑑𝑥 OR Evaluate 0

29 Find the particular solution of the differential equation:

xdy + (y – x3)dx = 0, when x = 1, y = 1.

𝑑𝑦 𝑦
OR Solve the differential equation : 𝑥 𝑑𝑥 − 𝑦 + 𝑥𝑠𝑖𝑛 (𝑥 ) = 0

30 3x − 5
Evaluate
 ( x − 1) ( x − 2
2)
dx

31 Two dice are thrown simultaneously. If X denotes the number of sixes, find its
probability distribution and mean.
OR
144

A random variable X has the following probability distribution:

X 0 1 2 3 4 5 6 7

P(X) 0 k 2k 2k 3k k2 2k2 7k2 + k

Determine (i) k (ii) P(X < 3)

SECTION D: ( each question carries 5 marks)

32 Solve the system of equations by matrix method.


x – 2y – 2z = 9 x–y+z=4 2x + y + 3z = 1

33 Find the area of the region enclosed by x2 + y2 ≤ 8, x ≥ √2 and x ≤ 2.

34 Find the vector equation of the line passing through the point (1, 2, – 4) and

x − 8 y + 19 z − 10 x − 15 y − 29 z − 5
= = = =
perpendicular to the two lines: 3 −16 7 & 3 8 −5
35 Define the relation R in the set 𝑁 × 𝑁 as follows:
For (a, b), (c, d) ∈ 𝑁 × 𝑁, (a, b) R (c, d) iff a+d = b+c. Prove that R is an equivalence
relation in 𝑁 × 𝑁.
OR Let a relation R defined on A = {1,2,3,4,5,6,7,8,9,10} be defined as
R = {(a,b) : |a - b| is an even number }. Prove that the relation is equivalence. Hence
find all elements which are related to 3.

SECTION E:
(This section comprises of 3 case study/ passage based questions of 4 marks each with two sub-parts.
First two case study questions have three sub-parts (i),(ii),(iii) of marks 1, 1, 2 respectively. The third
case study question has two sub-parts of 2 marks each.)

36 An engineer wants to cast metal into solid half


cylinders of given volume V with a rectangular
base and semi-circular ends.

Based on the information provided answer the


following.

(i) find the total surface area ‘S’ of the casted half cylinder.
145

ii) find the expression for total surface area S in terms of V, volume of half cylinder
and r.
iii) Find the relation between V and r for which surface area S is minimum.
OR
iii) Find the minimum surface area of the half cylinder.
37 The Relation between the height of the plant (y in cm)
with respect to exposure to sunlight is governed by
the following equation y = 4x – x2/2 where x is the
number of days exposed to sunlight.

(i)What is the number of days it will take for the plant to grow to the maximum height?

(ii)What is the maximum height of the plant?


OR
(ii) What will be the height of the plant after 2 days?

(iii) If the height of the plant is 7/2 cm, the number of days it has been exposed to the
sunlight is

38 An insurance company insured 2000 scooter drivers, 4000

car drivers and 6000 truck drivers. The probability of an

accidents are 0.01, 0.03 and 0.15 respectively.

i) what is the probability that an insured person will meet an

accident ?

ii) One of the insured persons meets with an accident. What


is the probability that he is a scooter driver?

**********

You might also like